Re: [obm-l] off topic - calculo numerico

2013-11-04 Por tôpico Johann Dirichlet

MatLab, C++, Fortran, Octave... É só achar uma biblioteca do gênero por aí!

Em 04-11-2013 20:16, regis barros escreveu:

Olá Herman
Aqui na Unicamp usa dependendo do professor C++ ou Matlab; Na UFSCar, 
em física, em fortran.


Regis



Em Segunda-feira, 4 de Novembro de 2013 17:26, Hermann 
 escreveu:

OFF-TOPIC
Por favor, alguém sabe qual o programa que usam para estudar, 
atualmente, o cálculo numérico?

abraços
Herman

--
Esta mensagem foi verificada pelo sistema de antivírus e
acredita-se estar livre de perigo.



--
Esta mensagem foi verificada pelo sistema de antivírus e
acredita-se estar livre de perigo. 



--
Esta mensagem foi verificada pelo sistema de antivírus e
acredita-se estar livre de perigo.



Re: [obm-l] Quadrados em PA

2013-11-01 Por tôpico Johann Dirichlet

Em 18-10-2013 09:19, marcone augusto araújo borges escreveu:

Determine três números inteiros distintos cujos quadrados estão em PA
[Upload Photo to Facebook]
[Google+]
[Twitt]
[Send by Gmail]
[Upload Video to Facebook]
[Google+]
[Twitt]
[Send by Gmail]


Infintas, infinitas soluções.


x^2+y^2=2z^2 é equivalente a 'quadrados em PA'.

Como x e y têm a mesma paridade, podemos escrever x=a+b, y=a-b.

(a+b)^2+(a-b)^2=2(a^2+b^2)
Assim, a^2+b^2=z^2

E as boas e velhas trincas pitagóricas voltam com tudo!


--
Esta mensagem foi verificada pelo sistema de antivírus e
acredita-se estar livre de perigo. 



--
Esta mensagem foi verificada pelo sistema de antivírus e
acredita-se estar livre de perigo.



Re: [obm-l] Álgebra

2013-10-05 Por tôpico Johann Dirichlet

Em 22-09-2013 21:31, marcone augusto araújo borges escreveu:

Sejam x,y inteiros positivos tais que 3x^2 + x = 4y^2 + y.Mostre que
[Upload Photo to Facebook]
[Google+]
[Twitt]
[Send by Gmail]
[Upload Video to Facebook]
[Google+]
[Twitt]
[Send by Gmail]
x - y é um quadrado perfeito.
Estou tentando.Uma ajuda?



Que tal substituir x=d+y e provar que d é quadrado perfeito? 
Provavelmente você precisará resolver uma equação de Pell, mas é bem 
mais provável que um teste simples de divisibilidade seja mais rápido...







--
Esta mensagem foi verificada pelo sistema de antivírus e
acredita-se estar livre de perigo. 



--
Esta mensagem foi verificada pelo sistema de antivírus e
acredita-se estar livre de perigo.



Re: [obm-l] Contagem(?)

2013-08-31 Por tôpico Johann Dirichlet

Em 30-08-2013 21:58, marcone augusto araújo borges escreveu:

De quantas maneiras podemos escrever 2010 como soma de dois
[Upload Photo to Facebook]
[Google+]
[Twitt]
[Send by Gmail]
[Upload Video to Facebook]
[Google+]
[Twitt]
[Send by Gmail]
inteiros positivos primos entre si?

Eu tirei todas as possíveis parcelas que fossem um múltiplo dos 
divisores de 2010,mas achei 528 e o gabarito dá 264.





Se x+y=2010 e d|x e d|y, então d|2010. Basta então contar todos os 
números primos com 2010 e menores que 2010 - e dividir o total por 2.


Isto é a infame função phi de Euler!



--
Esta mensagem foi verificada pelo sistema de antivírus e
acredita-se estar livre de perigo. 



--
Esta mensagem foi verificada pelo sistema de antivírus e
acredita-se estar livre de perigo.



[obm-l] Re: [obm-l] Re: [obm-l] RE: [obm-l] Re: [obm-l] Teoria dos números

2013-08-31 Por tôpico Johann Dirichlet

Em 30-08-2013 10:29, Ralph Teixeira escreveu:


Ah, droga, bobeei. Nao ajudou tanto quanto eu achava... :-( :-(



Mas o caminho deve ser este. Que tal o famigerado módulo 49? Afinal esse 
monte de primos incita raízes primitivas...


On Aug 29, 2013 12:23 PM, "marcone augusto araújo borges" 
mailto:marconeborge...@hotmail.com>> wrote:


7^x modulo 9 dá 1,7 e 4 e 3^y dá quase sempre 0
O que interessa  para 7^x modulo 9 é 4,o que ocorre apenas quando
x é da forma 3.k + 2
Como x tambem é ímpar,só pode ser da forma 6.n + 5,mas...


Date: Thu, 29 Aug 2013 09:21:24 -0300
Subject: [obm-l] Re: [obm-l] Teoria dos números
From: ralp...@gmail.com 
To: obm-l@mat.puc-rio.br 

Tente agora modulo 9.

On Aug 28, 2013 9:50 PM, "marcone augusto araújo borges"
mailto:marconeborge...@hotmail.com>>
wrote:

Eu já postei a questão aqui,mas infelizmente não obtive resposta.
Sei que vão aparecendo outras questões interessantes e por
isso peço licença para reapresentá-la
Determine todos os inteiros positivos x,y tais que 7^x - 3^y = 4

Claro que x = 1 e y = 1 satisfaz(desconfio que seja a unica
solução)
Eu só consegui concluir que x e y são ímpares,analisando módulo 4.
Desde já agradeço.
[Upload Photo to Facebook]
[Google+]
[Twitt]
[Send by Gmail]
[Upload Video to Facebook]
[Google+]
[Twitt]
[Send by Gmail]


-- 
Esta mensagem foi verificada pelo sistema de antivírus e

acredita-se estar livre de perigo.


-- 
Esta mensagem foi verificada pelo sistema de antivírus e

acredita-se estar livre de perigo.

-- 
Esta mensagem foi verificada pelo sistema de antivírus e

acredita-se estar livre de perigo.


--
Esta mensagem foi verificada pelo sistema de antivírus e
acredita-se estar livre de perigo. 



--
Esta mensagem foi verificada pelo sistema de antivírus e
acredita-se estar livre de perigo.



Re: [obm-l] Teoria dos números

2013-08-31 Por tôpico Johann Dirichlet

Em 28-08-2013 21:45, marcone augusto araújo borges escreveu:

Eu já postei a questão aqui,mas infelizmente não obtive resposta.
Sei que vão aparecendo outras questões interessantes e por isso peço 
licença para reapresentá-la

Determine todos os inteiros positivos x,y tais que 7^x - 3^y = 4

Claro que x = 1 e y = 1 satisfaz(desconfio que seja a unica solução)
Eu só consegui concluir que x e y são ímpares,analisando módulo 4.
Desde já agradeço.
[Upload Photo to Facebook]
[Google+]
[Twitt]
[Send by Gmail]
[Upload Video to Facebook]
[Google+]
[Twitt]
[Send by Gmail]



Módulo 7, temos o quê?

-3^y = 4-7

3^y = 3
3^(y-1) = 1

Temos 3^3=27=-1, logo 6|(y-1).

Supondo y>1, vamos tentar um módulo 9?

7^x=4
(-2)^x=4

Vou confiar em tu - x é ímpar:

-2^x=4
-2^(x-2)=1
2^(x-2)=-1=9-1=8=2^3
2^(x-5)=1

Temos
2^1=2
2^2=4
2^3=8
2^4=7
2^5=5
2^6=1

Logo 6|x+1


Que coisa! Ainda falta...






--
Esta mensagem foi verificada pelo sistema de antivírus e
acredita-se estar livre de perigo. 



--
Esta mensagem foi verificada pelo sistema de antivírus e
acredita-se estar livre de perigo.



Re: [obm-l] Re: [obm-l] Problema de lógica

2013-08-18 Por tôpico Johann Dirichlet

Em 18-08-2013 08:58, Artur Costa Steiner escreveu:


Eu apontaria para uma carta qualquer e perguntaria:

Se eu lhe perguntasse se esta carta é um ás, você diria que sim?



O problema é se esta carta for um ás. Aí você não tem como saber qual é 
a outra carta - pode ser valete ou ás.
Realmente, perguntar sobre outra carta é a estratégia. Eu apontaria a 
primeira e perguntaria sobre a carta da outra ponta.



Artur

Em 17/08/2013 21:30, "Mauricio de Araujo" 
mailto:mauricio.de.ara...@gmail.com>> 
escreveu:


Eu disponho de três cartas de baralho, dois ases e um valete, e as
disponho sobre uma mesa com as faces voltadas para baixo, uma ao
lado da outra. Antes de virar as faces, eu anotei a posição de
cada uma das cartas, de maneira que eu sei onde os ases e o valete
estão.
Tua missão é identificar um dos ases me fazendo apenas uma
pergunta que admita apenas SIM-NÃO como resposta. Para isso, você
deverá apontar para uma das cartas. Se a carta para a qual você
apontar for um ás, a resposta à tua pergunta será sincera (eu não
mentirei); se a carta for um valete, a resposta poderá ser sim ou
não, aleatoriamente.

-- 
Abraços


oɾnɐɹɐ ǝp oıɔıɹnɐɯ
/momentos excepcionais pedem ações excepcionais./
/Os cemitérios estão cheios de pessoas insubstituíveis em seus
ofícios./

-- 
Esta mensagem foi verificada pelo sistema de antivírus e
acredita-se estar livre de perigo. 



--
Esta mensagem foi verificada pelo sistema de antiv�rus e
acredita-se estar livre de perigo. 



--
Esta mensagem foi verificada pelo sistema de antiv�rus e
acredita-se estar livre de perigo.



Re: [obm-l] Re: [obm-l] Problema de lógica

2013-08-17 Por tôpico Johann Dirichlet

Em 17-08-2013 21:47, Mauricio de Araujo escreveu:
Sim, a pergunta não precisa ter relação com a carta para a qual você 
apontou... esta apenas vai orientar se a resposta vai ser sincera ou 
aleatória...




Então a resposta vai ter que, pelo menos, forçar o cabra a falar a 
verdade a qualquer custo. Eu me lembro de um problema parecido:


Mulheres de óculos escuros respondiam perguntas. Se os olhos fossem 
azuis, elas falavam a verdade. Se fossem castanhos, mentira. Como saber 
a cor dos olhos de cada uma?


A pergunta para fazer a elas era algo como "Se eu te perguntasse qual a 
cor dos olhos de cada uma de vós, qual seria a sua resposta?".






2013/8/17 Johann Dirichlet <mailto:peterdirich...@gmail.com>>


Em 17-08-2013 21:24, Mauricio de Araujo escreveu:

Eu disponho de três cartas de baralho, dois ases e um valete, e
as disponho sobre uma mesa com as faces voltadas para baixo, uma
ao lado da outra. Antes de virar as faces, eu anotei a posição de
cada uma das cartas, de maneira que eu sei onde os ases e o
valete estão.
Tua missão é identificar um dos ases me fazendo apenas uma
pergunta que admita apenas SIM-NÃO como resposta. Para isso, você
deverá apontar para uma das cartas. Se a carta para a qual você
apontar for um ás, a resposta à tua pergunta será sincera (eu não
mentirei); se a carta for um valete, a resposta poderá ser sim ou
não, aleatoriamente.



O apontar para a carta é independente da pergunta, certo? No
seguinte sentido: eu aponto para a carta do meio, mas pergunto se
a primeira carta é um valete. Pode ser?

-- 
Abraços


oɾnɐɹɐ ǝp oıɔıɹnɐɯ
/momentos excepcionais pedem ações excepcionais./
/Os cemitérios estão cheios de pessoas insubstituíveis em seus
ofícios./

-- 
Esta mensagem foi verificada pelo sistema de antiv�rus e
acredita-se estar livre de perigo. 



-- 
Esta mensagem foi verificada pelo sistema de antivírus e

acredita-se estar livre de perigo.




--
Abraços

oɾnɐɹɐ ǝp oıɔıɹnɐɯ
/momentos excepcionais pedem ações excepcionais./
/Os cemitérios estão cheios de pessoas insubstituíveis em seus ofícios./

--
Esta mensagem foi verificada pelo sistema de antiv�rus e
acredita-se estar livre de perigo. 



--
Esta mensagem foi verificada pelo sistema de antiv�rus e
acredita-se estar livre de perigo.



Re: [obm-l] Problema de lógica

2013-08-17 Por tôpico Johann Dirichlet

Em 17-08-2013 21:24, Mauricio de Araujo escreveu:
Eu disponho de três cartas de baralho, dois ases e um valete, e as 
disponho sobre uma mesa com as faces voltadas para baixo, uma ao lado 
da outra. Antes de virar as faces, eu anotei a posição de cada uma das 
cartas, de maneira que eu sei onde os ases e o valete estão.
Tua missão é identificar um dos ases me fazendo apenas uma pergunta 
que admita apenas SIM-NÃO como resposta. Para isso, você deverá 
apontar para uma das cartas. Se a carta para a qual você apontar for 
um ás, a resposta à tua pergunta será sincera (eu não mentirei); se a 
carta for um valete, a resposta poderá ser sim ou não, aleatoriamente.




O apontar para a carta é independente da pergunta, certo? No seguinte 
sentido: eu aponto para a carta do meio, mas pergunto se a primeira 
carta é um valete. Pode ser?



--
Abraços

oɾnɐɹɐ ǝp oıɔıɹnɐɯ
/momentos excepcionais pedem ações excepcionais./
/Os cemitérios estão cheios de pessoas insubstituíveis em seus ofícios./

--
Esta mensagem foi verificada pelo sistema de antiv�rus e
acredita-se estar livre de perigo. 



--
Esta mensagem foi verificada pelo sistema de antiv�rus e
acredita-se estar livre de perigo.



[obm-l] Re: [obm-l] RE: [obm-l] Números inteiros

2011-09-23 Por tôpico Johann Dirichlet
Talvez a pergunta dele tenha sido
Determine o numero de soloçoes de 1/x + 1/y = 1/1998 com x e y
inteiros positivos.

E é fácil:

(x+y)*1998 = xy
1998x-xy+1998y=0
x(1998-y)+1998y-1998^2=-1998^2
x(1998-y)+1998(y-1998)=-1998^2
(1998-y)(x-1998)=-1998^2
(1998-y)(1998-x)=1998^2


Em 22/09/11, João Maldonado escreveu:
>
>
>
> 1) É impossível que  1/x +  1/y seja maior que 2 né?
> 2)   4m²   +m(4n  -49) + 4n²  - 49n = 0
> delta  = 2401 + 392 n - 48 n   ²
> delta>=0,  -4<=n<=12Testando  achamos( 6,10)(10,6)
> []'s
> João
>
> From: marconeborge...@hotmail.com
> To: obm-l@mat.puc-rio.br
> Subject: [obm-l] Números inteiros
> Date: Thu, 22 Sep 2011 21:23:47 +
>
>
>
>
>
>
>
>
> 1) Determine o numero de soloçoes de 1/x + 1/y = 1998 com x e y inteiros
> positivos.
>
>
>
> 2) Se m e n sao naturais tais que (m + n)/(m^2 + mn + n^2) = 4/49,determinar
> m + n
>
>
>
> Agradeço a quem puder ajudar.
>
>
>
> Abraço,
>
>
>
> Marcone.
>   


-- 
/**/
神が祝福

Torres

=
Instru��es para entrar na lista, sair da lista e usar a lista em
http://www.mat.puc-rio.br/~obmlistas/obm-l.html
=


Re: [obm-l] Problema das Quatro Cores (Teoria dos Grafos)

2011-09-17 Por tôpico Johann Dirichlet
Existe uma demonstração fácil de que 5 cores bastam para pintar um grafo planar.
Acho que este é seu problema: tentar provar por absurdo algo que se
provaria diretamente.
Certamente, se você usa 4 cores para piontar, alguém que tem um
estoque de 5,6,7,2002 cores também consegue.

Mas o salto lógico é este:
Para que seja preciso 5 cores para pintar o grafo, eu teria que ter 5
nós ligados entre si.

Se isto não for corretamente demonstrado, adeus demonstração!



Em 17/09/11, Hugo Fernando Marques Fernandes escreveu:
> Olá, Lista.
>
> Seguinte, estava lendo sobre o "problema das quatro cores", que segundo
> entendi é um teorema da teoria dos grafos que afirma que se pode colorir
> qualquer grafo planar com quatro cores de modo que nós adjacentes (ou seja,
> que possuam aresta ligando-os) não sejam pintados da mesma cor.
>
> Consta que tal fato permaneceu por séculos sem demonstração, e a que existe
> hoje depende de recursos computacionais para ser completada, o que levanta
> dúvidas sobre a mesma.
>
> Minha pergunta então, é a seguinte:
>
> Para que seja preciso 5 cores para pintar o grafo, eu teria que ter 5 nós
> ligados entre si, isto é, eu teria que ter um sub-grafo do meu grafo inicial
> que fosse um grafo completo de 5 nós (K5). Ora, sabemos (é fácil demonstrar,
> já vi em vários livros a demonstração) que K5 não tem realizações
> planares... logo, o teorema segue.
>
> Sei que isso está errado (afinal de contas, se não estivesse, alguém teria
> visto de cara e o problema não teria ficado séculos em aberto...) mas não
> consigo ver onde está o erro desse raciocínio. Alguém pode me ajudar?
>
> Obrigado.
>


-- 
/**/
神が祝福

Torres

=
Instru��es para entrar na lista, sair da lista e usar a lista em
http://www.mat.puc-rio.br/~obmlistas/obm-l.html
=


[obm-l] Re: [obm-l] RE: [obm-l] Solução da série \sum_{i=1}^n 2^{n-i}i^2

2011-09-13 Por tôpico Johann Dirichlet
Na Eureka! 21, creio eu, tem o artigo Integrais Discretas. Ele ensina
a fazer este somatório de uma maneira bem automatizada.

Em 13/09/11, Luís Lopes escreveu:
>
> Sauda,c~oes, oi André,
>
> Comece calculando a série \sum_{i=1}^n i x^i.
>
> E depois \sum_{i=1}^n i^2 x^i.
>
> Para os detalhes, ver os exercícios 47 e 54 do Manual
> de Seq. e Series Vol I em http://www.escolademestres.com
>
> Abs,
> Luís
>
>
>
> From: andrerc...@gmail.com
> Date: Mon, 12 Sep 2011 20:06:35 -0300
> Subject: [obm-l] Solução da série \sum_{i=1}^n 2^{n-i}i^2
> To: obm-l@mat.puc-rio.br
>
> Pessoal,Alguém pode me dar alguma dica como se consegue obter a fórmula
> fecha da série \sum_{i=1}^n 2^{n-i}i^2O Wolfram Alpha indica que a fórmula
> fechada da mesma é 3 * 2^{n+1} - n^2 - 4n - 6:
>
>
>
> http://www.wolframalpha.com/input/?i=%5Csum_{i%3D1}^n+2^{n-i}i^2
> Porém, até então não obtive êxito.
>
>
>
> Abraço,__
> André
>
>
>
>
>   


-- 
/**/
神が祝福

Torres

=
Instru��es para entrar na lista, sair da lista e usar a lista em
http://www.mat.puc-rio.br/~obmlistas/obm-l.html
=


[obm-l] Re: [obm-l] (e + pi) e (e.pi) são irracionais?

2011-09-13 Por tôpico Johann Dirichlet
Como diria Capitão Nascimento,  'Nunca saberão!'Mas é fácil provar que pelo 
menos um deles é irracional.
2011/9/13, Paulo  Argolo :> Caros Colegas,|>> Sabemos 
que os famosos números "e" e "pi" são irracionais.> A soma (e + pi) e o 
produto (e.pi) são também irracionais?>> Abraços do Paulo!> 
=> 
Instruções para entrar na lista, sair da lista e usar a lista em> 
http://www.mat.puc-rio.br/~obmlistas/obm-l.html> 
=>

-- /**/神が祝福
Torres
=
Instru��es para entrar na lista, sair da lista e usar a lista em
http://www.mat.puc-rio.br/~obmlistas/obm-l.html
=


[obm-l] Re: [obm-l] Teoria do Caos - Análise combinatória

2011-09-12 Por tôpico Johann Dirichlet
Procure no Google por permutaçAo caótica ou desarranjo.

Em 12/09/11, João Maldonado escreveu:
>
> Olá, pra todo mundo
> Hoje meu professor me passou um problema sobre teoria do caos como desafio,
> a pergunta era
> Cinco livros caem de uma pratileira, quantas possibilidades existem de todos
> os cinco livros serem repostos,  um do lado do outro, de modo  que nenhum
> deles ocupe a mesma posição  de quando estavam na pratileira.
> Eu fiz  por recursão.
> Para  1 livro, P = 0Para dois livros ab, temos p = ba = 1Para 3 livros, abc,
> temos bac e bca = 2Para 4 livros,  temos 9...
> Para n livros temos:TOTAL  -  1 no mesmo lugar - 2 no mesmo lugar -...- n no
> mesmo lugar,
> f(n)  - n! -  C(n, 1)f(n-1)  - C(n, 2)f(n-2)...-C(n, n)f(0),  tal que f(0) =
> 1
> Para 5 livros:  5! -  5.9 - 10.2 - 10.1 - 0 - 1 = 44
> No final meu professor me disse que havia uma fórmula  direta para f(n),
> mas eu não consegui acharComo acho essa fórmula?
>
> []'sJoão  


-- 
/**/
神が祝福

Torres

=
Instru��es para entrar na lista, sair da lista e usar a lista em
http://www.mat.puc-rio.br/~obmlistas/obm-l.html
=


[obm-l] Re: [obm-l] Questão de geometria plana!! ajuda em nova solução

2011-09-10 Por tôpico Johann Dirichlet
Ué, deslizar o triângulo pra baixo já é sintético. E é a única que eu
imagino agora.

Em 10/09/11, 
douglas.olive...@grupoolimpo.com.br
escreveu:
>
>
> Olá boa tarde, estou com uma questão de geometria plana, que diz
> assim: Em um triângulo equilátero, um ponto P interno dista de sues
> vértices 5 , 7, e 8 de sues vértices, achar o lado.
>
> gostaria de uma
> ajudinha, para elaborar uma nova solução, pois conheco a do oswaldo
> dolce, que transporta um triângulo( esta solucao encontra-se no gabarito
> do livro de matematica elementar numero 9), tambem conheco uma fazendo
> duas leis dos cossenos, e tambem elaborei uma em geometria analitica
> fazendo distancia de ponto a ponto , gostaria de uma ajuda para elaborar
> outra mas totalmente voltada para geometria plana, dede ja agradeco.
>


-- 
/**/
神が祝福

Torres

=
Instru��es para entrar na lista, sair da lista e usar a lista em
http://www.mat.puc-rio.br/~obmlistas/obm-l.html
=


Re: [obm-l] Divisor central de n

2011-09-08 Por tôpico Johann Dirichlet
Mole!Já foi resolvido por mim aqui, mas a ideia é que os produtos dosextremos 
do conjunto dos divisores são iguais a n
Em 08/09/11, Paulo  Argolo escreveu:> Caros Colegas,>> 
Sabe-se que o número natural n>1 tem uma quantidade ímpar de divisores e d> 
ocupa a posição central, quando eles estão dispostos em ordem crescente.> 
Mostrar que n é o quadrado de d.>> Grato,> Paulo> 
=> 
Instru�ões para entrar na lista, sair da lista e usar a lista em> 
http://www.mat.puc-rio.br/~obmlistas/obm-l.html> 
=>

-- /**/神が祝福
Torres
=
Instru��es para entrar na lista, sair da lista e usar a lista em
http://www.mat.puc-rio.br/~obmlistas/obm-l.html
=


[obm-l] Re: [obm-l] Sem soluções racionais

2011-09-07 Por tôpico Johann Dirichlet
Coloca cada cara na forma x =x/A. teremos:

x^2+y^2+z^2=7A^2, com todas as variáveis inteiras positivas.
Tentando um módulo 8, acho que sai...

Em 06/09/11, Vitor Alves escreveu:
>
> Não estou conseguinodo resolver o seguinte problema: Prove que não existem
> racionais x,y e z tais que x^{2} + y^{2} + z^{2}=7. Qualquer sugestão será
> bem vinda,abraços.
>   


-- 
/**/
神が祝福

Torres

=
Instru��es para entrar na lista, sair da lista e usar a lista em
http://www.mat.puc-rio.br/~obmlistas/obm-l.html
=


[obm-l] Re: [obm-l] Re: [obm-l] Re: [obm-l] RE: Mas alguém deu resultado negativo no quinto problema da OBM hoje?

2011-09-03 Por tôpico Johann Dirichlet
Nem. É só fuçar nos sites de matemática mais obscuros da Internet -
OBM, mathlinks...

Em 03/09/11, Bruno França dos Reis escreveu:
> Nossa, essa é uma regra um tanto quanto difícil de se aplicar, não?
>
> --
> Bruno FRANÇA DOS REIS
>
> msn: brunoreis...@hotmail.com
> skype: brunoreis666
> tel: +55 11 9961-7732
>
> http://brunoreis.com
> http://brunoreis.com/tech (en)
> http://brunoreis.com/blog (pt)
>
> GPG Key: http://brunoreis.com/bruno-public.key
>
> e^(pi*i)+1=0
>
>
> 2011/9/3 Carlos Yuzo Shine 
>
>> Caros, até terça, quando sai o gabarito, não é permitido discutir a prova
>> pela Internet. Isso está escrito na ficha de inscrição da prova.
>>
>> []'s
>> Shine
>>
>>
>> 
>> From: João Maldonado 
>> To: obm-l@mat.puc-rio.br
>> Sent: Saturday, September 3, 2011 1:48 PM
>> Subject: [obm-l] RE: Mas alguém deu resultado  negativo no quinto problema
>> da OBM hoje?
>>
>>
>>
>>
>>
>>
>> Esquece, já descobri no que errei.
>> O problema mais difícil  da prova e eu justo vou errar na coisa mais tosca
>>
>> []'s
>> João
>>
>>
>>
>>
>> 
>> From: joao_maldona...@hotmail.com
>> To: obm-l@mat.puc-rio.br
>> Subject: Mas alguém deu resultado  negativo no quinto problema da OBM
>> hoje?
>> Date: Sat, 3 Sep 2011 13:40:06 -0300
>>
>>
>>  Mais alguém achou um resultado negativo no 5 problema de hoje ou foi só
>>  eu??
>>
>> []'s
>> João
>>
>> =
>> Instruções para entrar na lista, sair da lista e usar a lista em
>> http://www.mat.puc-rio.br/~obmlistas/obm-l.html
>> =
>>
>


-- 
/**/
神が祝福

Torres

=
Instru��es para entrar na lista, sair da lista e usar a lista em
http://www.mat.puc-rio.br/~obmlistas/obm-l.html
=


Re: [obm-l] Desafio limite.

2011-09-02 Por tôpico Johann Dirichlet
limite de x^x, x tende a 0+

lim log x^x=lim (x*log x)
lim log (x*log x) = lim log x + lim log log x

lim log x x tende a 0

O que eu fiz ajuda?

Em 29/08/11, Felippe Coulbert Balbi escreveu:
>
> Que legal... não sabia que já tinha uma definição de algo assim... Mas
> enfim... eu escreve errado é 1 se n é par e 0 se n é impar.
>
> Date: Mon, 29 Aug 2011 20:50:12 -0300
> Subject: Re: [obm-l] Desafio limite.
> From: wgapetre...@gmail.com
> To: obm-l@mat.puc-rio.br
>
> http://en.wikipedia.org/wiki/Knuth's_up-arrow_notation
>
> 2011/8/29 Felippe Coulbert Balbi 
>
>
>
>
>
>
> Meu amigo Lucas Colucci e eu resolvemos esse problema que surgiu de uma aula
> de calculo.
> Espero que gostem bastante dele.
> Definição: Dado um "x" pertencendo ao conjunto dos numeros reais e um número
> "n" pertencendo ao conjunto dos numeros naturais.
> definimos: x|||n= e^(ln(x).x|||n-1)
>
> definimos: x|||0= 1 (ao invés de x|||n, meu amigo Lucas sugeriu x flecha pra
> cima n, mas enfim, não faz muita diferença)
>
> Por exemplo:
>
> x|||3= x^(x^x)
> x|||5= x^(x^(x^(x^x)))
>
> Prove que
>
>
> Lim x|||n =
> x->0+
> =
>
> 1, se n é impar
> 0 se n é par
>
> Grato.Coulbert
>   
>
>   


-- 
/**/
神が祝福

Torres

=
Instru��es para entrar na lista, sair da lista e usar a lista em
http://www.mat.puc-rio.br/~obmlistas/obm-l.html
=


Re: [obm-l] Logica

2011-08-18 Por tôpico Johann Dirichlet
Não sei se é isso, mas ele prefere algo mais sistemático, menos ad
hoc. Mas isto é meio difícil - creio que impossível: parece um
problema P-NP.

Em 18/08/11, Bernardo Freitas Paulo da Costa escreveu:
> Eu acho que se você fizer uma "matriz" do que pode ser, e for marcando
> 0 / 1, sai. (Por exemplo, se na linha Erica e na coluna Inteligente
> tem um 0, isso quer dizer que Érica não é inteligente). Esse seu
> problema é razoavelmente fácil porque já tem uma afirmação sobre um
> dos 0/1 : Joyce. O resto sai daí direto. Talvez essa afirmação seja
> redundante... (eu não usei a penúltima, e nesse tipo de problema,
> haver uma informação redundante é quase sempre "permutável").
>
> Mas não sei se isso você não chamaria de "lógica". Eu sim... mesmo que
> a tabelinha faça parecer mais do que não é. Enfim, você tem um
> problema de lógica, porque você quer usar outra coisa? Ou fazendo uma
> pergunta mais simples: se fosse um problema de geometria, porque você
> acha que uma solução por geometria não é bom?
>
> Abraços,
> --
> Bernardo Freitas Paulo da Costa
>
>
> 2011/8/18 Marcus Aurelio Gonçalves Rodrigues :
>> Galera existe alguma forma "matemática"  sem usar a própria lógica para
>> resolver esses tipos de problemas?
>>
>> Considere que são verdadeiras as seguintes afirma coes:
>> Se Adriane não  e inteligente, então Joyce e linda.
>> Se Joyce não  e comunicativa, então  Erica não  e linda.
>> Se Luciana não  e inteligente, então  Erica e comunicativa.
>> Joyce e imatura.
>> Deduza o atributo (imatura, inteligente, comunicativa ou linda) de cada
>> uma
>> das garotas, sabendo-se que  cada mo ca tem exatamente um atributo e duas
>> mo
>> cas quaisquer nao tem o mesmo atributo.
>> --
>> Prof Marcus
>
> =
> Instruções para entrar na lista, sair da lista e usar a lista em
> http://www.mat.puc-rio.br/~obmlistas/obm-l.html
> =
>


-- 
/**/
神が祝福

Torres

=
Instru��es para entrar na lista, sair da lista e usar a lista em
http://www.mat.puc-rio.br/~obmlistas/obm-l.html
=


Re: [obm-l] Logica

2011-08-18 Por tôpico Johann Dirichlet
O mais matemático que eu conseguiria pensar é lógica booleana de
circuitos. Mas é lógica do mesmo jeito: analisar as premissas e obter
uma conclusão.

-- 
/**/
神が祝福

Torres

=
Instru��es para entrar na lista, sair da lista e usar a lista em
http://www.mat.puc-rio.br/~obmlistas/obm-l.html
=


Re: [obm-l] Divisibilidade

2011-08-17 Por tôpico Johann Dirichlet
Basta demonstrar que (2^a)-(2^q) é múltiplo de  (2^b)-1.
Assim, escreva a=bX+q, fatore e conclua!

Em 17/08/11, Kleber Bastos escreveu:
> Olá Galera,
>
> Estou com dúvida no seguitne problema:
>
> *Sejam a>b  inteiros positivos. Mostre que se o resto da divisão de a por b
> é q então o resto da divisão de (2^a)-1 por (2^b)-1 é (2^q)-1.*
>
>
> Att,
> Kleber
>


-- 
/**/
神が祝福

Torres

=
Instru��es para entrar na lista, sair da lista e usar a lista em
http://www.mat.puc-rio.br/~obmlistas/obm-l.html
=


Re: [obm-l] Teorema sobre o resto

2011-08-17 Por tôpico Johann Dirichlet
Eureka! 2, Divisibilidade, Congruências e Aritmética módulo n.
Em 17/08/11, Pedro Chaves escreveu:>> Caros amigos,>>> 
Como podemos provar o teorema abaixo?>> Dados n números inteiros (x_1, x_2, 
..., x_n),  cujo produto é P, então o> resto da divisão de P por d (d é inteiro 
diferente de zero) é o resto da> divisão do produto (r_1).( r_2) .( r_n) 
por d, onde r_1, r_2, ..., r_n> são os respectivos restos das divisões de x_1, 
x_2, ..., x_n por d.>> Muitíssimo grato!>> Pedro Chaves 
   > 
=> 
Instruções para entrar na lista, sair da lista e usar a lista em> 
http://www.mat.puc-rio.br/~obmlistas/obm-l.html> 
=>

-- /**/神が祝福
Torres
=
Instru��es para entrar na lista, sair da lista e usar a lista em
http://www.mat.puc-rio.br/~obmlistas/obm-l.html
=


Re: [obm-l] Determinante nulo (outra vez)

2011-08-16 Por tôpico Johann Dirichlet
Bem, a melhor ideia é tentar usar o Lema de Gauss, e ir diminuindo a
ordem do determinante.
Um caso qualquer:

a b c
d e f
g h i

Suponha a!=0 (trocando linhas e colunas)

Podemos, usando transforma,cões lineares, obter isto:
a b c
0 E F
0 H I

Fatorando o a, temos
E F
H I

Siga daí!

Em 16/08/11, Paulo  Argolo escreveu:
> Colegas da Lista,
>
> Solicito, assim como já fez o Ennius, há algum tempo, ajuda na demonstração
> do teorema abaixo.
> TROREMA:
> O determinante de uma matriz quadrada M, de ordem maior que 1, só é nulo
> quando M possui alguma fila que seja combinação linear das filas paralelas
> (a essa fila).
>
> Um grande abraço do Paulo!
> =
> Instru�ões para entrar na lista, sair da lista e usar a lista em
> http://www.mat.puc-rio.br/~obmlistas/obm-l.html
> =
>


-- 
/**/
神が祝福

Torres

=
Instru��es para entrar na lista, sair da lista e usar a lista em
http://www.mat.puc-rio.br/~obmlistas/obm-l.html
=


Re: [obm-l] 2^70 + 3^70 eh divisivel por 13

2011-08-10 Por tôpico Johann Dirichlet
Use o Teorema de Fermat: 2^(12a+b)=2^b e 3^(12a+b)=3^b módulo 13

Em 10/08/11, Luís Lopes escreveu:
>
> Sauda,c~oes,
>
> Alguém poderia resolver?
>
>>Solicitaria a voce uma solução para a questão :
>>demonstre que 270 + 370 é divisível por 13.
>
>
>
>
> []'s
> Luis
>
>   


-- 
/**/
神が祝福

Torres

=
Instru��es para entrar na lista, sair da lista e usar a lista em
http://www.mat.puc-rio.br/~obmlistas/obm-l.html
=


[obm-l] Re: [obm-l] Re: [obm-l] Demonstração

2011-08-05 Por tôpico Johann Dirichlet
a^2=3^k*b, em que 3 não divide b.

Sabemos que k>1, pois 3 é divisor de a^2.
Mas k deve ser necessariamente par, pois os expoentes da foatoração de
um quadrado perfeito são pares. Logo k=2l, com l>1.
Então a^2=3^(2l)*b, o que acarreta (a/(3^l))^2 = b. Portanto, como b é
inteiro, b é quadrado perfeito:

(a/(3^l))^2 = c^2
(a/(3^l)) = c
a = 3^l*c
Como l>1, está provado: 3 é divisor de a.

Em 05/08/11, Ricardo Lopes escreveu:
> Multiplo de 3?
>
> Abraços
>
> Em 5 de agosto de 2011 14:33, Marcus Aurelio Gonçalves Rodrigues <
> marcusaureli...@globo.com> escreveu:
>
>> Alguém da uma forcinha?
>>
>> se a^2 e divisível por 3, então a também é?
>>
>> --
>> Prof Marcus
>>
>
>
>
> --
> Ricardo Shydo
> (71)8126-2111
> ricardo.lopesmore...@gmail.com
> ricardo.blackj...@gmail.com
> sh...@bol.com.br
> moreira_lopes2...@ig.com.br
> sh...@linuxmail.org
> ntsh...@hotmail.com
>


-- 
/**/
神が祝福

Torres

=
Instru��es para entrar na lista, sair da lista e usar a lista em
http://www.mat.puc-rio.br/~obmlistas/obm-l.html
=


[obm-l] Re: [obm-l] Re: [obm-l] Re: [obm-l] Re: [obm-l] Números Primos

2011-08-04 Por tôpico Johann Dirichlet
7^a*11^b têm 16 divisores no total.
(a+1)(b+1)=16

Liste as possibilidades e finalize!


Em 04/08/11, Marcus Aurelio Gonçalves
Rodrigues escreveu:
> Determine os números inteiros positivos cujos únicos divisores primos são 7
> e 11 e que possuem exatamente 15 divisores positivos diferentes de 1
>


-- 
/**/
神が祝福

Torres

=
Instru��es para entrar na lista, sair da lista e usar a lista em
http://www.mat.puc-rio.br/~obmlistas/obm-l.html
=


[obm-l] Re: [obm-l] Re: [obm-l] Números Primos

2011-08-04 Por tôpico Johann Dirichlet
Bem, eu conheço um assim:

Como estudo de caso, seja 7 o primo que estamos pesquisando.

1 - Encontre um divisor da forma M*10+1. No caso, 7*3=21, M=2.

2 - A cada passo, faça isto aqui:
2a - Arranque o último dígito, e duplique-o (M=2, e 7*3=2*10+1);
2b - Subtraia do restante do número.

Por exemplo, 1001 é múltiplo de 7?

1001 => 100-2=98 => 9-2*8=-7, OK, pois 7 é múltiplo!

Encontrar divisores da forma 10K+1 é fácil, basta olhar a tabuada.

Em 03/08/11, regis barros escreveu:
> Boa Tarde Pessoal
> Gostaria algum material sobre criterio de divisibilidade que nesta lista
> mandou algum tempo atrás sobre o assunto e do qual não estou encontrando o
> email com o link sobre o assunto.
>
> Regis Godoy BarrosGraduado em Licenciatura em Fisica - IFSPGraduando em
> Licenciatura em Matemática - UNICAMP
>
>


-- 
/**/
神が祝福

Torres

=
Instru��es para entrar na lista, sair da lista e usar a lista em
http://www.mat.puc-rio.br/~obmlistas/obm-l.html
=


[obm-l] Re: [obm-l] ANÁLISE COMBINATÓRIA

2011-07-27 Por tôpico Johann Dirichlet
Bem, para o 2, dou uma dica: divida o intervalo [0,1] em n partes, e
pense onde cairiam as partes fracionárias dos Kx.

Em 27/07/11, Marcelo Costa escreveu:
> *1 - Prove que dado qualquer conjunto de dez inteiros positivos de dois
> dígitos cada, é possível obter dois subconjuntos disjuntos cujos elementos
> têm a mesma soma.
>
> 2 - Sejam x um número real e n um inteiro positivo. Mostre que entre os
> números x, 2x, 3x, . . ., (n – 1)x, existe um cuja distância a algum inteiro
> é, no máximo, 1/n.
>
> AGRADEÇO DESDE JÁ VOSSA ATENÇÃO
> *
>


-- 
/**/
神が祝福

Torres

=
Instru��es para entrar na lista, sair da lista e usar a lista em
http://www.mat.puc-rio.br/~obmlistas/obm-l.html
=


[obm-l] Re: [obm-l] Re: [obm-l] Re: [obm-l] Essa ainda não consegui!!!

2011-07-26 Por tôpico Johann Dirichlet
Mas esse é bem mais moleza!
Os pontos são da forma (x_i,y_i)
Os médios são da forma ((x_i+x_j)/2,(y_i+y_j)/2)

Se conseguirmos garantir que existem dois pontos (x_i,y_i) e (x_j,y_j)
tais que as coordenadas x tenham igual paridade, bem como as
coordenadas y, acabou.

Se isto não ocorresse, o que se daria?
Temos pontos do tipo (par,par), (par, impar), (impar, par) e (impar, impar).
Como são cinco pontos, um dos tipos se repete. E achamos os pontos!

Agor, seria interessante se pudéssemos ver este problema acima. Creio
que existe um numero tao grande de pontos quantos se queira, de modo
que as coordenadas de intersecção sejam sempre fracionárias.



Em 24/07/11, Pedro Júnior escreveu:
> Exatamente caríssimo Ralph, tens razão, é que estava tentanto lembrar do
> problema e fui escrevendo, mas vc me fez lembrar direitinho, como sempre!!!
> Parabéns.
>
> Em 24 de julho de 2011 11:23, Ralph Teixeira  escreveu:
>
>> Ah... aposto que o problema original era para mostrar que um dos PONTOS
>> MEDIOS desses 10 segmentos tem coordenadas inteiras, nao? Ai tudo faz
>> sentido: basta olhar a paridade de ambas as coordenadas. Ha 4 "classes" de
>> possibilidades: (Par,Par), (Par, Impar), (Impar, Par), (Impar, Impar).
>> Como
>> voce tem 5 pontos, pombas, tem que haver dois deles dentro da mesma
>> "classe", digamos, X e Y. Mas entao as coordenadas de X+Y serao ambas
>> pares,
>> isto eh, as coordenadas do ponto medio (X+Y)/2 serao inteiras.
>>
>> Aposto 10 pratas que era esse o problema! Em dolar! :)
>>
>> Abraco,
>> Ralph
>>
>> 2011/7/24 Pedro Júnior 
>>
>>> Sejam A, B, C, D e E pontos do plano cartesiano de coordenadas inteiras.
>>> Três quaisquer desses pontos não estão alinhados, logo formam dez
>>> segmentos.
>>> Mostre que pelo menos um dos pontos de intersecção desses segmentos é um
>>> ponto, também, de coordenadas inteiras.
>>> Desde já agradeço.
>>>
>>> --
>>>
>>> Pedro Jerônimo S. de O. Júnior
>>>
>>> Professor de Matemática
>>>
>>> Geo João Pessoa – PB
>>>
>>>
>>
>
>
> --
>
> Pedro Jerônimo S. de O. Júnior
>
> Professor de Matemática
>
> Geo João Pessoa – PB
>


-- 
/**/
神が祝福

Torres

=
Instru��es para entrar na lista, sair da lista e usar a lista em
http://www.mat.puc-rio.br/~obmlistas/obm-l.html
=


[obm-l] Re: [obm-l] RE: [obm-l] Soma dos dígitos de um número

2011-07-26 Por tôpico Johann Dirichlet
Bem, você tem mesmo que torcer para que isto tenha um só dígito. Umas
contas e desigualdades do tipo 'o maior número de 100 dígitos é
9...9' dão conta, mais um pouquinho de logaritmos.

Depois, usa módulo 9!

Em 25/07/11, Frederico Matos escreveu:
>
> Bem, se adotarmos que F(F(F(2000^2000) só tem um digito tomamos a seguinte
> provisão:
> como jah bem citou: F(F(F(F(2000^2000) = F(F(F(F(2^2000) jah que a soma dos
> digitos de um numero n.10^x é n.
> Agora veja como no 2 a soma do digitos eh cíclica:
>
> 2^0 = 1   = 1
> 2^1 = 2   = 2
> 2^2 = 4   = 4
> 2^3 = 8   = 8
> 2^4 = 16 = 7
> 2^5 = 32 = 5
>
> 2^6   = 64 = 10 = 1
> 2^7   = 128   = 11 = 2
> 2^8   = 256   = 13 = 4
> 2^9   = 512   = 8
> 2^10 = 1024 = 7
> 2^11 = 2048 = 14 = 5
>
> Então F(2^n) = F(2^n_mod(6))
> Logo F(2^2000) = F(2^2000mod(6)) = F(2^2) = 4
>
>
>
>
>
> From: joao_maldona...@hotmail.com
> To: obm-l@mat.puc-rio.br
> Subject: [obm-l] Soma dos dígitos de um número
> Date: Sun, 24 Jul 2011 13:08:05 -0300
>
>
>
>
>
> Dado a função F(x) =  soma dos dígitos de x,
>
>
> calcule F(F(F(F(2000^2000
>
> Parece que se aplicarmos inúmeras vezes F,até que o número só tenha um
> dígito, o resultado é o resto da divisão do número por 9 (também não sei
> porque),  a não ser que o  número seja divisível por 9, daí o resto é  9.
> Como  F(2000^ 2000) = F(2^2000)  não sendo divísivel 9, pela regra daria
> 2^1998. 2^2 mod(9) =  (-1)^1998.4 = 4 mod(9)
>
> Mas ainda  falta provar  tal regra e também que  F(F(F(F( 2^2000 só  tem
> 1 dígito
>
> []'s
> João
>   


-- 
/**/
神が祝福

Torres

=
Instru��es para entrar na lista, sair da lista e usar a lista em
http://www.mat.puc-rio.br/~obmlistas/obm-l.html
=


[obm-l] Re: [obm-l] Fatorial [último dígito não nulo]

2011-07-23 Por tôpico Johann Dirichlet
É bem mais divertido saber qual é o último dígito diferente de zero de
um fatorial.
Tente!

Em 23/07/11, Victor Seixas Souza escreveu:
> Existe uma fórmula geral para isso:
> http://latex.codecogs.com/gif.latex?\dpi{150}&space;N&space;=&space;\sum_{k=1}^{\infty&space;}&space;\left&space;\lfloor&space;\frac{n}{5^{k}}&space;\right&space;\rfloor
>
> N = quantidade de zeros em n!
> N = somatório de k=1 até infinito de (aproxima para baixo (n/5^k))
>
> Ou seja, para 1500 fatorial seria:
> 1500/5 = 300
> 1500/25 = 60
> 1500/125 = 12
> 1500/625 = 2.4 => 2
> 1500/3125 = 0.4 => 0
> 
> N = 300 + 60 + 12 + 2 + 0 + 0 + 0 + ... = 374
>
> Agora vou tentar explicar porque essa forma funciona.
> A chave para entender a fórmula é perceber que os multiplos de 2 são mais
> comuns do que os de 5.
> Em um produto de inteiros, a única forma de aparecer 0 na terminação é
> multiplicar por 10 = 5x2, explicita ou implicitamente.
> Mas,
> 2x5 = 10
> 4x25 = 100
> 8x125 = 1000
> 16x625 = 1
> ...
> 2^n x 5^n = 10^n
> Como em o fatorial é um produtório, você teria de contar quantos pares 2ˆn x
> 5ˆn você acha.
> Os 2 são desnecessários no caso do fatorial, pois sempre existirão e
> sobrarão multiplos de 2 em relação aos de 5.
> O Fato de que você vai somando as divisões por 5^n é que os produtos de 4x25
> produz 2 zeros, 8x125 produz 3 zeros, logo você precisa contar estes mais de
> uma vez, no caso, n vezes.
> Isso contudo não é uma prova, apenas um feeling e uma explicação que espero
> que esteja clara.
>
> Victor Seixas Souza
>


-- 
/**/
神が祝福

Torres

=
Instru��es para entrar na lista, sair da lista e usar a lista em
http://www.mat.puc-rio.br/~obmlistas/obm-l.html
=


[obm-l] Re: [obm-l] Re: [obm-l] Brasil conquista medalhas de Prata e Bronze na Olimpíada Internacional de Matemática (IMO)

2011-07-23 Por tôpico Johann Dirichlet
Valeu o papinho ufanista, mas... Cadê a prova da IMO, meu povo? Nunca
mais esta lista se divertiu resolvendo os problemas dela não?

Em 22/07/11, Fernando A Candeias escreveu:
> É um juso motivo de orgulho para esta sofrida nação. E sem nenhum apoio do
> papai governo. Temos ótimos matemáticos, o que certamente eleva nossa auto
> estima, e  que  deveria ser  adequadamente  valorizado.
>
> Fernando Candeias
> Em 22 de julho de 2011 10:11, Olimpiada Brasileira de Matematica <
> o...@impa.br> escreveu:
>
>> **
>>
>> *Brasil conquista medalhas de Prata e Bronze*
>>
>> *na Olimpíada Internacional de Matemática (IMO)*
>>
>>
>> O Brasil obteve um excelente resultado este ano na 52a. Olimpíada
>> Internacional de Matemática (IMO), que acontece até o dia 24 de julho na
>> cidade de Amsterdã na Holanda,* *conquistando três medalhas de prata e
>> três de bronze. Os estudantes: André Macieira Braga (Belo Horizonte – MG),
>> João Lucas Camelo Sá (Fortaleza – CE) e Henrique Fiúza do Nascimento
>> (Brasília – DF), conquistaram as medalhas de prata, enquanto Débora
>> Barbosa
>> Alves (São Paulo – SP), Maria Clara Mendes Silva (Pirajuba – MG) e Gustavo
>> Lisbôa Empinotti (Florianópolis – SC) conquistaram medalhas de bronze. Com
>> este resultado o Brasil classificou em vigésimo lugar entre os países
>> participantes.
>>
>> Considerada pela Unesco como a competição mais importante da área, a IMO
>> contou este ano com a participação de 101 países reunindo 564 estudantes,
>> entre 14 e 19 anos, mais talentosos do mundo no assunto. O Brasil foi
>> representado por uma equipe de seis estudantes liderados pelos professores
>> Nicolau
>> Corção Saldanha (Rio de Janeiro – RJ) e Eduardo Tengan (São Carlos – SP).
>>
>>
>> Um comitê internacional elegeu os problemas que seriam resolvidos entre os
>> propostos pelos países participantes. As provas foram realizadas em dois
>> dias consecutivos abrangendo disciplinas como Álgebra, Teoria dos números,
>> Geometria e Combinatória. Em cada dia, os participantes resolveram três
>> problemas, com valor de sete pontos cada, aplicados em 4 horas e meia de
>> prova. A resolução destes problemas requer mais criatividade, engenho e
>> habilidade em matemática do que conhecimentos e fórmulas aplicadas.
>>
>> *Brasil e as medalhas na IMO*
>>
>> A Olimpíada Internacional de Matemática (IMO) é realizada desde 1959. O
>> Brasil participa da competição desde 1979 conquistando desde então um
>> total
>> de 96 medalhas, sendo oito de ouro, 26 de prata e 62 de bronze.
>>
>> A participação brasileira na competição é organizada através da Olimpíada
>> Brasileira de Matemática (OBM), iniciativa que tem desempenhado um
>> importante papel em relação à melhoria do ensino e descoberta de talentos
>> para a pesquisa em Matemática nas modalidades de ensino fundamental e
>> médio
>> nas escolas públicas e privadas de todo o Brasil.  A Olimpíada Brasileira
>> de Matemática é um projeto conjunto do Instituto Nacional de Matemática
>> Pura
>> e Aplicada (IMPA) da Sociedade Brasileira de Matemática (SBM) e conta com
>> o
>> apoio do Conselho Nacional de Desenvolvimento Científico e Tecnológico
>> (CNPq) e do Instituto Nacional de Ciência e Tecnologia de Matemática
>> (INCT–
>> Mat).
>>
>>
>>
>> *Informações:*
>>
>> Nelly Carvajal – Assessoria de Imprensa
>>
>> Tel: 21-25295077 – Fax: 21-25295023
>>
>> e-mail:o...@impa.br
>>
>> --
>> Secretaria da Olimpíada Brasileira de Matemática
>> Estrada Dona Castorina, 110 Jd. Botânico,
>> Rio de Janeiro - RJ, 22460-320, Brasil
>> Tel: 55-21-25295077 Fax:55-21-25295023
>> e-mail: o...@impa.br
>> web site: www.obm.org.br
>>
>>
>


-- 
/**/
神が祝福

Torres

=
Instru��es para entrar na lista, sair da lista e usar a lista em
http://www.mat.puc-rio.br/~obmlistas/obm-l.html
=


[obm-l] Re: [obm-l] Questão Eureka 33

2011-07-21 Por tôpico Johann Dirichlet
Ué, você acabou de demonstrar! É claro, se todas as contas estiverem
corretas, você não precisa fazer mais nada.
Se para os casos abaixo de 8 não deu certo, só daria de 8 para cima.
Mas deu certo para 8, logo 8 é o mínimo!

Em 20/07/11, João Maldonado escreveu:
>
> Olá
> 3) Encontre o menor k > 2 para o qual existem k números inteiros
> consecutivos, tais que a soma dos seus quadrados é um quadrado.
> Minha resolução:
> para  k =3
> (r-1)²+r²+(r+1)² = x²3r²+2 = x², x = 3n+1 ou 3n-1, x² = 3p+1, impossível
> para k = 44r²+4r+6 = x² -> x² é múltiplo de 2 mas não de 4, impossível
> para k=55r²+10 = x²5(r²+2)=x²r²+2 = 5kr=5p+2, 5p-2, 5 p+1, 5p-15n+6 ou 5n +
> 3 = 5k, impossível
> para k=66r²+6r+19 = x²6(r²+r+3)+1 = x²x=6p+  3, 6p+2, 6p-2, 6p+1, 6p-1temos
> x =  6p+1 ou 6p-1
> 6(r²+r+3)+1 =  36p² -+ 12p + 1X = r² + r + 3 = 2(3p² +-p)
> ser é par, X é ímpar, se r é ímpar, X é ímpar
> para k = 7   7r² + 28 = x²7 (r²+ 4)  = x²
> r²+4 múltiplo de 7,
>
> r = 7p+1, 7p-1, 7p+2, 7p-2, 7p+3, 7p-3r²+4 =  7n -1, 7n-2, 7n+1, absurdo
> para k = 8
>
> 8r²+8r+44 = x²
> 4(2r²  +2r+11) = x²
>
> Não vejo nenhum problema aqui,  será k = 8 a resposta? Se sim, como provar?
> []'s,  Joaao
>
>
>
>
>
>
>
>   


-- 
/**/
神が祝福

Torres

=
Instru��es para entrar na lista, sair da lista e usar a lista em
http://www.mat.puc-rio.br/~obmlistas/obm-l.html
=


[obm-l] Re: [obm-l] Re: [obm-l] Qual o software usado para as questões de geometria?

2011-07-18 Por tôpico Johann Dirichlet
Eu recomendo o CaR (Compass and Ruler). Muito simples e bem prático.
http://zirkel.sourceforge.net/

Em 18/07/11, regis barros escreveu:
> Bom dia João
> Você conhece o geogebra, este é um software com código livre, acho que serve
> para vocês.
>
> Regis
> --- Em dom, 17/7/11, João Maldonado  escreveu:
>
> De: João Maldonado 
> Assunto: [obm-l] Qual o software usado para as questões de geometria?
> Para: obm-l@mat.puc-rio.br
> Data: Domingo, 17 de Julho de 2011, 23:15
>
>
>
>
>
>
> Boa Tarde,
>
> Para quem não sabe estou construindo um site online (faz uma semana ), com
> preparação para  vestibulares focados no contexto de exatas, como ITA/ IME.
>
>
> Quem quiser entrar "resumaovestibular.com"
> Mas  fica muito difícil fazer construções geométricas no fireworks ou
> photoshop.  Alguém sabe de algum programa bom de desenvolvimento geométrico?
> Queria saber qual o programa usado nas construções  na prova da OBM  (isto
> claro, se o programa for aberto e seja possível me dizer).
>
>
> Muito   Obrigado,
> João      
>


-- 
/**/
神が祝福

Torres

=
Instru��es para entrar na lista, sair da lista e usar a lista em
http://www.mat.puc-rio.br/~obmlistas/obm-l.html
=


Re: [obm-l] UMA SENHORA AJUDA

2011-07-18 Por tôpico Johann Dirichlet
Mas isso é maldade! Ele quer apenas a prova de que e^2 é irracional,
não da transcedencia (que é mais treta, mas sai). É matar um cachorro
com espingarda de elefante.

Te recomendo o famigerado Proofs from THE BOOK. Lá tem uma
demonstração para este e os casos e^racional.

Em 17/07/11, Artur Costa Steiner escreveu:
> e é transcendente e potências inteiras de transcendentes são transcendentes.
> Como todo real trancendente é irracional, temos a conclusão desejada.
> Abraços.
>
> Artur Costa Steiner
> Em 17/07/2011 18:35, "Marcelo Costa"  escreveu:
>>
>> PROVE QUE e^2 É IRRACIONAL.
>>
>> JÁ DEMONSTREI QUE e É IRRACINAL, MAS ACREDITE O PROCESSO NÃO SE "ENCAIXA"
> PARA e^2, SÓ SEI QUE SERÁ POR ABSURDO, ABRAÇOS A TODOS!
>


-- 
/**/
神が祝福

Torres

=
Instru��es para entrar na lista, sair da lista e usar a lista em
http://www.mat.puc-rio.br/~obmlistas/obm-l.html
=


[obm-l] Re: [obm-l] Média harmônica igual média geométrica

2011-07-12 Por tôpico Johann Dirichlet
Eureka! 5, artigo do Caminha sobre desigualdades.

Dá uma boa lida, é um bom material introdutório sobre desigualdades em
olimpíadas. Aliás, a Eureka! é um farto material pra discussões desta
natureza.

Me desculpe pela secura da resposta, mas é que Desigualdade das Médias
é algo tão comum para mim - e muitos da lista - quanto o teorema de
Pitágoras. Eu não faria nada além de um C-y aqui.

Pra não dizer que fui um cara malvado >:-P, eis um site:
http://erdos.ime.usp.br/index.php/Desigualdades

Em 11/07/11, ennius escreveu:
> Caros Colegas,
>
> Peço-lhes uma prova, se possível for, da propriedade abaixo.
>
> Propriedade: A média harmônica e a média geométrica, de n números reais
> positivos dados, são iguais somente se os n números forem todos iguais.
>
> Agradeço-lhes muitíssimo desde já.
>
> Ennius Lima
> =
> Instru�ões para entrar na lista, sair da lista e usar a lista em
> http://www.mat.puc-rio.br/~obmlistas/obm-l.html
> =
>


-- 
/**/
神が祝福

Torres

=
Instru��es para entrar na lista, sair da lista e usar a lista em
http://www.mat.puc-rio.br/~obmlistas/obm-l.html
=


Re: [obm-l] P(x^2+1)=P(x)^2+1

2011-07-08 Por tôpico Johann Dirichlet
Bem, no avanço da ciencia, estou crente de algumas coisas:

1 - Fixado o grau, se o polinomio existir será único.
Quando eu abro os expoentes, não há margem para substituições arbitrárias.
Eu começo a crer nisto por experiência.

2 - O grau do polinômio seria uma potência de 2: grau 6 falha.

Em 08/07/11, Johann Dirichlet escreveu:
> Que coisa... Onde tá o meu erro, again??
>
> De todo modo, o problema original tá resolvido!
> Acho que testei casos de graus que darão errado, e não os que dariam
> certo... Mas é facto: o polinômio deve ser par, e só terá expoentes
> pares.
>
> Em 07/07/11, Carlos Yuzo Shine escreveu:
>> Ué, P(x) = x^2 + 1 não dá certo? Essencialmente, sendo f(x) = x^2 + 1,
>> estamos
>> querendo resolver P(f(x)) = f(P(x)), e se f = P isso dá certo.
>>
>> Na verdade, f(x) = P(P(...P(x)...)) dá certo para qualquer quantidade de
>> vezes
>> que aplicamos P. Não sei se são todas as soluções, porém.
>>
>> []'s
>> Shine
>>
>>
>> - Original Message 
>> From: Johann Dirichlet 
>> To: obm-l@mat.puc-rio.br
>> Sent: Thu, July 7, 2011 10:01:22 AM
>> Subject: Re: [obm-l] P(x^2+1)=P(x)^2+1
>>
>> Bem, voltando ao novo problema:
>> P(x^2+1)=(P(x))^2+1.
>>
>> O polinômio é mônico, basta aplicar uma ideia básica de limite para
>> saber o valor do coeficiente líder de P.
>>
>> Primeiro, se P(c)=c para algum c real então P(x)=x. É só usar a
>> formula acima para achar infinitos x tais que P(x)=x. E dois
>> polinomios que se intersectam infinitamente são iguais.
>>
>> Logo, podemos supor P(x) != x para todo x.
>>
>> O polinomio acima ou é par ou é ímpar. Basta ver que
>> (P(x))^2=(P(-x))^2, e ou P(x)=P(-x) pu P(x)=-P(-x). Como pelo menos
>> uma destas alternativas ocorre infinitas vezes, o polinômio ou é par
>> ou é ímpar.
>>
>> Mas se P(x) fosse ímpar, P(0)=0. E sabemos que P(x)!=x. Logo, o
>> polinomio procurado é par.
>>
>> Meu problema está sendo o seguinte: quando eu testo um polinômio (isso
>> mesmo, P(x)=x^4+bx^2+c, substituir e suar a caneta!), ele falha
>> miseravelmente.
>> Creio que este problema está sem solução, also...
>>
>> Em 01/07/11, Ralph Teixeira escreveu:
>>> Melhorando aos poucos, ainda usando as ideias do Dirichlet: p(x) não
>>> pode
>>> ser ímpar. Se fosse, 0 seria raiz. Mas então 0^2+1=1 seria raiz, e
>>> 1^2+1=2
>>> seria raiz, e 2^2+1=5 seria raiz... e p(x) não pode ter infinitas
>>> raízes.
>>> Então estamos à procura de um polinômio **par** p(x) tal que
>>> p(x^2+1)=[p(x)]^2.
>>>
>>> Aliás, esse raciocínio mostra que esse p(x) não pode ter nenhuma raiz
>>> real
>>> -- se tiver uma raiz real x, terá infinitas, já que x^2+1>x para todo x
>>> real.
>>>
>>> (Por enquanto, fico com a terrível impressão de que tal polinômio não
>>> existe... Alguém achou o dito cujo?)
>>> 2011/7/1 Ralph Teixeira 
>>>
>>>> O raciocínio do Dirichlet mostra que basta achar UM polinômio (não
>>>> constante) que tenha esta propriedade. Afinal, como ele mostrou, se
>>>> p(x)
>>>> serve, então q(x)=(p(x))^2 também serve.
>>>>
>>>> Mas seja lá quem for o polinômio mágico, eu sei que ou ele é um
>>>> polinômio
>>>> par ou ele é ímpar. Afinal, escreva p(x)=P(x)+I(x) onde P(x) tem apenas
>>>> os
>>>> termos de grau par e I(x) tem apenas os de grau ímpar.
>>>>
>>>> Ora, p(x)^2=(P^2+I^2)+2PI. Note que P^2+I^2 é um polinômio par e 2PI é
>>>> ímpar.
>>>>
>>>> Mas a condição manda que p^2=p(x^2+1), que é uma função par. Então o
>>>> termo
>>>> 2PI não pode existir, isto é, P=0 ou I=0. Assim, p(x) é par ou ímpar. E
>>>> x^2-x+1 não é um nem outro, então não funcionou...
>>>>
>>>> Então precisamos ainda mostrar que existe UM tal polinômio!
>>>>
>>>> Abraço,
>>>>   Ralph
>>>>
>>>> P.S.: Tem certeza que o enunciado é esse mesmo? Não seria, sei lá,
>>>> p(x^2+1)=(p(x))^2+1 ao invés?
>>>> 2011/7/1 Johann Dirichlet 
>>>>
>>>>> Em 01/07/11, Johann Dirichlet escreveu:
>>>>> > Em 30/06/11, marcone augusto araújo
>>>>> > borges escreveu:
>>>>> >>
>>>>> >> 1) Se p é inteiro primo ímpar,mostre que o numerador da fração
>>>>> >> 1+1/2+1/3+...1/(p-1) é um múltiplo de p.
>>>>> >
>>>>> > 1) T

Re: [obm-l] P(x^2+1)=P(x)^2+1

2011-07-08 Por tôpico Johann Dirichlet
Que coisa... Onde tá o meu erro, again??

De todo modo, o problema original tá resolvido!
Acho que testei casos de graus que darão errado, e não os que dariam
certo... Mas é facto: o polinômio deve ser par, e só terá expoentes
pares.

Em 07/07/11, Carlos Yuzo Shine escreveu:
> Ué, P(x) = x^2 + 1 não dá certo? Essencialmente, sendo f(x) = x^2 + 1,
> estamos
> querendo resolver P(f(x)) = f(P(x)), e se f = P isso dá certo.
>
> Na verdade, f(x) = P(P(...P(x)...)) dá certo para qualquer quantidade de
> vezes
> que aplicamos P. Não sei se são todas as soluções, porém.
>
> []'s
> Shine
>
>
> - Original Message 
> From: Johann Dirichlet 
> To: obm-l@mat.puc-rio.br
> Sent: Thu, July 7, 2011 10:01:22 AM
> Subject: Re: [obm-l] P(x^2+1)=P(x)^2+1
>
> Bem, voltando ao novo problema:
> P(x^2+1)=(P(x))^2+1.
>
> O polinômio é mônico, basta aplicar uma ideia básica de limite para
> saber o valor do coeficiente líder de P.
>
> Primeiro, se P(c)=c para algum c real então P(x)=x. É só usar a
> formula acima para achar infinitos x tais que P(x)=x. E dois
> polinomios que se intersectam infinitamente são iguais.
>
> Logo, podemos supor P(x) != x para todo x.
>
> O polinomio acima ou é par ou é ímpar. Basta ver que
> (P(x))^2=(P(-x))^2, e ou P(x)=P(-x) pu P(x)=-P(-x). Como pelo menos
> uma destas alternativas ocorre infinitas vezes, o polinômio ou é par
> ou é ímpar.
>
> Mas se P(x) fosse ímpar, P(0)=0. E sabemos que P(x)!=x. Logo, o
> polinomio procurado é par.
>
> Meu problema está sendo o seguinte: quando eu testo um polinômio (isso
> mesmo, P(x)=x^4+bx^2+c, substituir e suar a caneta!), ele falha
> miseravelmente.
> Creio que este problema está sem solução, also...
>
> Em 01/07/11, Ralph Teixeira escreveu:
>> Melhorando aos poucos, ainda usando as ideias do Dirichlet: p(x) não pode
>> ser ímpar. Se fosse, 0 seria raiz. Mas então 0^2+1=1 seria raiz, e 1^2+1=2
>> seria raiz, e 2^2+1=5 seria raiz... e p(x) não pode ter infinitas raízes.
>> Então estamos à procura de um polinômio **par** p(x) tal que
>> p(x^2+1)=[p(x)]^2.
>>
>> Aliás, esse raciocínio mostra que esse p(x) não pode ter nenhuma raiz real
>> -- se tiver uma raiz real x, terá infinitas, já que x^2+1>x para todo x
>> real.
>>
>> (Por enquanto, fico com a terrível impressão de que tal polinômio não
>> existe... Alguém achou o dito cujo?)
>> 2011/7/1 Ralph Teixeira 
>>
>>> O raciocínio do Dirichlet mostra que basta achar UM polinômio (não
>>> constante) que tenha esta propriedade. Afinal, como ele mostrou, se p(x)
>>> serve, então q(x)=(p(x))^2 também serve.
>>>
>>> Mas seja lá quem for o polinômio mágico, eu sei que ou ele é um polinômio
>>> par ou ele é ímpar. Afinal, escreva p(x)=P(x)+I(x) onde P(x) tem apenas
>>> os
>>> termos de grau par e I(x) tem apenas os de grau ímpar.
>>>
>>> Ora, p(x)^2=(P^2+I^2)+2PI. Note que P^2+I^2 é um polinômio par e 2PI é
>>> ímpar.
>>>
>>> Mas a condição manda que p^2=p(x^2+1), que é uma função par. Então o
>>> termo
>>> 2PI não pode existir, isto é, P=0 ou I=0. Assim, p(x) é par ou ímpar. E
>>> x^2-x+1 não é um nem outro, então não funcionou...
>>>
>>> Então precisamos ainda mostrar que existe UM tal polinômio!
>>>
>>> Abraço,
>>>   Ralph
>>>
>>> P.S.: Tem certeza que o enunciado é esse mesmo? Não seria, sei lá,
>>> p(x^2+1)=(p(x))^2+1 ao invés?
>>> 2011/7/1 Johann Dirichlet 
>>>
>>>> Em 01/07/11, Johann Dirichlet escreveu:
>>>> > Em 30/06/11, marcone augusto araújo
>>>> > borges escreveu:
>>>> >>
>>>> >> 1) Se p é inteiro primo ímpar,mostre que o numerador da fração
>>>> >> 1+1/2+1/3+...1/(p-1) é um múltiplo de p.
>>>> >
>>>> > 1) Teorema de Wolstenholme, se não me engano...
>>>> >
>>>> > Bora lá, usar o velho truque das pontas de Gauss:
>>>> 1/k+1/(p-k)=p/(k(p-k));
>>>> > assim sendo, temos um monte de frações p/(alguma coisa). Esta coisa
>>>> > não será múltipla de p em momento nenhum, logo nada aniquila este
>>>> > fator p.
>>>> >
>>>> >>
>>>> >> 2) Mostre que existem infinitos polinômios p(x) com coeficientes
>>>> >> reais
>>>> >> tais
>>>> >> que p(x^2+1) = [p(x)]^2.
>>>>
>>>> É mais mole do que eu pensei!
>>>>
>>>> 1 - Se P e Q são soluções da equação acima, P*Q também será. Óbvio!
>>>> 2 - 

Re: [obm-l] P(x^2+1)=P(x)^2+1

2011-07-07 Por tôpico Johann Dirichlet
Bem, voltando ao novo problema:
P(x^2+1)=(P(x))^2+1.

O polinômio é mônico, basta aplicar uma ideia básica de limite para
saber o valor do coeficiente líder de P.

Primeiro, se P(c)=c para algum c real então P(x)=x. É só usar a
formula acima para achar infinitos x tais que P(x)=x. E dois
polinomios que se intersectam infinitamente são iguais.

Logo, podemos supor P(x) != x para todo x.

O polinomio acima ou é par ou é ímpar. Basta ver que
(P(x))^2=(P(-x))^2, e ou P(x)=P(-x) pu P(x)=-P(-x). Como pelo menos
uma destas alternativas ocorre infinitas vezes, o polinômio ou é par
ou é ímpar.

Mas se P(x) fosse ímpar, P(0)=0. E sabemos que P(x)!=x. Logo, o
polinomio procurado é par.

Meu problema está sendo o seguinte: quando eu testo um polinômio (isso
mesmo, P(x)=x^4+bx^2+c, substituir e suar a caneta!), ele falha
miseravelmente.
Creio que este problema está sem solução, also...

Em 01/07/11, Ralph Teixeira escreveu:
> Melhorando aos poucos, ainda usando as ideias do Dirichlet: p(x) não pode
> ser ímpar. Se fosse, 0 seria raiz. Mas então 0^2+1=1 seria raiz, e 1^2+1=2
> seria raiz, e 2^2+1=5 seria raiz... e p(x) não pode ter infinitas raízes.
> Então estamos à procura de um polinômio **par** p(x) tal que
> p(x^2+1)=[p(x)]^2.
>
> Aliás, esse raciocínio mostra que esse p(x) não pode ter nenhuma raiz real
> -- se tiver uma raiz real x, terá infinitas, já que x^2+1>x para todo x
> real.
>
> (Por enquanto, fico com a terrível impressão de que tal polinômio não
> existe... Alguém achou o dito cujo?)
> 2011/7/1 Ralph Teixeira 
>
>> O raciocínio do Dirichlet mostra que basta achar UM polinômio (não
>> constante) que tenha esta propriedade. Afinal, como ele mostrou, se p(x)
>> serve, então q(x)=(p(x))^2 também serve.
>>
>> Mas seja lá quem for o polinômio mágico, eu sei que ou ele é um polinômio
>> par ou ele é ímpar. Afinal, escreva p(x)=P(x)+I(x) onde P(x) tem apenas os
>> termos de grau par e I(x) tem apenas os de grau ímpar.
>>
>> Ora, p(x)^2=(P^2+I^2)+2PI. Note que P^2+I^2 é um polinômio par e 2PI é
>> ímpar.
>>
>> Mas a condição manda que p^2=p(x^2+1), que é uma função par. Então o termo
>> 2PI não pode existir, isto é, P=0 ou I=0. Assim, p(x) é par ou ímpar. E
>> x^2-x+1 não é um nem outro, então não funcionou...
>>
>> Então precisamos ainda mostrar que existe UM tal polinômio!
>>
>> Abraço,
>>   Ralph
>>
>> P.S.: Tem certeza que o enunciado é esse mesmo? Não seria, sei lá,
>> p(x^2+1)=(p(x))^2+1 ao invés?
>> 2011/7/1 Johann Dirichlet 
>>
>>> Em 01/07/11, Johann Dirichlet escreveu:
>>> > Em 30/06/11, marcone augusto araújo
>>> > borges escreveu:
>>> >>
>>> >> 1) Se p é inteiro primo ímpar,mostre que o numerador da fração
>>> >> 1+1/2+1/3+...1/(p-1) é um múltiplo de p.
>>> >
>>> > 1) Teorema de Wolstenholme, se não me engano...
>>> >
>>> > Bora lá, usar o velho truque das pontas de Gauss:
>>> 1/k+1/(p-k)=p/(k(p-k));
>>> > assim sendo, temos um monte de frações p/(alguma coisa). Esta coisa
>>> > não será múltipla de p em momento nenhum, logo nada aniquila este
>>> > fator p.
>>> >
>>> >>
>>> >> 2) Mostre que existem infinitos polinômios p(x) com coeficientes reais
>>> >> tais
>>> >> que p(x^2+1) = [p(x)]^2.
>>>
>>> É mais mole do que eu pensei!
>>>
>>> 1 - Se P e Q são soluções da equação acima, P*Q também será. Óbvio!
>>> 2 - Um polinômio possível é x^2-x+1.
>>> Como sei? Simples:
>>>
>>> Se L é um zero de P, então L^2+1 também será.
>>> Se eu conseguir L=L^2+1, terei uma solução pronta!
>>> Basta abrir o polinomio sem medo.
>>>
>>>
>>> P.S.: saber todas as soluções me parece mais desgastante. Aplicando a
>>> transformação T(L)=L^2+1 um numero finito de vezes, todos os
>>> polinômios dos pontos fixos são soluções. A treta é saber se não
>>> escapa nenhum (até porque muitos desses polinomios são fatoráveis, I
>>> think so).
>>>
>>> >>
>>> >> 3) Uma corda AB,de comprimento constante,desliza sobre uma
>>> >> semicircunferência determinada por um diâmetro d.
>>> >> Considere o triângulo cujos vértices são: o ponto médio da corda e as
>>> >> projeções ortogonais dos seus extremos A e B
>>> >> sobre o diâmetro d.Mostre que ,durante o deslizamento da corda,esse
>>> >> triângulo é sempre isósceles e nunca muda de formato(i.é.,os ângulos
>>> >> do
>>> >> triângulo são constantes)
>>> >
&g

Re: [obm-l] Problemas(polinomoi- ideias!)

2011-07-06 Por tôpico Johann Dirichlet
Continuando: acho que, quando se faz alguma manipulação algébrica, a
conta falha miseravelmente para graus grandes.

Usando a ideia do Ralph, o polinomio em questão é par ou ímpar. Mas
quando eu abro as contas, usando um "exemplo finito" (uma tentativa do
genero f(x)=ax^2+bx+c), dá muito desencontro de expoentes. Vou mostrar
as contas outro dia (elas sao grandinhas!).

Em 05/07/11, Johann Dirichlet escreveu:
> Eu, na verdade, tentei achar um polinomio que desse certo. E cantei
> vitória antes do tempo...
> E a sua ideia de par-ou-impar matou de vez as esperanças: L^2+1
> aumenta o módulo.
>
> O Marcone tambem me enviou este e-mail corrigido. Eu estou matutando
> nele, e achei alguns exemplos. Ao que me parece, para cada grau de
> polinomio, existe um único polinômio que resolve a equação funcional.
>
> Eu descobri que estes poucos polinomios tem coeficiente líder 1 e
> sub-líder 0. Mas não avencei muito para dar um parecer final.
>
> No mais, acho que x^2+1 me lembra de usar i^2=-1...
>
> Em 04/07/11, Ralph Teixeira escreveu:
>> Como voce disse, se a eh uma raiz de P(x), entao a^2+1 tem que ser raiz
>> de
>> P(x) tambem. Entao se voce pegar as raizes de P(x) e "aplicar" x^2+1
>> nelas,
>> voce ainda tem que cair em raizes. Portanto, dada uma raiz qualquer a,
>> temos
>> que a^2+1, (a^2+1)^2+1, etc. gera varias raizes de P(x). Como P(x) tem
>> que
>> ter um numero finito de raizes distintas, essa sequencia tem que gerar um
>> ciclo, tem que repetir em algum momento.
>>
>> Que ciclo? O caso mais simples seria fazer um ciclo de tamanho 1, ou
>> seja,
>> fazer logo a^2+1=a, para o ciclo soh ter um termo. Foi a ideia que voce
>> colocou... mas nao dah certo -- a condicao deste ciclo ser finita eh
>> NECESSARIA para ter a igualdade pedida, mas ter um ciclo de raizes nao eh
>> SUFICENTE para garantir a igualdade pedida.
>>
>> O Shine botou a bola embaixo do braco e levou para casa: nao ha polinomio
>> com a condicao pedida... :)
>>
>> Mas, olha soh:* o Marcone, que propos o problema original, me mandou um
>> E-mail dizendo que realmente o enunciado original era mesmo
>> p(x^2+1)=(p(x))^2+1, mas por algum motivo ele nao conseguiu colocar a
>> correcao na lista.* Entao ainda ha um problema interessante (mas bem
>> diferente) para fazer.
>>
>> (Eu jah vi isso em algum lugar, mas nao lembro onde...)
>>
>> Abraco,
>> Ralph
>> 2011/7/4 Johann Dirichlet 
>>
>>> Puxa! Mas onde esta o erro da minha solução?
>>>
>>> Anyway, inicialmente pensei em fatorar o dito polinomio.
>>> Creio que ele seja mônico, abrindo a expressão geral o fator máximo é
>>> a^2=a.
>>> Aí, escreve ele na forma deprodutos (x-a_i).. Basicamente, um lado
>>> fica na forma
>>> x^2+1-a_ i, e o outro como (x-a_ i)^2. Supóndo que as raízes são, em
>>> alguma ordem, iguais, dá pra chegar em algum lugar.
>>>
>>>
>>>
>>>
>>> Em 01/07/11, Ralph Teixeira escreveu:
>>> > O raciocínio do Dirichlet mostra que basta achar UM polinômio (não
>>> > constante) que tenha esta propriedade. Afinal, como ele mostrou, se
>>> > p(x)
>>> > serve, então q(x)=(p(x))^2 também serve.
>>> >
>>> > Mas seja lá quem for o polinômio mágico, eu sei que ou ele é um
>>> > polinômio
>>> > par ou ele é ímpar. Afinal, escreva p(x)=P(x)+I(x) onde P(x) tem
>>> > apenas
>>> os
>>> > termos de grau par e I(x) tem apenas os de grau ímpar.
>>> >
>>> > Ora, p(x)^2=(P^2+I^2)+2PI. Note que P^2+I^2 é um polinômio par e 2PI é
>>> > ímpar.
>>> >
>>> > Mas a condição manda que p^2=p(x^2+1), que é uma função par. Então o
>>> termo
>>> > 2PI não pode existir, isto é, P=0 ou I=0. Assim, p(x) é par ou ímpar.
>>> > E
>>> > x^2-x+1 não é um nem outro, então não funcionou...
>>> >
>>> > Então precisamos ainda mostrar que existe UM tal polinômio!
>>> >
>>> > Abraço,
>>> >   Ralph
>>> >
>>> > P.S.: Tem certeza que o enunciado é esse mesmo? Não seria, sei lá,
>>> > p(x^2+1)=(p(x))^2+1 ao invés?
>>> > 2011/7/1 Johann Dirichlet 
>>> >
>>> >> Em 01/07/11, Johann Dirichlet escreveu:
>>> >> > Em 30/06/11, marcone augusto araújo
>>> >> > borges escreveu:
>>> >> >>
>>> >> >> 1) Se p é inteiro primo ímpar,mostre que o numerador da fração
>>> >> >> 1+1/2+1/3

Re: [obm-l] Problemas(polinomoi- ideias!)

2011-07-05 Por tôpico Johann Dirichlet
Eu, na verdade, tentei achar um polinomio que desse certo. E cantei
vitória antes do tempo...
E a sua ideia de par-ou-impar matou de vez as esperanças: L^2+1
aumenta o módulo.

O Marcone tambem me enviou este e-mail corrigido. Eu estou matutando
nele, e achei alguns exemplos. Ao que me parece, para cada grau de
polinomio, existe um único polinômio que resolve a equação funcional.

Eu descobri que estes poucos polinomios tem coeficiente líder 1 e
sub-líder 0. Mas não avencei muito para dar um parecer final.

No mais, acho que x^2+1 me lembra de usar i^2=-1...

Em 04/07/11, Ralph Teixeira escreveu:
> Como voce disse, se a eh uma raiz de P(x), entao a^2+1 tem que ser raiz de
> P(x) tambem. Entao se voce pegar as raizes de P(x) e "aplicar" x^2+1 nelas,
> voce ainda tem que cair em raizes. Portanto, dada uma raiz qualquer a, temos
> que a^2+1, (a^2+1)^2+1, etc. gera varias raizes de P(x). Como P(x) tem que
> ter um numero finito de raizes distintas, essa sequencia tem que gerar um
> ciclo, tem que repetir em algum momento.
>
> Que ciclo? O caso mais simples seria fazer um ciclo de tamanho 1, ou seja,
> fazer logo a^2+1=a, para o ciclo soh ter um termo. Foi a ideia que voce
> colocou... mas nao dah certo -- a condicao deste ciclo ser finita eh
> NECESSARIA para ter a igualdade pedida, mas ter um ciclo de raizes nao eh
> SUFICENTE para garantir a igualdade pedida.
>
> O Shine botou a bola embaixo do braco e levou para casa: nao ha polinomio
> com a condicao pedida... :)
>
> Mas, olha soh:* o Marcone, que propos o problema original, me mandou um
> E-mail dizendo que realmente o enunciado original era mesmo
> p(x^2+1)=(p(x))^2+1, mas por algum motivo ele nao conseguiu colocar a
> correcao na lista.* Entao ainda ha um problema interessante (mas bem
> diferente) para fazer.
>
> (Eu jah vi isso em algum lugar, mas nao lembro onde...)
>
> Abraco,
> Ralph
> 2011/7/4 Johann Dirichlet 
>
>> Puxa! Mas onde esta o erro da minha solução?
>>
>> Anyway, inicialmente pensei em fatorar o dito polinomio.
>> Creio que ele seja mônico, abrindo a expressão geral o fator máximo é
>> a^2=a.
>> Aí, escreve ele na forma deprodutos (x-a_i).. Basicamente, um lado
>> fica na forma
>> x^2+1-a_ i, e o outro como (x-a_ i)^2. Supóndo que as raízes são, em
>> alguma ordem, iguais, dá pra chegar em algum lugar.
>>
>>
>>
>>
>> Em 01/07/11, Ralph Teixeira escreveu:
>> > O raciocínio do Dirichlet mostra que basta achar UM polinômio (não
>> > constante) que tenha esta propriedade. Afinal, como ele mostrou, se p(x)
>> > serve, então q(x)=(p(x))^2 também serve.
>> >
>> > Mas seja lá quem for o polinômio mágico, eu sei que ou ele é um
>> > polinômio
>> > par ou ele é ímpar. Afinal, escreva p(x)=P(x)+I(x) onde P(x) tem apenas
>> os
>> > termos de grau par e I(x) tem apenas os de grau ímpar.
>> >
>> > Ora, p(x)^2=(P^2+I^2)+2PI. Note que P^2+I^2 é um polinômio par e 2PI é
>> > ímpar.
>> >
>> > Mas a condição manda que p^2=p(x^2+1), que é uma função par. Então o
>> termo
>> > 2PI não pode existir, isto é, P=0 ou I=0. Assim, p(x) é par ou ímpar. E
>> > x^2-x+1 não é um nem outro, então não funcionou...
>> >
>> > Então precisamos ainda mostrar que existe UM tal polinômio!
>> >
>> > Abraço,
>> >   Ralph
>> >
>> > P.S.: Tem certeza que o enunciado é esse mesmo? Não seria, sei lá,
>> > p(x^2+1)=(p(x))^2+1 ao invés?
>> > 2011/7/1 Johann Dirichlet 
>> >
>> >> Em 01/07/11, Johann Dirichlet escreveu:
>> >> > Em 30/06/11, marcone augusto araújo
>> >> > borges escreveu:
>> >> >>
>> >> >> 1) Se p é inteiro primo ímpar,mostre que o numerador da fração
>> >> >> 1+1/2+1/3+...1/(p-1) é um múltiplo de p.
>> >> >
>> >> > 1) Teorema de Wolstenholme, se não me engano...
>> >> >
>> >> > Bora lá, usar o velho truque das pontas de Gauss:
>> >> > 1/k+1/(p-k)=p/(k(p-k));
>> >> > assim sendo, temos um monte de frações p/(alguma coisa). Esta coisa
>> >> > não será múltipla de p em momento nenhum, logo nada aniquila este
>> >> > fator p.
>> >> >
>> >> >>
>> >> >> 2) Mostre que existem infinitos polinômios p(x) com coeficientes
>> reais
>> >> >> tais
>> >> >> que p(x^2+1) = [p(x)]^2.
>> >>
>> >> É mais mole do que eu pensei!
>> >>
>> >> 1 - Se P e Q são soluções da equação acima, P*Q também será. Óbvio

Re: [obm-l] Problemas(polinomoi- ideias!)

2011-07-04 Por tôpico Johann Dirichlet
Sobre a sua mesg: por "indução infinita" na derivação, nunca acharemos
um polinômio?

Comments:
Bem, não tô com a mensagem full, mas basicamente o Ralph provou que o
polinômio é par ou ímpar. Não dá pra ser coluna do meio.
Realmente, raiz 0 falha miseravelmente, isto é imediato. Mas, o lance
é: não existe nenhum polinômio que funcione??

Realmente,pensando na minha ideia das raizes, pareece que L^2+1 tem
modulo maior que L (uma desigualdade das medias mataria isso). Ou
seja, não tem como, depois de aplicar L^2+1 um numero finito de vezes,
voltar a L.

Quanto a outra, comecei a atacar hoje.


Em 04/07/11, Carlos Yuzo Shine escreveu:
> Eu acho que não existem infinitos polinômios (a única solução é o polinômio
> nulo). Isso, é claro, falando do problema P(x^2 + 1) = [P(x)]^2.
>
> Antes, lembramos que zero não pode ser raiz de P: como o Ralph já provou,
> isso
> gera infinitas raízes e aí P é o polinômio nulo.
>
> Olha só: derivando P(x^2 + 1) = P(x)^2 dos dois lados encontramos
> 2xP'(x^2+1) =
> 2P(x)P'(x). Aí vou tentar usar uma ideia parecida com a do Ralph. x = 0 só
> pode
> ser raiz de P' (o que atesta o fato de P ser par, que também foi provado).
> Agora
> note que se x diferente de zero é raiz de P' então x^2 + 1 também é. Pena
> que só
> temos o zero. Mas aí fazemos de trás para frente (isto é, aplicamos uma
> inversa
> f de x^2 + 1): troca x = f(0) = i. Então i é raiz de P ou P'. Mas se for
> raiz de
> P, então i^2 + 1 = 0 é raiz e dá problema do mesmo jeito. Então é raiz de
> P'.
> Mas aí é só continuar.
>
> Definindo f melhor: f(r) é a raiz quadrada de r - 1 que tem argumento arg(r
> -
> 1)/2. A minha pergunta é: partindo do r = 0, ele entra em loop?
>
> Eu acho que não. De fato, f é injetiva: f(r) = f(s) implica (f(r))^2 + 1 =
> (f(s))^2 + 1 que é o mesmo que r = s. Então se a sequência n_0 = 0 e n_k =
> f(n_{k-1}) é periódica, ela é puramente periódica. Mas 0 não é periódica
> para
> f^{-1}(x) = x^2 + 1, então acabou (eu acho).
>
>
> Isso faz sentido?
>
> []'s
> Shine
>
>
> - Original Message 
> From: Johann Dirichlet 
> To: obm-l@mat.puc-rio.br
> Sent: Mon, July 4, 2011 12:46:11 PM
> Subject: Re: [obm-l] Problemas(polinomoi- ideias!)
>
> Puxa! Mas onde esta o erro da minha solução?
>
> Anyway, inicialmente pensei em fatorar o dito polinomio.
> Creio que ele seja mônico, abrindo a expressão geral o fator máximo é a^2=a.
> Aí, escreve ele na forma deprodutos (x-a_i).. Basicamente, um lado
> fica na forma
> x^2+1-a_ i, e o outro como (x-a_ i)^2. Supóndo que as raízes são, em
> alguma ordem, iguais, dá pra chegar em algum lugar.
>
>
>
>
> Em 01/07/11, Ralph Teixeira escreveu:
>> O raciocínio do Dirichlet mostra que basta achar UM polinômio (não
>> constante) que tenha esta propriedade. Afinal, como ele mostrou, se p(x)
>> serve, então q(x)=(p(x))^2 também serve.
>>
>> Mas seja lá quem for o polinômio mágico, eu sei que ou ele é um polinômio
>> par ou ele é ímpar. Afinal, escreva p(x)=P(x)+I(x) onde P(x) tem apenas os
>> termos de grau par e I(x) tem apenas os de grau ímpar.
>>
>> Ora, p(x)^2=(P^2+I^2)+2PI. Note que P^2+I^2 é um polinômio par e 2PI é
>> ímpar.
>>
>> Mas a condição manda que p^2=p(x^2+1), que é uma função par. Então o termo
>> 2PI não pode existir, isto é, P=0 ou I=0. Assim, p(x) é par ou ímpar. E
>> x^2-x+1 não é um nem outro, então não funcionou...
>>
>> Então precisamos ainda mostrar que existe UM tal polinômio!
>>
>> Abraço,
>>   Ralph
>>
>> P.S.: Tem certeza que o enunciado é esse mesmo? Não seria, sei lá,
>> p(x^2+1)=(p(x))^2+1 ao invés?
>> 2011/7/1 Johann Dirichlet 
>>
>>> Em 01/07/11, Johann Dirichlet escreveu:
>>> > Em 30/06/11, marcone augusto araújo
>>> > borges escreveu:
>>> >>
>>> >> 1) Se p é inteiro primo ímpar,mostre que o numerador da fração
>>> >> 1+1/2+1/3+...1/(p-1) é um múltiplo de p.
>>> >
>>> > 1) Teorema de Wolstenholme, se não me engano...
>>> >
>>> > Bora lá, usar o velho truque das pontas de Gauss:
>>> > 1/k+1/(p-k)=p/(k(p-k));
>>> > assim sendo, temos um monte de frações p/(alguma coisa). Esta coisa
>>> > não será múltipla de p em momento nenhum, logo nada aniquila este
>>> > fator p.
>>> >
>>> >>
>>> >> 2) Mostre que existem infinitos polinômios p(x) com coeficientes reais
>>> >> tais
>>> >> que p(x^2+1) = [p(x)]^2.
>>>
>>> É mais mole do que eu pensei!
>>>
>>> 1 - Se P e Q são soluções da equação 

Re: [obm-l] Problemas(polinomoi- ideias!)

2011-07-04 Por tôpico Johann Dirichlet
Puxa! Mas onde esta o erro da minha solução?

Anyway, inicialmente pensei em fatorar o dito polinomio.
Creio que ele seja mônico, abrindo a expressão geral o fator máximo é a^2=a.
Aí, escreve ele na forma deprodutos (x-a_i).. Basicamente, um lado
fica na forma
x^2+1-a_ i, e o outro como (x-a_ i)^2. Supóndo que as raízes são, em
alguma ordem, iguais, dá pra chegar em algum lugar.




Em 01/07/11, Ralph Teixeira escreveu:
> O raciocínio do Dirichlet mostra que basta achar UM polinômio (não
> constante) que tenha esta propriedade. Afinal, como ele mostrou, se p(x)
> serve, então q(x)=(p(x))^2 também serve.
>
> Mas seja lá quem for o polinômio mágico, eu sei que ou ele é um polinômio
> par ou ele é ímpar. Afinal, escreva p(x)=P(x)+I(x) onde P(x) tem apenas os
> termos de grau par e I(x) tem apenas os de grau ímpar.
>
> Ora, p(x)^2=(P^2+I^2)+2PI. Note que P^2+I^2 é um polinômio par e 2PI é
> ímpar.
>
> Mas a condição manda que p^2=p(x^2+1), que é uma função par. Então o termo
> 2PI não pode existir, isto é, P=0 ou I=0. Assim, p(x) é par ou ímpar. E
> x^2-x+1 não é um nem outro, então não funcionou...
>
> Então precisamos ainda mostrar que existe UM tal polinômio!
>
> Abraço,
>   Ralph
>
> P.S.: Tem certeza que o enunciado é esse mesmo? Não seria, sei lá,
> p(x^2+1)=(p(x))^2+1 ao invés?
> 2011/7/1 Johann Dirichlet 
>
>> Em 01/07/11, Johann Dirichlet escreveu:
>> > Em 30/06/11, marcone augusto araújo
>> > borges escreveu:
>> >>
>> >> 1) Se p é inteiro primo ímpar,mostre que o numerador da fração
>> >> 1+1/2+1/3+...1/(p-1) é um múltiplo de p.
>> >
>> > 1) Teorema de Wolstenholme, se não me engano...
>> >
>> > Bora lá, usar o velho truque das pontas de Gauss:
>> > 1/k+1/(p-k)=p/(k(p-k));
>> > assim sendo, temos um monte de frações p/(alguma coisa). Esta coisa
>> > não será múltipla de p em momento nenhum, logo nada aniquila este
>> > fator p.
>> >
>> >>
>> >> 2) Mostre que existem infinitos polinômios p(x) com coeficientes reais
>> >> tais
>> >> que p(x^2+1) = [p(x)]^2.
>>
>> É mais mole do que eu pensei!
>>
>> 1 - Se P e Q são soluções da equação acima, P*Q também será. Óbvio!
>> 2 - Um polinômio possível é x^2-x+1.
>> Como sei? Simples:
>>
>> Se L é um zero de P, então L^2+1 também será.
>> Se eu conseguir L=L^2+1, terei uma solução pronta!
>> Basta abrir o polinomio sem medo.
>>
>>
>> P.S.: saber todas as soluções me parece mais desgastante. Aplicando a
>> transformação T(L)=L^2+1 um numero finito de vezes, todos os
>> polinômios dos pontos fixos são soluções. A treta é saber se não
>> escapa nenhum (até porque muitos desses polinomios são fatoráveis, I
>> think so).
>>
>> >>
>> >> 3) Uma corda AB,de comprimento constante,desliza sobre uma
>> >> semicircunferência determinada por um diâmetro d.
>> >> Considere o triângulo cujos vértices são: o ponto médio da corda e as
>> >> projeções ortogonais dos seus extremos A e B
>> >> sobre o diâmetro d.Mostre que ,durante o deslizamento da corda,esse
>> >> triângulo é sempre isósceles e nunca muda de formato(i.é.,os ângulos do
>> >> triângulo são constantes)
>> >
>> > Faz um desenho!
>> > Diâmetro r;centro O, raio 1; corda AB, tamanho d, médio M; AB
>> > projetado em r dá XY.
>> >
>> > O triangulo AOB é obviamente isósceles.
>> > Os quadrilateros XOMA e YOMB são inscritíveis de diâmetros OA e OB
>> > respectivamente (angulos de 90 graus).
>> >
>> > Temos OXM=OAM=OBM=OYM, logo XMY é isosceles. E o angulo OBA depende
>> > unicamente de d.
>> >
>> > P.S.: duvido que os triangulos sejam todos congruentes. O angulo XOM
>> > define o tamanho de XM.
>> >
>> >>
>> >> Meus agradecimentos por qualquer esclarecimento.
>> >
>> >
>> > --
>> > /**/
>> > 神が祝福
>> >
>> > Torres
>> >
>>
>>
>> --
>> /**/
>> 神が祝福
>>
>> Torres
>>
>> =
>> Instru�ões para entrar na lista, sair da lista e usar a lista em
>> http://www.mat.puc-rio.br/~obmlistas/obm-l.html
>> =
>>
>


-- 
/**/
神が祝福

Torres

=
Instru��es para entrar na lista, sair da lista e usar a lista em
http://www.mat.puc-rio.br/~obmlistas/obm-l.html
=


[obm-l] Correção do Enunciado [era: Enunciado errado(lista da obm)]

2011-07-04 Por tôpico Johann Dirichlet
Reescrevendo: p(x^2+1)=(p(x))^2+1
É isto?
Vou dar uma pensada, mas acho que a ideia das raízes ainda rola... Ou
um dose de complexos?

Em 02/07/11, marcone augusto araújo
borges escreveu:
>
> Desculpe enviar diretamente,tentei várias vezes pela lista,não consegui.
> Na questão 2,onde tá escrito [p(x)]^2,o correto é [p(x)]^2 + 1.
> Perdão pelo erro.
> Abraço,Marcone.   


-- 
/**/
神が祝福

Torres

=
Instru��es para entrar na lista, sair da lista e usar a lista em
http://www.mat.puc-rio.br/~obmlistas/obm-l.html
=


Re: [obm-l] Problemas(ajuda)

2011-07-01 Por tôpico Johann Dirichlet
Em 01/07/11, Johann Dirichlet escreveu:
> Em 30/06/11, marcone augusto araújo
> borges escreveu:
>>
>> 1) Se p é inteiro primo ímpar,mostre que o numerador da fração
>> 1+1/2+1/3+...1/(p-1) é um múltiplo de p.
>
> 1) Teorema de Wolstenholme, se não me engano...
>
> Bora lá, usar o velho truque das pontas de Gauss: 1/k+1/(p-k)=p/(k(p-k));
> assim sendo, temos um monte de frações p/(alguma coisa). Esta coisa
> não será múltipla de p em momento nenhum, logo nada aniquila este
> fator p.
>
>>
>> 2) Mostre que existem infinitos polinômios p(x) com coeficientes reais
>> tais
>> que p(x^2+1) = [p(x)]^2.

É mais mole do que eu pensei!

1 - Se P e Q são soluções da equação acima, P*Q também será. Óbvio!
2 - Um polinômio possível é x^2-x+1.
Como sei? Simples:

Se L é um zero de P, então L^2+1 também será.
Se eu conseguir L=L^2+1, terei uma solução pronta!
Basta abrir o polinomio sem medo.


P.S.: saber todas as soluções me parece mais desgastante. Aplicando a
transformação T(L)=L^2+1 um numero finito de vezes, todos os
polinômios dos pontos fixos são soluções. A treta é saber se não
escapa nenhum (até porque muitos desses polinomios são fatoráveis, I
think so).

>>
>> 3) Uma corda AB,de comprimento constante,desliza sobre uma
>> semicircunferência determinada por um diâmetro d.
>> Considere o triângulo cujos vértices são: o ponto médio da corda e as
>> projeções ortogonais dos seus extremos A e B
>> sobre o diâmetro d.Mostre que ,durante o deslizamento da corda,esse
>> triângulo é sempre isósceles e nunca muda de formato(i.é.,os ângulos do
>> triângulo são constantes)
>
> Faz um desenho!
> Diâmetro r;centro O, raio 1; corda AB, tamanho d, médio M; AB
> projetado em r dá XY.
>
> O triangulo AOB é obviamente isósceles.
> Os quadrilateros XOMA e YOMB são inscritíveis de diâmetros OA e OB
> respectivamente (angulos de 90 graus).
>
> Temos OXM=OAM=OBM=OYM, logo XMY é isosceles. E o angulo OBA depende
> unicamente de d.
>
> P.S.: duvido que os triangulos sejam todos congruentes. O angulo XOM
> define o tamanho de XM.
>
>>
>> Meus agradecimentos por qualquer esclarecimento. 
>> 
>
>
> --
> /**/
> 神が祝福
>
> Torres
>


-- 
/**/
神が祝福

Torres

=
Instru��es para entrar na lista, sair da lista e usar a lista em
http://www.mat.puc-rio.br/~obmlistas/obm-l.html
=


Re: [obm-l] Problemas(ajuda)

2011-07-01 Por tôpico Johann Dirichlet
Em 30/06/11, marcone augusto araújo
borges escreveu:
>
> 1) Se p é inteiro primo ímpar,mostre que o numerador da fração
> 1+1/2+1/3+...1/(p-1) é um múltiplo de p.

1) Teorema de Wolstenholme, se não me engano...

Bora lá, usar o velho truque das pontas de Gauss: 1/k+1/(p-k)=p/(k(p-k));
assim sendo, temos um monte de frações p/(alguma coisa). Esta coisa
não será múltipla de p em momento nenhum, logo nada aniquila este
fator p.

>
> 2) Mostre que existem infinitos polinômios p(x) com coeficientes reais tais
> que p(x^2+1) = [p(x)]^2.
>
> 3) Uma corda AB,de comprimento constante,desliza sobre uma
> semicircunferência determinada por um diâmetro d.
> Considere o triângulo cujos vértices são: o ponto médio da corda e as
> projeções ortogonais dos seus extremos A e B
> sobre o diâmetro d.Mostre que ,durante o deslizamento da corda,esse
> triângulo é sempre isósceles e nunca muda de formato(i.é.,os ângulos do
> triângulo são constantes)

Faz um desenho!
Diâmetro r;centro O, raio 1; corda AB, tamanho d, médio M; AB
projetado em r dá XY.

O triangulo AOB é obviamente isósceles.
Os quadrilateros XOMA e YOMB são inscritíveis de diâmetros OA e OB
respectivamente (angulos de 90 graus).

Temos OXM=OAM=OBM=OYM, logo XMY é isosceles. E o angulo OBA depende
unicamente de d.

P.S.: duvido que os triangulos sejam todos congruentes. O angulo XOM
define o tamanho de XM.

>
> Meus agradecimentos por qualquer esclarecimento.  
> 


-- 
/**/
神が祝福

Torres

=
Instru��es para entrar na lista, sair da lista e usar a lista em
http://www.mat.puc-rio.br/~obmlistas/obm-l.html
=


[obm-l] Re: [obm-l] Re: [obm-l] questão trigonometria complicada

2011-06-28 Por tôpico Johann Dirichlet
Este foi um problema da revista Kvant, na verdade um artigo.

Eis o site (pra quem encarar um russinho básico...)
http://kvant.mccme.ru/


Em 27/06/11, Ralph Teixeira escreveu:
> Ah, já vi errinho de sinal no meio do caminho, no sinal de ab+ac+bc e no de
> abc. Corrigi abaixo, mas deve haver outros. De qualquer forma, a ideia ainda
> vale.
> 2011/6/27 Ralph Teixeira 
>
>> Hmmm, vejamos. Será que a gente arruma algum polinômio cujas raízes sejam
>> as 3 parcelas da sua soma?
>>
>> Considere a "famosa" identidade
>> trigonométrica sin7t=(8(cos2t)^3+4(cos2t)^2-4(cos2t)-1).sint
>>
>> (Desculpa, não pude resistir.)
>>
>> Note que t=kpi/7 (k=1,2,4) dá três raízes de sin7t, mas nenhum deles dá
>> raiz de sint. Então estes valores de t devem anular o termo entre
>> parênteses... Em outras palavras, se você considerar o polinômio
>> P(x)=8x^3+4x^2-4x-1, você verá que suas raízes são exatamente cos(2pi/7),
>> cos(4pi/7) e cos(8pi/7) -- exatamente porque é um polinômio do 3o grau,
>> então se eu achei 3 raízes distintas, achei todas.
>>
>> (O argumento também vale para k=3,5,6, mas então obtemos
>> cos(6pi/7)=cos(8pi/7), cos(10pi/7)=cos(4pi/7) e cos(12pi/7)=cos(2pi/7),
>> que
>> são aquelas raízes de novo)
>>
>> Em suma, o problema agora é: sejam a,b e c as raízes de
>> P(x)=8x^3+4x^2-4x-1. Encontre a^(1/3)+b^(1/3)+c^(1/3).
>>
>> Vou escrever a^(1/3)=A, b^(1/3)=B e c^(1/3)=C. Mas, do polinômio sabemos
>> que
>> a+b+c=-1/2, isto é, A^3+B^3+C^3=-1/2
>> ab+ac+bc=-1/2, isto é, A^3B^3+A^3C^3+B^3C^3=-1/2
>> abc=1/8, isto é, ABC=1/2.
>>
>> Poxa, eu até consigo fazer o resto, mas é HORRENDO. Vamos lá.
>>
>> Agora, talvez você já tenha visto a identidade
>> x^3+y^3+z^3-3xyz=(x+y+z)((x+y+z)^2-3(xy+xz+yz))
>>
>> Aplicando esta identidade com (x,y,z)=(A,B,C) temos:
>> -1/2-3/2=-2=S(S^2-3D) (onde S=A+B+C e D=AB+AC+BC)
>>
>> Aplicando esta identidade com (x,y,z)=(AB,AC,BC), temos:
>> -1/2-3(1/4)=-5/4=D(D^2-3SP)=D(D^2-3S/2) (onde P=ABC=1/2)
>>
>> Enfim, duas equações e duas incógnitas! Tire D da primeira e jogue na
>> segunda -- fica horrendo, mas dá uma equação polinomial de grau 9 em S,
>> com
>> termos apenas em S^3, S^6 e S^9. Faça S^3=Z, resolva a equação cúbica em
>> Z,
>> S é a raiz cúbica de Z.
>>
>> Argh! Tá, fiquei sem vontade de terminar as contas, e devo ter errado algo
>> no meio do caminho, mas saiu!
>>
>> Abraço,
>>Ralph
>>
>>
>> 2011/6/26 Jefferson Franca 
>>
>>> Boa tarde senhores. Será que alguém poderia me iluminar nesta questão:
>>> Calcule o valor da soma (cos(2*pi/7)^1/3 + (cos(4*pi/7))^1/3 +
>>> (cos(8*pi/7))^1/3 ?
>>> abs
>>>
>>
>>
>


-- 
/**/
神が祝福

Torres

=
Instru��es para entrar na lista, sair da lista e usar a lista em
http://www.mat.puc-rio.br/~obmlistas/obm-l.html
=


Re: [obm-l] Desigualdade (Como provar?)

2011-06-21 Por tôpico Johann Dirichlet
Você,Você,Você,Você,Você,Você,Você quer uma demo por PIF?
Bem, vou te dar a dica: prove de n para 2n, e depois de n para n-1.
Em 21/06/11, Carlos Nehab escreveu:> Oi, Paulo.>> É 
simples e clássico.> Basta usar média aritmética >= média geométrica em S e 
S'.>> Abraços> Nehab>> Em 21/6/2011 08:34, Paulo Argolo escreveu:>> Caros 
Colegas, Não consegui ainda uma demonstração. Seria possível fazê-la por 
indução>> finita? Abraços do Paulo.>> 
->
 Date: Mon, 13 Jun 2011 22:49:41 +0200>>> Subject: Re: [obm-l] Desigualdade 
(Como provar?)>>> From: bernardo...@gmail.com>>> To: obm-l@mat.puc-rio.br>> 
2011/6/13 Paulo Argolo: Caros Colegas, Como 
podemos provar que, dados n numeros reais positivos (n>1), nem todos 
iguais, vale a desigualdade abaixo? S . S'>  n^2 (S é a soma dos n 
números, S' é a soma dos inversos desses n números.)>>> Tente mostrar 
isso para n = 2, n = 3, expandindo tudo. Dá poucos>>> termos, e daí acho qu!
e você vai ver como prova para n qualquer.>> Abraços,>>> -->>> Bernardo 
Freitas Paulo da Costa>> 
=>>> 
Instruções para entrar na lista, sair da lista e usar a lista em>>> 
http://www.mat.puc-rio.br/~obmlistas/obm-l.html>>> 
=>>>
   >> 
=>> 
Instruções para entrar na lista, sair da lista e usar a lista em>> 
http://www.mat.puc-rio.br/~obmlistas/obm-l.html>> 
= 
=> 
Instruções para entrar na lista, sair da lista e usar a lista em> 
http://www.mat.puc-rio.br/~obmlistas/obm-l.html> 
=>

-- /**/神が祝福
Torres
=
Instru��es para entrar na lista, sair da lista e usar a lista em
http://www.mat.puc-rio.br/~obmlistas/obm-l.html
=


[obm-l] Re: [obm-l] Quadrilátero Circunscritível

2011-06-17 Por tôpico Johann Dirichlet
Engraçado, isso me lembra Brianchon:

"Em um hexagono circunscritivel, as diagonais principais sao
concorrentes" (na verdade ele vale para qualquer conica, nao so o
circulo).

Explicando: hexagono ABCDEF, diagonais AD,BE,CF.

Se usarmos uma "passagem ao limite", aproximando B e E da
circunferencia (fazendo estes angulos tenderem a 180 graus), os lados
AB e BC viram AC, e do mesmo modo produzimos DE.
Ou seja, AD, BE e CF tem um ponto comum. O que e isomorfo ao que voce
quer descobrir.

A demonstracao de Brianchon, bem, fica pra outra hora.

Em 09/06/11, Vitor Alves escreveu:
>
> Seja ABCD um quadrilátero circunscritível e E,F,G,H os pontos em que seu
> incírculo toca AB, BC, CD, DA, respectivamente. Prove que AC, BD, EG e FH
> são concorrentes. 


-- 
/**/
神が祝福

Torres

=
Instru��es para entrar na lista, sair da lista e usar a lista em
http://www.mat.puc-rio.br/~obmlistas/obm-l.html
=


[obm-l] Re: [obm-l] Re: [obm-l] Re: [obm-l] Re: [obm-l] FW: Teoria dos números

2011-05-30 Por tôpico Johann Dirichlet
O que eu posso fazer se eu sou lento e preguiçoso no computador?

Ah, deixa eu ficar ninja no dvorak... huahuahuahua!

Em 27/05/11, Rogerio Ponce escreveu:
> Pois e', Dirichlet, o Ralph tem este pessimo habito...
> :)
>
> []'s
> Rogerio Ponce
>
> Em 27 de maio de 2011 17:39, Ralph Teixeira  escreveu:
>
>> Yeah! Ninjei de novo! :) :) :) ;)
>> 2011/5/27 Johann Dirichlet 
>>
>>> Poxa! O Ralph destruiu minha mensagem! Mas acabei respondendo do mesmo
>>> jeito (ou nao!:))
>>>
>>> Em 27/05/11, Johann Dirichlet escreveu:
>>> > Ce já estudou congruencias? Um bom começo é pegar a Eureka! 2 na
>>> > página da OBM, www.obm.org.br (ou comprar da OBM! É baratinho, uma
>>> > anuidade de uns 30 reais e uns 4 contos por cada atrasado que quiser).
>>> > Anyway, vou tentar deixar fácil...
>>> >
>>> > 1)
>>> > 2^n=(x-1)(x^2+x+1)
>>> >
>>> > Vamos tentar calcular o MDC:
>>> > d|x-1
>>> > d|x^2+x+1
>>> >
>>> > x =1 (mod d)
>>> > x^2+x+1=0 (mod d)
>>> >
>>> > primeira na segunda, d|3. Como d=3 é impossível (potencias de 2 nao
>>> > tem fatores 3 :) ), d=1.
>>> >
>>> > Em especial, x-1=1 ou x^2+x+1=1 (ambos sao potencias de 2, e o MDC é
>>> > 1, logo um deles é 1).
>>> > Ou seja, x=2 ou 0. Substitui e chora!
>>> >
>>> > 2)
>>> > 7|4n^2-3
>>> > Multiplica por 2
>>> > 7|8n^2-6=n^2+1+(7n^2-7)
>>> >
>>> > 7|n^2+1
>>> > Por congruências, é possível provar que basta testar n de 0 a 6.Mas
>>> > vou usar descenso infinito.
>>> >
>>> > Teste de 0 a 6 (larga a mão de ser preguiçoso!). Vai falhar (eu acho :)
>>> ).
>>> >
>>> > Se funcionar para algum cara maior que 6, seja F o menor dos caras
>>> > para os quais funciona (se existe, existe o menor, este é o lema da
>>> > boa ordem).
>>> >
>>> > Seja J=F-7. Então J é maior ou igual a 0.
>>> > 7|(J+7)^2+1=J^2+2*7*J+7^2+1=7*(um termo chato que não interessa)+J^2+1
>>> > 7|J^2+1
>>> >
>>> > Mas epa! Achei um cara (J) menor que o menor(F)!
>>> > E este é um absurdo, que surgiu quando eu disse que funcionava para
>>> > algum cara maior que 6!
>>> > Então, só faltaria testar para caras menores que 7. Você já testou,
>>> > então sabe que não funciona!
>>> >
>>> > É isso.
>>> >
>>> > P.S.: otruque de multiplicar por 2 facilita a vida pacas, mas não
>>> > precisava aplica-lo: a ideia do descenso infinito ainda daria conta.
>>> >
>>> >
>>> > Em 27/05/11, marcone augusto araújo
>>> > borges escreveu:
>>> >>
>>> >>
>>> >>
>>> >>
>>> >>
>>> >> From: marconeborge...@hotmail.com
>>> >> To: obm-l@mat.puc-rio.br
>>> >> Subject: Teoria dos números
>>> >> Date: Fri, 27 May 2011 12:28:34 +
>>> >>
>>> >>
>>> >>
>>> >>
>>> >>  1) Mostrar que para nenhum número natural n ,( 2^n)+1 nunca é
>>> um
>>> >> cubo.
>>> >>
>>> >>  Pensei:2^n=x^3-1=(x-1)(x^2+x+1).Se eu conseguisse mostrar q
>>> >> mdc((x-1,x^2+x+1)=1 e que x-1 e
>>> >>x^2+x+1 não podem ser cubos ao mesmo tempo,acredito q
>>> >> resolveria
>>> a
>>> >> questão.
>>> >>  Tentei outras formas também ,mas não consegui.
>>> >>
>>> >>  2) Provar q não exiiste número natural n tal q 7 divide
>>> 4n^2-3.
>>> >>
>>> >>   Considerei n= 7k+ 1 ou 7k-1 ou 7k+2 ou 7k-2 ou 7k+3 ou 7k-3
>>> >> e
>>> >> verifiquei q 4n^2-3 não é múltiplo de 7.
>>> >>   Sei q há outras formas(e talvez mais interessantes).
>>> >>
>>> >
>>> >
>>> > --
>>> > /**/
>>> > 神が祝福
>>> >
>>> > Torres
>>> >
>>>
>>>
>>> --
>>> /**/
>>> 神が祝福
>>>
>>> Torres
>>>
>>> =
>>> Instru�ões para entrar na lista, sair da lista e usar a lista em
>>> http://www.mat.puc-rio.br/~obmlistas/obm-l.html
>>> =
>>>
>>
>>
>


-- 
/**/
神が祝福

Torres

=
Instru��es para entrar na lista, sair da lista e usar a lista em
http://www.mat.puc-rio.br/~obmlistas/obm-l.html
=


[obm-l] Re: [obm-l] FW: Teoria dos números

2011-05-27 Por tôpico Johann Dirichlet
Poxa! O Ralph destruiu minha mensagem! Mas acabei respondendo do mesmo
jeito (ou nao!:))

Em 27/05/11, Johann Dirichlet escreveu:
> Ce já estudou congruencias? Um bom começo é pegar a Eureka! 2 na
> página da OBM, www.obm.org.br (ou comprar da OBM! É baratinho, uma
> anuidade de uns 30 reais e uns 4 contos por cada atrasado que quiser).
> Anyway, vou tentar deixar fácil...
>
> 1)
> 2^n=(x-1)(x^2+x+1)
>
> Vamos tentar calcular o MDC:
> d|x-1
> d|x^2+x+1
>
> x =1 (mod d)
> x^2+x+1=0 (mod d)
>
> primeira na segunda, d|3. Como d=3 é impossível (potencias de 2 nao
> tem fatores 3 :) ), d=1.
>
> Em especial, x-1=1 ou x^2+x+1=1 (ambos sao potencias de 2, e o MDC é
> 1, logo um deles é 1).
> Ou seja, x=2 ou 0. Substitui e chora!
>
> 2)
> 7|4n^2-3
> Multiplica por 2
> 7|8n^2-6=n^2+1+(7n^2-7)
>
> 7|n^2+1
> Por congruências, é possível provar que basta testar n de 0 a 6.Mas
> vou usar descenso infinito.
>
> Teste de 0 a 6 (larga a mão de ser preguiçoso!). Vai falhar (eu acho :) ).
>
> Se funcionar para algum cara maior que 6, seja F o menor dos caras
> para os quais funciona (se existe, existe o menor, este é o lema da
> boa ordem).
>
> Seja J=F-7. Então J é maior ou igual a 0.
> 7|(J+7)^2+1=J^2+2*7*J+7^2+1=7*(um termo chato que não interessa)+J^2+1
> 7|J^2+1
>
> Mas epa! Achei um cara (J) menor que o menor(F)!
> E este é um absurdo, que surgiu quando eu disse que funcionava para
> algum cara maior que 6!
> Então, só faltaria testar para caras menores que 7. Você já testou,
> então sabe que não funciona!
>
> É isso.
>
> P.S.: otruque de multiplicar por 2 facilita a vida pacas, mas não
> precisava aplica-lo: a ideia do descenso infinito ainda daria conta.
>
>
> Em 27/05/11, marcone augusto araújo
> borges escreveu:
>>
>>
>>
>>
>>
>> From: marconeborge...@hotmail.com
>> To: obm-l@mat.puc-rio.br
>> Subject: Teoria dos números
>> Date: Fri, 27 May 2011 12:28:34 +
>>
>>
>>
>>
>>  1) Mostrar que para nenhum número natural n ,( 2^n)+1 nunca é um
>> cubo.
>>
>>  Pensei:2^n=x^3-1=(x-1)(x^2+x+1).Se eu conseguisse mostrar q
>> mdc((x-1,x^2+x+1)=1 e que x-1 e
>>x^2+x+1 não podem ser cubos ao mesmo tempo,acredito q resolveria a
>> questão.
>>  Tentei outras formas também ,mas não consegui.
>>
>>  2) Provar q não exiiste número natural n tal q 7 divide 4n^2-3.
>>
>>   Considerei n= 7k+ 1 ou 7k-1 ou 7k+2 ou 7k-2 ou 7k+3 ou 7k-3 e
>> verifiquei q 4n^2-3 não é múltiplo de 7.
>>   Sei q há outras formas(e talvez mais interessantes).
>>  
>
>
> --
> /**/
> 神が祝福
>
> Torres
>


-- 
/**/
神が祝福

Torres

=
Instru��es para entrar na lista, sair da lista e usar a lista em
http://www.mat.puc-rio.br/~obmlistas/obm-l.html
=


[obm-l] Re: [obm-l] FW: Teoria dos números

2011-05-27 Por tôpico Johann Dirichlet
Ce já estudou congruencias? Um bom começo é pegar a Eureka! 2 na
página da OBM, www.obm.org.br (ou comprar da OBM! É baratinho, uma
anuidade de uns 30 reais e uns 4 contos por cada atrasado que quiser).
Anyway, vou tentar deixar fácil...

1)
2^n=(x-1)(x^2+x+1)

Vamos tentar calcular o MDC:
d|x-1
d|x^2+x+1

x =1 (mod d)
x^2+x+1=0 (mod d)

primeira na segunda, d|3. Como d=3 é impossível (potencias de 2 nao
tem fatores 3 :) ), d=1.

Em especial, x-1=1 ou x^2+x+1=1 (ambos sao potencias de 2, e o MDC é
1, logo um deles é 1).
Ou seja, x=2 ou 0. Substitui e chora!

2)
7|4n^2-3
Multiplica por 2
7|8n^2-6=n^2+1+(7n^2-7)

7|n^2+1
Por congruências, é possível provar que basta testar n de 0 a 6.Mas
vou usar descenso infinito.

Teste de 0 a 6 (larga a mão de ser preguiçoso!). Vai falhar (eu acho :) ).

Se funcionar para algum cara maior que 6, seja F o menor dos caras
para os quais funciona (se existe, existe o menor, este é o lema da
boa ordem).

Seja J=F-7. Então J é maior ou igual a 0.
7|(J+7)^2+1=J^2+2*7*J+7^2+1=7*(um termo chato que não interessa)+J^2+1
7|J^2+1

Mas epa! Achei um cara (J) menor que o menor(F)!
E este é um absurdo, que surgiu quando eu disse que funcionava para
algum cara maior que 6!
Então, só faltaria testar para caras menores que 7. Você já testou,
então sabe que não funciona!

É isso.

P.S.: otruque de multiplicar por 2 facilita a vida pacas, mas não
precisava aplica-lo: a ideia do descenso infinito ainda daria conta.


Em 27/05/11, marcone augusto araújo
borges escreveu:
>
>
>
>
>
> From: marconeborge...@hotmail.com
> To: obm-l@mat.puc-rio.br
> Subject: Teoria dos números
> Date: Fri, 27 May 2011 12:28:34 +
>
>
>
>
>  1) Mostrar que para nenhum número natural n ,( 2^n)+1 nunca é um
> cubo.
>
>  Pensei:2^n=x^3-1=(x-1)(x^2+x+1).Se eu conseguisse mostrar q
> mdc((x-1,x^2+x+1)=1 e que x-1 e
>x^2+x+1 não podem ser cubos ao mesmo tempo,acredito q resolveria a
> questão.
>  Tentei outras formas também ,mas não consegui.
>
>  2) Provar q não exiiste número natural n tal q 7 divide 4n^2-3.
>
>   Considerei n= 7k+ 1 ou 7k-1 ou 7k+2 ou 7k-2 ou 7k+3 ou 7k-3 e
> verifiquei q 4n^2-3 não é múltiplo de 7.
>   Sei q há outras formas(e talvez mais interessantes).
>   


-- 
/**/
神が祝福

Torres

=
Instru��es para entrar na lista, sair da lista e usar a lista em
http://www.mat.puc-rio.br/~obmlistas/obm-l.html
=


Re: [obm-l] problem numero 15 eureka numero 5

2011-05-17 Por tôpico Johann Dirichlet
1 - Enunciado completo,please!

Vou tentar reescrever para deixar mais claro:

"Em um conjunto de MN+1 inteiros positivos, postos em ordem crescente,
uma das duas situações abaixo ocorrerá:

-- haverá uma subsequencia de M+1 inteiros, tais nenhum deles é
divisor de algum outro;
--haverá uma subsequencia de N+1 inteiros, tal que cada termo da
subsequencia será divisor do seguinte;
"

Um exemplo:
Em um conjunto de 16=3*5+1 elementos,
ou há 3+1=4 inteiros tais que nenhum divide outros,
ou há 5+1=6 tais que cada um divide o seguinte.


Acho que você pensou algo como "5 divide 5+1", o que é obviamente falso ;-P

Em 16/05/11, maurikleber araujo escreveu:
>
> pessoal alguem ai poderia explicar o problema numero 15 (principio das
> gavetas) da eureka numero 5
> onde ele diz n +1 numeros divide o seguinte ele quer dizer que divide o
> sucessor ou qualquer numero depois
> devo estar entendendo errado porque se for divide o sucessor nao tem como
> provar ou ao menos penso ter achado um contra exemplo
> alguem pode explicar sem resolver a questao   


-- 
/**/
神が祝福

Torres

=
Instru��es para entrar na lista, sair da lista e usar a lista em
http://www.mat.puc-rio.br/~obmlistas/obm-l.html
=


[obm-l] Re: [obm-l] Re: [obm-l] número primo e soma de quadrados

2011-05-17 Por tôpico Johann Dirichlet
Você encontrará umas três demonstrações bem legais no livro Proofs
from THE BOOK, Martin Aigner e Günter M. Ziegler.



Em 16/05/11, Tiago escreveu:
> Existem diversas maneiras de demonstrar isso. Algumas delas usando ideias e
> áreas da matemática bem diferentes.
>
> http://en.wikipedia.org/wiki/Proofs_of_Fermat%27s_theorem_on_sums_of_two_squares
>
> 2011/5/16 marcone augusto araújo borges 
>
>>  Todo número primo da forma 4k+1pode ser escrito de uma única maneira como
>> soma de dois quadrados.Como demonstrar?
>>
>
>
>
> --
> Tiago J. Fonseca
> http://legauss.blogspot.com
>


-- 
/**/
神が祝福

Torres

=
Instru��es para entrar na lista, sair da lista e usar a lista em
http://www.mat.puc-rio.br/~obmlistas/obm-l.html
=


Re: [obm-l] Geometria Cone Sul

2011-05-13 Por tôpico Johann Dirichlet
Em 12/05/11, Luís Lopes escreveu:
>
> Sauda,c~oes,
>
> Fonte: Treinamento Cone Sul Volume 2.
>
> Problema 26 p. 135
>
> H_b , H_c pés das alturas de B e C.
> H ortocentro
> M_a médio de BC
> Gamma Circuncírculo de ABC
> phi Circuncírculo de AH_bH_c
> S segunda interseção de phi com Gamma
>
> Mostre que S, H, M_a são colineares

Opa! É Geometria Projetiva na cabeça!

Pegue em mãos sua Eureka! 8, dê uma estudada no artigo do Luciano, e
mãos à obra!
Para melhor acompanhar a demonstração, pegue um papel e faça um
desenho caprichado!

Vamos antes tomar um leminha:

Triângulo ABC;
Alturas AH_a, BH_b, CH_c; ortocentro H
Médios M_a,M_b,M_c;
X_a é o ponto comum a BC e H_bH_c

Demonstrar que M_aH e AX_a são perpendiculares!

Vamlá:
Tome o circulo de centro M_a e raio BM_a, chame-o Epsilon. Como BH_bC
e CH_cB são angulos retos, esta circunferencia passa pelos pontos H_b
e H_c.
Vamos dualizar em Epsilon agora.

A reta polar de A é HX_a
A reta polar de X_a é AH.

Portanto, o ponto polar (polo) de AX_a é H.
Logo, M_aH é perpendicular a AX_a.

Seja Y o ponto comum a AX_a e M_aH.
Pelo lema, AYH=90 graus.

Agora, potência de ponto!

AH_bHH_c é cíclico (angulos retos);
CH_bH_cB é cíclico (angulos retos);
ABC é cíclico (triangulo).

X_a é centro radical dos tres circulos acima descritos (é meio óbvio,
mas basta calcular as potencias de ponto em relação a cada par de
circulos).

Seja Z o ponto em que AX_a corta o circulo AH_bHH_c. Por potencia de
ponto, A,Z,B,C são concíclicos, bem como A, Z, H_c, H.
Logo angulo AZH = angulo AH_cH=90 graus = angulo AYH. Portanto, Y=Z, e
tá demonstrado!

>
> Como fazer? Com geometria sintética de preferência.

Pra variar um pouco, não vou fazer com trigonometria. Mas a ideia é simples:
Demonstrar que M_aH e AX_a são perpendiculares!

>
> []'s
> Luís
>
>   


-- 
/**/
神が祝福

Torres

=
Instru��es para entrar na lista, sair da lista e usar a lista em
http://www.mat.puc-rio.br/~obmlistas/obm-l.html
=


Re: [obm-l] Teorema sobre mediana (OFFTOPIC)

2011-05-13 Por tôpico Johann Dirichlet
Em 13/05/11, Ralph Teixeira escreveu:
> Isso é legal, né?
>
> -- A média minimiza a soma dos quadrados dos desvios
> -- A mediana minimiza a soma dos módulos dos desvios.
>
> Olhando deste jeito, a mediana parece mais "natural" do que média para
> resumir os dados de uma sequência alíás, vocês já pararam para
> pensar PORQUE a gente usa a média o tempo todo? No fundo no fundo, é
> só por costume, não há uma razão matemática muito forte não... vou
> exagerar um pouco: se eu pudesse, usava a mediana para calcular as
> notas finais dos meus alunos.
>

A ideia é que a média é um balanço entre perdas e ganhos.
Por exemplo, a média entre 5 e 7 é 6, e 6 perde 1 de 7 mas ganha 1 de 5.
O problema é que a média é muito sensível a variações dos dados.

Antes de eu fugir da facul, já tive professores que usavam média
harmônica para as provas!
Zerou uma prova, já era! Ou tão pior quanto, ao se tirar vários 10, um
5 te jogava pra baixo.

Mas enfim, acho que a média é mais usada porque embute a ideia que eu
exibi acima.

-- 
/**/
神が祝福

Torres

=
Instru��es para entrar na lista, sair da lista e usar a lista em
http://www.mat.puc-rio.br/~obmlistas/obm-l.html
=


Re: [obm-l] produto interno

2011-05-11 Por tôpico Johann Dirichlet
Em 06/05/11, Samuel Wainer escreveu:
>
> qual a diferença entre produto hermetiano e produto interno?sempre ouvi
> falar em operador hermetiano, não em produto hermetiano. Eles são a mesma
> coisa?
>
> Para toda matriz simétrica A, existe uma matriz invertível P tq: A = (P^-1)
> D (P) onde D é diagonal.?Usa isso num teorema que estou lendo, mas é fato?

Bem, é um fato demonstrável; é fácil achar em bons livros de álgebra linear.

Normalmente ele vem depois de uma série de teoremas. Mas o que você
precisa saber para pesquisar é o seguinte:

0 - Duas matrizes A,B são ditas semelhantes se A=P*B*(P^-1) em que P é
uma matriz conveniente.

1 - Uma matriz quadrada A é dita diagonalizável se é semelhante a uma
matriz diagonal.

Teorema: toda matriz simétrica é diagonalizável!

Entre o 1 e o teorema, se costuma apelar para umas teorias fáceis de
aprender e um pouco difíceis de testar ;-P

Topa uma leitura online?
http://joshua.smcvt.edu/linearalgebra/

100% full, disponível para download.

E um review
http://www.randomhacks.net/articles/2007/03/07/hefferon-linear-algebra-review

-- 
/**/
神が祝福

Torres

=
Instru��es para entrar na lista, sair da lista e usar a lista em
http://www.mat.puc-rio.br/~obmlistas/obm-l.html
=


Re: [obm-l] Descobrir formula geral e provar f(n+1)=2f(n) +3

2011-05-09 Por tôpico Johann Dirichlet
Outra maneira:
f(0)=0
f(n+1)=2f(n) +3

Vendo que f(n+1) é quase o dobro de f(n), uma ideia seria obter uma PG.

f(n+1)+C=2f(n) +3+C= 2(f(n)+(C+3)/2)
Se C=(C+3)/2, ou C=3, obtemos uma relacao interessante:

f(n+1)+3=2(f(n)+3).
E isto é uma PG!
O resto segue acima.

Em 06/05/11, Julio Teixeira escreveu:
> Pessoal, a um tempo acho que vi essa questao aki  e por acaso, ontem me
> deparei com ela em alguns foruns, e o pessoal estava com dificuldades..entao
> vou por aki a minha resolucao..
>
> questao 157 do Vol. 1 da colecao do G. Iezzi - Fundamentos de matematica
> elemtentar
>
> 157 - Seja f uma funcao, definida no conjunto dos numeros naturais, tal que,
> f(n+1)=2f(n) +3
> com f(0) = 0. Achar a formula geral de f(n) e prova-la por inducao..
>
> equacao: f(n+1)=2f(n) + 3  e f(0)=0
>
> para..
> n=0 => f(0+1)=2f(0)+3 => f(1)=3
> n=1 => f(1+1)=2f(1)+3 => f(2)=9
> n=2 => f(2+1)=2f(2)+3 => f(3)=21
> n=3 => f(3+1)=2f(3)+3 => f(4)=45
> n=4 => f(4+1)=2f(4)+3 => f(5)=93
>
> observando os valores retornado pelas imagens e pondo em produto de um fator
> por 3..
> f(1)=3  => f(1)=3*1
> f(2)=9  => f(2)=3*3
> f(3)=21 => f(3)=3*7
> f(4)=45 => f(4)=3*15
> f(5)=93 => f(5)=3*31
>
> agora observando os segundos fatores dos produtos acima nas imagens...
> comecamos com 1, depois 3, depois 7, e
>
> assim temos:
> a diferenca entre  3 e 1 = 2
> a diferenca entre  7 e 3 = 4
> a diferenca entre 15 e 7 = 8
> a diferenca entre 31 e 15 = 16
>
> obrservando essas diferencas, nota-se que temos uma PG, de razao 2, e com o
> primeiro termo sendo igual a 1
>
> assim a formula ja comeca a ficar evidente.. sendo 3 vezes essas
> diferencas...
>
> agora se montarmos essa PG, teremos..
>
> a1 = 1
> a2 = 2
> a3 = 4
> a4 = 8
> a5 = 16
>
> opa.. entao a proxima observacao a ser feita eh que, com os resultados
> obtidos temos que,por exemplo,
> f(1)=3*( a1 de nossa PG)
> f(2)=3*( a soma de a1 com o a2 de nossa PG)
> f(3)=3*( a soma de a1 com o a2  e a3 de nossa PG)
> f(4)=3*( a soma de a1 com o a2  e a3 e a4 de nossa PG)
>
> agora a formula do somatorio de nossa PG seria:
> Sn = a1 * (q^n - 1)/ (q - 1)
>
> onde substituindo, obteriamos:
> 2^n -1
>
> agora deduzimos entao que a formula geral seria: f(n)= 3 * ( 2^n - 1)
>
> para provarmos por inducao, vamos provar que eh valido para n=1
> f(1) = 3 * ( 2^1 -1)
> f(1) = 3 * ( 1 ) => f(1) = 3   ( OK, provamos para n=1 )
>
> agora substituimos por n, por um k, qualquer e obtemos:
> f(k)= 3 * (2^k -1)
>
> agora substituimos por k+1
> f(k+1)= 3 * (2^(k+1) -1)
>
> ok, agora note que se pegarmos a formula inicial e aplicarmos n=k, obteremos
> o seguinte..
> f(k+1)=2 * f(k) + 3
>
> ja que obtemos f(k+1) de nossa formula e f(k+1) da formula original, para
> provarmos que descobrimos a formula geral
> entao o resultado de f(k+1), tem que ser igual, assim tb testamos se eh
> valida para qualquer elemento, provando isso para qualquer sucessor de k, ou
> seja (k+1)
> entao temos o seguinte..
> f(k)= 3 * (2^k -1)
> f(k+1)= 3 * (2^(k+1) -1)
> f(k+1)=2 * f(k) + 3
>
> agora igualando os f(k+1), obtemos..
> 2 * f(k) + 3 = 3 * (2^(k+1) -1)
> substituindo f(k), pelo valor conhecido tb.. ( da nossa formula geral )
> 2 * (3 * (2^k -1)) + 3 = 3 * (2^(k+1) -1)
> 6 * (2^k -1) + 3 = 3 * (2^(k+1)) -3
> agora, dividimos amobs os lados por 3
> 2 * (2^k -1) + 1 = 2^(k+1) - 1
> 2^(k+1) -2 + 1 = 2^(k+1) - 1
> 2^(k+1) - 1 = 2^(k+1) - 1(OK)
> obtemos assim, a nossa prova...
>


-- 
/**/
神が祝福

Torres

=
Instru��es para entrar na lista, sair da lista e usar a lista em
http://www.mat.puc-rio.br/~obmlistas/obm-l.html
=


[obm-l] Re: [obm-l] Re: [obm-l] Olimpíada Universitária.

2011-05-06 Por tôpico Johann Dirichlet
Bem, a dica de estudo é a mesma. Eu sugiro que você pegue as
olimpiadas internacionais também (a IMC é muito legal! e serve bem pra
estudar a OBM universitária).

Na verdade, acho que nem mesmo restrição de graduação deve ter. Lembro
de um aluno que tinha um ano a mais de graduação (por ter quebrado a
grade em porções mais digeríveis) e pôde participar. De todo modo, a
Nelly deve saber :-).

Em 25/04/11, Tiago escreveu:
> Olá, acho que não tem limite de idade, só de anos de graduação.
>
> Nas olimpíadas universitárias, a teoria que você tem que saber do ensino
> superior são basicamente Cálculo I e Álgebra linear. Mas o melhor jeito de
> estudar, imagino eu, é pegando as provas anteriores (quem sabe as dos níveis
> mais baixos também) e tentar entender as soluções (obviamente, tentando
> trabalhar nos problemas sozinho antes).
>
> Então o meu conselho é este, não tente seguir uma bibliografia, tente
> trabalhar nos problemas e, assim que certas assuntos surgirem e você
> perceber que não sabe muito bem a teoria, estude esse assunto.
>
> 2011/4/25 Luís Junior 
>
>> Olá a todos,
>>
>> Gostaria de ouvir a opnião de vcs com relação a esse meu sonho/projeto.
>> Sempre gostei de matemática mas frequentemente, na minha vida, um grande
>> esforço se fez necessário para que eu alcançasse a média escolar. De fato,
>> posso afirmar que sou um aluno abaixo da média e que 'rala' bastante para
>> ser mediano. Ontem, tomei conhecimento das Olimpíadas Universitárias.
>> Sempre
>> tive esse sonho, de me preparar e participar de uma dessas Olimpíadas.
>> Pois
>> bem, tenho 30 anos e estou no primeiro semestre de um curso universitário
>> regular. Procurei pelo regulamento para saber se há um limite de idade
>> para
>> os participantes mas não encontrei, então esta se torna a minha primeira
>> dúvida. Sendo possível a minha participação, então se iniciaria um projeto
>> de 5 anos (tempo médio da graduação) que contemplaria a minha preparação e
>> participação no evento. Neste ponto, gostaria de saber a opnião de vcs
>> sobre
>> a possibilidade/dificuldade de empreender um projeto desses e como começar
>> (Revisando o conteúdo de 2° grau?, seguindo uma bibliografia específica?,
>> contratando um mestre?) visto que não tenho a mínima idéia.
>> Agradeço pela atenção e peço desculpas pelo incômodo. Por favor participem
>> com sua opnião!
>>
>>
>> ~Carpe Diem~
>>
>> L.
>>
>
>
>
> --
> Tiago J. Fonseca
> http://legauss.blogspot.com
>


-- 
/**/
神が祝福

Torres

=
Instru��es para entrar na lista, sair da lista e usar a lista em
http://www.mat.puc-rio.br/~obmlistas/obm-l.html
=


Re: [obm-l] Geometria

2011-05-05 Por tôpico Johann Dirichlet
[cuidado! resposta longa e chata detected!!]

Cara, esse tipo de problema eu sempre fiz do mesmo jeito:
trigonometria até enjoar!
Eu sempre preferi desta maneira, pois pra mim usar álgebra é mais
rápido que usar "magia". Nem sempre estes truques são reaplicáveis, e
minha mente computeira se acostuma melhor a coisas que podem ser
guardadas para o futuro.

Mas, antes que falem mal, em hipótese nenhuma estou dizendo que
trigonometria é melhor, mas sim que eu tenho esta preferência por
resolver assim. E também concordo que geometria sintética é muito boa,
e que trigonometria não é superpoderosa (que o diga Erdos-Mordell!)

Como exemplo do que estou falando:
Tem um problema da Inglaterra, 1970, do mesmo estilo destes. Eu
resolvi bem rápido com trigonometria, mas tive que pedir ajuda ao
Naoki Sato, do MathLinks (na verdade ele foi o primeiro a responder),
para obter uma solução sintética (usando somente geometria e
construções auxiliares). Para todos os efeitos, ele reconstrói o
desenho usando uma sequência de lemas, e depois prova que este desenho
construído é igual ao do enunciado.

Vejam em
http://www.artofproblemsolving.com/Forum/viewtopic.php?f=47&t=329268
e depois tirem suas conclusões...

P.S.: depois eu vou resolver este com trigonometria, haha!

Em 03/05/11, Marcelo Gomes escreveu:
> Olá professor Albert,
>
> por favor, se for possível, poderia enviar sua solução também para mim ?
>
> Abraços, Marcelo.
>
> Em 28 de abril de 2011 18:04, Albert Bouskela  escreveu:
>
>> Olá, Nehab e João,
>>
>> O trabalho da Silvana é mesmo bem legal, mas...
>>
>> Para resolver o problema proposto - o Nehab tem razão: é um dos mais
>> clássicos - prefiro fazer um truque mais palatável: construir triângulos
>> auxiliares. Estou enviando - através de um arquivo PDF - a solução para o
>> e-mail de vocês.
>>
>> Sds.,
>> Albert Bouskela
>> bousk...@msn.com
>>
>> > -Mensagem original-
>> > De: owner-ob...@mat.puc-rio.br [mailto:owner-ob...@mat.puc-rio.br] Em
>> > nome de Carlos Nehab
>> > Enviada em: 28 de abril de 2011 17:30
>> > Para: obm-l@mat.puc-rio.br
>> > Assunto: Re: [obm-l] Geometria
>> >
>> > Oi, João,
>> >
>> > O seu exercício é um clássico.
>> > Ai vai a dica. Um trabalho legal da Silvana: você vai gostar.
>> >
>> > http://www.mat.puc-rio.br/~hjbortol/complexidade/complexidade-em-
>> > geometria.pdf
>> >
>> > Capítulo 2 a partir da página 28
>> > Olhe também a página 36.
>> >
>> > Abraços,
>> > Nehab
>> >
>> > Em 26/4/2011 20:22, João Maldonado escreveu:
>> > > O seguinte problema está no livro  Geometria I de Morgado, e não sei
>> > porque  não estou conseguindo resolvê-lo. Sei que a resposta é 30º, se
>> > alguém  puder ajudar fico grato.
>> > >
>> > > Em um triângulo isósceles ABC, se base BC, o ângulo  vale 20º. P é um
>> > ponto sobre AB tal que o ângulo PCB = 60º. Q é um ponto em AC tal que
>> > QBC
>> > = 50º. Qual a medida do ângulo CPQ?
>> > > []'sJoão
>> >
>> > =
>> > 
>> > Instruções para entrar na lista, sair da lista e usar a lista em
>> > http://www.mat.puc-rio.br/~obmlistas/obm-l.html
>> > =
>> > 
>>
>>
>> =
>> Instruções para entrar na lista, sair da lista e usar a lista em
>> http://www.mat.puc-rio.br/~obmlistas/obm-l.html
>> =
>>
>


-- 
/**/
神が祝福

Torres

=
Instru��es para entrar na lista, sair da lista e usar a lista em
http://www.mat.puc-rio.br/~obmlistas/obm-l.html
=


[obm-l] Re: [obm-l] Geometrias não Euclidianas - Como realizar as construções geométricas ?

2011-04-11 Por tôpico Johann Dirichlet
Bem, creio que não tenha muita lógica em falar de "construçoes
euclidianas" em espaços não-euclidianos. Mas, levando em conta os
postulados, dá pra brincar um pouco (só não espere algo com muito
sentido :) )

De todo modo, achei este site via Google:
http://cs.unm.edu/~joel/NonEuclid/NonEuclid.html

Em 09/04/11, Marcelo Gomes escreveu:
> Caros amigos da lista,
>
> Estou impressionado coma  falta de material sobre as geometrias elíptica e
> hiperbólica. Estou tentando reproduzir construções geométricas que envolvam
> estas geometrias em softwares como o Geogebra e ReC.
>
> Gostaria de perguntar aos senhores se conhecem materiais que demonstrem como
> operar as construções (com régua e compasso) destas geometrias. Se alguém
> souber e quiser compartilhar agradeço muito.
>
> Abraços, Marcelo.
>


-- 
/**/
神が祝福

Torres

=
Instru��es para entrar na lista, sair da lista e usar a lista em
http://www.mat.puc-rio.br/~obmlistas/obm-l.html
=


[obm-l] Re: [obm-l] Re: [obm-l] Nº de funções sobrejetoras

2011-03-29 Por tôpico Johann Dirichlet
Cara, algo me diz que isso tá errado. Eu lembro de um artigo na antiga
RPM que contava, de um modo meio complicado, quantas funções existem.
Por exemplo, nesta fórmula (um somatório esquisito usando números de
Stirling), se |A| < |B|, tinha que dar 0.

Se pensarmos de B para A, cada elemento de B terá uma flechinha OU
MAIS chegando de elementos de A.

Posso estar errado, mas não dou certeza...

Em 28/03/11, Marcelo Salhab Brogliato escreveu:
> Olá, Pedro,
> para cada elemento de B, temos que ter pelo menos um elemento de A que leve
> a ele.
> Logo, para o primeiro elemento de B, temos n opções.
> Para o segundo elemento de B, temos n-1 opções.
> E assim por diante.
>
> Assim, ficamos com:
> n*(n-1)*(n-2)*...*(n-m+1) = n! / (n-m)! = Arranjo(n, m) = A(n, m)
>
> Mas, ainda sobram n-m elementos em A.
> Para esses n-m elementos, tanto faz em qual elemento de B eles levam.
> Logo, para cada um desses n-m elementos, temos m opções.
> Ficando com: m^(n-m)
>
> Portanto, a resposta fica: A(n, m) * m^(n-m) = m^(n-m) * n! / (n-m)!
>
> Abraços,
> Salhab
>
>
> 2011/3/26 Pedro Júnior 
>
>> Um colega me propôs o seguinte problema, e não consegui modelar:
>> Seja A um conjunto com n elementos e seja B um conjunto com m elementos,
>> com n >= m. Quantas funções sobrejetoras, f : A --> B, podemos formar?
>>
>> --
>>
>> Pedro Jerônimo S. de O. Júnior
>>
>> Professor de Matemática
>>
>> Geo João Pessoa – PB
>>
>>
>


-- 
/**/
神が祝福

Torres

=
Instru��es para entrar na lista, sair da lista e usar a lista em
http://www.mat.puc-rio.br/~obmlistas/obm-l.html
=


[obm-l] Re: [obm-l] Questão de teoria dos números

2011-03-26 Por tôpico Johann Dirichlet
Isto parece óbvio: a parte inteira de uma fração é justamente oquociente na 
divisão euclidiana clássica. Logo, se aumentamos odivisor, o quociente 
naturalmente diminui.
Talvez a parte difícil seja usar álgebra nisso aí...
Em 26/03/11, ennius escreveu:> Caros Colegas,>> Como podemos 
provar que a parte inteira de n/(2^k) é maior que a parte> inteira de 
n/(5^k),> para todo inteiro n>1 e n maior ou igual a 2^k?  (k é inteiro 
positivo.)>> Abraços do Ennius.> 
=> 
Instruções para entrar na lista, sair da lista e usar a lista em> 
http://www.mat.puc-rio.br/~obmlistas/obm-l.html> 
=>

-- /**/神が祝福
Torres
=
Instru��es para entrar na lista, sair da lista e usar a lista em
http://www.mat.puc-rio.br/~obmlistas/obm-l.html
=


Re: [obm-l] Prova da OBM-nivel U (problema 6)

2011-03-26 Por tôpico Johann Dirichlet
Concordo plenamente, apesar de eu não ter conseguido fazer sozinho.

Depois de uma grata ajuda do Lopes, foi fácil:
1 - Se n é ímpar, a expressão do numerador é uma soma de quadrados,
logo p teria que ser da forma 4k+1
2 - Se n é par, o seu rtaciocínio prova que k é par.

A próxima ideia que tive era saber se, para todo n, existe um k, e vice-versa.

Em 26/03/11, charles<9char...@gmail.com> escreveu:
> Agora que papirei reciprocidade quadrática sei fazer :
>
> 10^2n + 8*10^n + 1 = (10^n + 1)^2 + 6*10^n = 0 (60*k + 7)->
> (-6*10^n / 60*k + 7) = 1  (Símbolo de Legendre) .
>
> Agora é só aplicar as propriedades fundamentais ( tentem fazer! ) :
>
> (a*b / p) = (a/p) * (b/p)   (p primo ímpar)
> (-1 / p) = (-1)^((p-1)/2)
> (2 / p) = (-1)^((p^2 - 1) / 8)
> e o teorema da reciprocidade quadrática.
>
> Depois vc chega em algo do tipo : n+nk+k = 0 (mod 2) -> (n+1)(k+1) = 0 (mod
> 2) ->
> n e k são pares!
>
> É incrivelmente fácil pra um problema 6, vcs não acham?
>


-- 
/**/
神が祝福

Torres

=
Instru��es para entrar na lista, sair da lista e usar a lista em
http://www.mat.puc-rio.br/~obmlistas/obm-l.html
=


Re: [obm-l] raizes

2011-03-24 Por tôpico Johann Dirichlet
Em 21/03/11, marcone augusto araújo
borges escreveu:
>
> Onde encontro a fórmula para achar as raízes de uma equação do quarto grau?
A princípio, conheço dois métodos, e ambos só servem pra dizer que
"existe uma fórmula usando radicais" para as raízes de uma equação de
quarto grau.

O mais legal, EMMO, é o método Gugu-Euler:

Reduza a quártica a uma forma do tipo x^4=p*x^2+q*x+r, e escreva
x=sqrt(y1)+sqrt(y2)+sqrt(y3).

Abrindo as expressões feito um doido, você pode agrupar uns termos e
fazer umas suposições acerca dos y's, e descobrir que os y's são
raízes de uma equação de grau 3.
Aí é só resolver esta coisinha fofa que se origina, e pronto!

Estou sendo bem conciso, mas prometo que alguém poderá explicar em
detalhes depois...

>   


-- 
/**/
神が祝福

Torres

=
Instru��es para entrar na lista, sair da lista e usar a lista em
http://www.mat.puc-rio.br/~obmlistas/obm-l.html
=


[obm-l] Re: [obm-l] Re: [obm-l] RES: [obm-l] nome de Matemático

2011-03-02 Por tôpico Johann Dirichlet
Quem me contou algo semelhante foi o Tengan ou o Humberto Naves: um
problema que foi resolvido por um aluno, porque ele se atrasou. Depois
ele entregou o "trabalho de casa" pro professor, que ficou apavorado!
com a notícia. Resolveu ate publica-los em umas revistas.

O nome e esse mesmo, George Dantzig. O cara virou a maior lenda urbana
da Matematica com isso, e a historia e mais lembrada do que a pessoa.

Outro caso curioso foi o de Erdos:

Ele tinha chegado numa universidade, e perguntou "O que e isso na lousa?"
Era um problema de uma area da qual Erdos nunca ouviu falar. Ademais,
a solução obtida pelos caras daquela faculdade tinha um monte de
folhas escritas.

Erdos fez umas perguntas sobre alguns detalhes que ele não entendia do
problema (algo de terminologia, creio eu), e depois deu uma solução em
poucas linhas.

Tem uma do Eduardo Tengan que eu conto outra hora...


Em 28/02/11, Welma Pereira escreveu:
> Obrigada Albert. Eu não tenho certeza se história é real pois ouvi de um
> colega há uns anos atrás na universidade.
>
> 2011/2/27 Albert Bouskela 
>
>> Olá, Welma,
>>
>>
>>
>> Estudo, com razoável afinco, a história da Matemática e a biografia dos
>> seus expoentes. Contudo, desconheço completamente o fato pitoresco ao qual
>> você se refere.
>>
>>
>>
>> O mais próximo dele, que tenho na memória, é atribuído a Gauss: –
>> Acredita-se que Gauss, quando estava no ensino básico, por castigo imposto
>> pelo seu professor de Aritmética, tenha sido obrigado a somar todos os
>> números compreendidos entre 1 e 100 (incluindo-os).  Gauss, então, deduziu
>> a
>> fórmula da soma dos termos de uma P.A., ao observar que 1+100 = 2+99 =
>> 3+98
>> etc. = soma dos extremos [ a(1)+a(n) ].
>>
>>
>>
>> Atenção:
>>
>> i.Não há comprovação de que esta história (ou fábula)
>> seja verdadeira;
>>
>>   ii.A fórmula da soma dos termos de uma P.A. já era, há
>> muito, conhecida.
>>
>>
>>
>> Albert Bouskela
>>
>> bousk...@msn.com
>>
>>
>>
>> *De:* owner-ob...@mat.puc-rio.br [mailto:owner-ob...@mat.puc-rio.br] *Em
>> nome de *Welma Pereira
>> *Enviada em:* 27 de fevereiro de 2011 13:07
>> *Para:* obm-l@mat.puc-rio.br
>> *Assunto:* [obm-l] nome de Matemático
>>
>>
>>
>> Olá Pessoal,
>>
>>
>>
>> Será que podiam me ajudar? Estou a procura do nome do matemático que
>> resolveu um grande problema porque pensou que era lição de casa?
>>
>>
>>
>> Agradeço
>>
>> Welma
>>
>


-- 
/**/
神が祝福

Torres

=
Instru��es para entrar na lista, sair da lista e usar a lista em
http://www.mat.puc-rio.br/~obmlistas/obm-l.html
=


Re: [obm-l] Prova da OBM-nivel U (problema 6)

2011-01-10 Por tôpico Johann Dirichlet
Aonde eu acho esse cara??


Em 01/01/11, charles<9char...@gmail.com> escreveu:
> O Leandro Farias fez!
>


-- 
/**/
Quadrinista e Taverneiro!

http://tavernadofimdomundo.blogspot.com >> Quadrinhos, histórioas e afins
http://baratoeletrico.blogspot.com />> Um pouco sobre elétrons em movimento
http://bridget-torres.blogspot.com/ >> Personal! Do not edit!

=
Instru��es para entrar na lista, sair da lista e usar a lista em
http://www.mat.puc-rio.br/~obmlistas/obm-l.html
=


[obm-l] Re: [obm-l] Re: [obm-l] RE: [obm-l] Infinitas soluç ões(números inteiros)

2011-01-10 Por tôpico Johann Dirichlet
Eu propus este problema (na verdade uma versao) na Eureka!
Bem, ele ja foi resolvido, no numero 30 se nao me engano.

Em 09/01/11, Ralph Teixeira escreveu:
> Aprendi esta ideia num problema de uma IMO:
>
> -- (1,1,1) eh solucao.
> -- Pense na equacao como uma quadratica em x: x^2-(3yz)x+(y^2+z^2)=0.
> A soma das raizes eh 3yz. Entao, se x=a eh uma solucao, a outra eh
> x=3yz-a.
> -- Em outras palavras, o que mostramos eh que se (x,y,z) eh solucao,
> entao (3yz-x,y,z) tambem eh (o que poderia ser verificado
> diretamente).
> -- Por simetria, (x,3xz-y,z) e (x,y,3xy-z) tambem servem.
> -- Isso gera uma famila de solucoes:
> (1,1,1) -> (3.1.1-1,1,1)=(2,1,1) -> (2,3.1.1-1,1)=(2,5,1) ->
> (2,5,3.2.5-1)=(2,5,29) -> (3.5.29-2,5,29) -> ...->
> -- Note que na construcao eu escolhi sempre trocar o MENOR dos numeros
> (x,y,z). Entao a cada passo a soma da terna passou de x+y+z para
> (3yz-x)+(y+z), onde x eh o menor dos tres numeros. Como claramente
> 3yz-x>x (pois 3yz>=3.x.1>2x), entao a soma eh estritamente crescente,
> e as ternas sao todas diferentes.
>
> Abraco, Ralph.
>
> 2011/1/9 marcone augusto araújo borges :
>> corrigindo: x^2 + y^2 + z^2 = 3xyz
>>
>> 
>> From: marconeborge...@hotmail.com
>> To: obm-l@mat.puc-rio.br
>> Subject: [obm-l] Infinitas soluções(números inteiros)
>> Date: Sun, 9 Jan 2011 02:10:07 +
>>
>> mostre q a equação x^2 + y^2 +z^2 = xyz tem infinitas soluções onde x,y,z
>> são números inteiros.
>> Agradeço a todos q ajudarem.
>>
>
> =
> Instruções para entrar na lista, sair da lista e usar a lista em
> http://www.mat.puc-rio.br/~obmlistas/obm-l.html
> =
>


-- 
/**/
Quadrinista e Taverneiro!

http://tavernadofimdomundo.blogspot.com >> Quadrinhos, histórioas e afins
http://baratoeletrico.blogspot.com />> Um pouco sobre elétrons em movimento
http://bridget-torres.blogspot.com/ >> Personal! Do not edit!

=
Instru��es para entrar na lista, sair da lista e usar a lista em
http://www.mat.puc-rio.br/~obmlistas/obm-l.html
=


[obm-l] Prova da OBM-nivel U (problema 6)

2011-01-01 Por tôpico Johann Dirichlet
Ola povo!
Alguem resolveu o problema 6 da OBMU?
"Se p e um primo da forma 60k+7, e p divide 10^(2n)+8*10^n+1, então n
e k são ambos pares".

Por ora, não estou conseguindo ter nenhuma ideia... Por ora, pensei em
hensel, mas nao testei ainda.

-- 
/**/
Quadrinista e Taverneiro!

http://tavernadofimdomundo.blogspot.com >> Quadrinhos, histórioas e afins
http://baratoeletrico.blogspot.com />> Um pouco sobre elétrons em movimento
http://bridget-torres.blogspot.com/ >> Personal! Do not edit!

=
Instru��es para entrar na lista, sair da lista e usar a lista em
http://www.mat.puc-rio.br/~obmlistas/obm-l.html
=


[obm-l] Re: [obm-l] SEMANA OLÍMPICA

2011-01-01 Por tôpico Johann Dirichlet
Bem, eu direi em carater nao-oficial: acho que ate os medalhistas de
prata sao bancados pela OBM. Ja o bronze, e uma especie de
50-50(passagem, mas nao os dias de hospedagem), e os menção honrosa e
100-0.
De todo modo, se voce foi agraciado com um premio, voce sera melhor informado.

Em 28/12/10, charles<9char...@gmail.com> escreveu:
> Os premiados com medalha do nível universitário recebem medalha como nos
> outros níveis?
> E quanto a semana olímpica, devemos pagar para participar?
>
> Obrigado.
>


-- 
/**/
Quadrinista e Taverneiro!

http://tavernadofimdomundo.blogspot.com >> Quadrinhos, histórioas e afins
http://baratoeletrico.blogspot.com />> Um pouco sobre elétrons em movimento
http://bridget-torres.blogspot.com/ >> Personal! Do not edit!

=
Instru��es para entrar na lista, sair da lista e usar a lista em
http://www.mat.puc-rio.br/~obmlistas/obm-l.html
=


Re: [obm-l] Teoria dos Conjuntos

2011-01-01 Por tôpico Johann Dirichlet
Bem, isto me parece um pouco com um daqueles paradoxos (acho que o de
Russel). Acho que nao e muito facil construir uma coisa dessas.
Antes de mais nada, isto vai incorrer em perguntas do tipo "mas isto é
um axioma da teoria dos conjuntos?". Como eu conheço bem pouco, eu
prefiro dar uma referência: Paul Halmos, Naive Set Theory (Teoria
Ingenua dos Conjuntos),. aliás um livro nada ingênuo :P

P.S.:  Um errinho de portugues: este consertando e com s. Mas acho que
se eu repetir isto provocarei uma flame war...

Em 22/12/10, Vinícius Harlock escreveu:
> Concertando:
>
> Agora eu só quero um exemplo.
> Seja R o conjunto dos números reais, diga-me um elemento que pertença ao
> conjunto A, tal que A={x∈R|x∉x} e outro elemento que pertença a B, tal  B={x
> ∈R|x∈x}. Se por acaso R não servir como conjunto universo, troque-o por um
> que sirva.
>
>
> Agradeço muito a atenção de todos =8^B
> a[b]'s
>


-- 
/**/
Quadrinista e Taverneiro!

http://tavernadofimdomundo.blogspot.com >> Quadrinhos, histórioas e afins
http://baratoeletrico.blogspot.com />> Um pouco sobre elétrons em movimento
http://bridget-torres.blogspot.com/ >> Personal! Do not edit!

=
Instru��es para entrar na lista, sair da lista e usar a lista em
http://www.mat.puc-rio.br/~obmlistas/obm-l.html
=


[obm-l] Re: [obm-l] Re: [obm-l] Demonstrar Frações Parciai s com Álgebra Linear

2010-12-19 Por tôpico Johann Dirichlet
O titulo era simplesmente "O Calculo com Algebra Linear". Nao sei nem
os autores direito... Ele versava sobre Calculo e bem pouco sobre
AlgeLin, A mais marcante aplicação foi justamente esta.

Em 19/12/10, João Luís Gomes Guimarães escreveu:
>
> Olá Johann,
>
> Não se lembra qual era o livro?
>
> JL
>
> -Mensagem Original-
> From: Johann Dirichlet
> Sent: Sunday, December 19, 2010 3:05 PM
> To: obm-l
> Subject: [obm-l] Demonstrar Frações Parciais com Álgebra Linear
>
> Olá pessoas!
>
> Faz algum tempo atrás, eu tinha um livro de Cálculo 1 + Álgebra
> Linear. Entre outras coisas, ele ensinava a calcular integrais de
> funcoes racionais (aquelas que estao ficando famosas na lista:
> integral de (P(x)/Q(x)), em que P e Q são polinômios).
>
> Nisto, ele tinha um apêndice em que demonstrava, usando Álgebra
> Linear, que e possivel quebrar P/Q em fracoes parciais.
>
> Mais precisamente, todos devem conhecer o resultado: se o grau de P e
> menor que o grau de Q, e Q se fatora como (x-r)^m, as fracoes parciais
> tem a forma
> C/(x-r)^(1)+C/(x-r)^(2)+...+C/(x-r)^(m)
>
> Pois bem, eu nao tenho mais o livro :(
> Portanto, eu queria uma demonstração usando Álgebra Linear do fato acima.
> Eu lembro que era algo corriqueiro: demonstrar que as fracoes acima
> formavam um espaco vetorial de dimensao K, e depois achar uma K-base.
> Mas os detalhes me fogem...
>
> Desde já, agradeço!
>
> --
> /**/
> Quadrinista e Taverneiro!
>
> http://tavernadofimdomundo.blogspot.com >> Quadrinhos, histórioas e afins
> http://baratoeletrico.blogspot.com />> Um pouco sobre elétrons em movimento
> http://bridget-torres.blogspot.com/ >> Personal! Do not edit!
>
> =
> Instru��es para entrar na lista, sair da lista e usar a lista em
> http://www.mat.puc-rio.br/~obmlistas/obm-l.html
> =
>
>
> =
> Instru�ões para entrar na lista, sair da lista e usar a lista em
> http://www.mat.puc-rio.br/~obmlistas/obm-l.html
> =
>


-- 
/**/
Quadrinista e Taverneiro!

http://tavernadofimdomundo.blogspot.com >> Quadrinhos, histórioas e afins
http://baratoeletrico.blogspot.com />> Um pouco sobre elétrons em movimento
http://bridget-torres.blogspot.com/ >> Personal! Do not edit!

=
Instru��es para entrar na lista, sair da lista e usar a lista em
http://www.mat.puc-rio.br/~obmlistas/obm-l.html
=


[obm-l] Demonstrar Frações Parciais com Álgebra Linear

2010-12-19 Por tôpico Johann Dirichlet
Olá pessoas!

Faz algum tempo atrás, eu tinha um livro de Cálculo 1 + Álgebra
Linear. Entre outras coisas, ele ensinava a calcular integrais de
funcoes racionais (aquelas que estao ficando famosas na lista:
integral de (P(x)/Q(x)), em que P e Q são polinômios).

Nisto, ele tinha um apêndice em que demonstrava, usando Álgebra
Linear, que e possivel quebrar P/Q em fracoes parciais.

Mais precisamente, todos devem conhecer o resultado: se o grau de P e
menor que o grau de Q, e Q se fatora como (x-r)^m, as fracoes parciais
tem a forma
C/(x-r)^(1)+C/(x-r)^(2)+...+C/(x-r)^(m)

Pois bem, eu nao tenho mais o livro :(
Portanto, eu queria uma demonstração usando Álgebra Linear do fato acima.
Eu lembro que era algo corriqueiro: demonstrar que as fracoes acima
formavam um espaco vetorial de dimensao K, e depois achar uma K-base.
Mas os detalhes me fogem...

Desde já, agradeço!

-- 
/**/
Quadrinista e Taverneiro!

http://tavernadofimdomundo.blogspot.com >> Quadrinhos, histórioas e afins
http://baratoeletrico.blogspot.com />> Um pouco sobre elétrons em movimento
http://bridget-torres.blogspot.com/ >> Personal! Do not edit!

=
Instru��es para entrar na lista, sair da lista e usar a lista em
http://www.mat.puc-rio.br/~obmlistas/obm-l.html
=


[obm-l] Re: [obm-l] Campeões da Matemática - Resultados da 32a. Olimpíada Brasileira de Matemática - OBM

2010-12-19 Por tôpico Johann Dirichlet
faltou o resultado do Torneio das Cidades :)
Apesar de eu nao saber se ele e realizado pela SBM ou OBM, seria bom
ter alguma info.


Em 16/12/10, Olimpiada Brasileira de Matematica escreveu:
>
> Campeões da Matemática - Resultados da 32a. Olimpíada Brasileira de
> Matemática - OBM
>
> Por mais um ano consecutivo estamos finalizando a realização da
> Olimpíada Brasileira de Matemática. Conheça a listagem de premiados da
> OBM -- 2010 no endereço: www.obm.org.br 
>
> A Olimpíada Brasileira de Matemática tem crescido nos últimos anos,
> contando, este ano, com a adesão ao Programa de mais de 4.651 escolas,
> sendo 2.770 da rede pública e 1.881 da rede privada de ensino, o que
> implica em uma participação na Olimpíada Brasileira de Matemática de
> cerca de 350.000 jovens estudantes e seus professores.Além disso, o
> Programa Nacional de Olimpíadas de Matemática conta com a colaboração de
> professores universitários em 155 instituições de ensino superior: eles
> participam de todas as atividades da Olimpíada Brasileira de Matemática,
> inclusive aquelas referentes à OBM Nível Universitário em atividades de
> coordenação, divulgação, treinamento de alunos, aperfeiçoamento de
> professores e aplicação das distintas fases da Olimpíada Brasileira de
> Matemática.
>
> No que se refere à participação em competições internacionais, os
> resultados são excelentes:
>
> * Olimpíada de Matemática do Cone Sul (Águas de São Pedro -- São
>   Paulo): uma medalha de Ouro, duas de Prata e uma de Bronze.
> * Olimpíada Internacional de Matemática -- IMO (Astana --
>   Cazaquistão): duas medalhas de Prata, uma medalha de Bronze.
> * Olimpíada Internacional de Matemática para Estudantes
>   Universitários -- IMC (Blagoevgrad -- Bulgária): uma medalha de
>   Ouro, duas medalhas de Prata, oito medalhas de Bronze e oito
>   Menções Honrosas.
> * Olimpíada Iberoamericana de Matemática (Assunção -- Paraguai.):
>   três medalhas de Ouro e uma medalha de Prata.
> * Competição Interuniversitária Iberoamericana de Matemática -- CIIM
>   (Rio de Janeiro -- RJ): seis medalhas de Ouro (sendo duas de ouro
>   especial), cinco medalhas de Prata e oito medalhas de Bronze.
> * Olimpíada de Maio (Organizada pela Argentina): duas medalhas de
>   Ouro, quatro medalhas de Prata e oito medalhas de Bronze
>   distribuídas entre os dois níveis da competição.
> * Asian Pacific Mathematical Olympiad (APMO): três medalhas de
>   Prata, quatro medalhas de Bronze distribuídas entre os dois níveis
>   da competição.
> * Romanian Máster in Mathematics (RMM) (Bucarest -- Romênia): três
>   medalhas de Bronze e uma menção honrosa.
> * Este ano também realizamos o Concurso Canguru Matemático Sem
>   Fronteiras, concurso que reúne participantes de 42 países.
>
> Durante 2009 a CAPES e o CNPq lançaram o Programa de Iniciação
> Científica -- Mestrado (PICME) para medalhistas da OBMEP e OBM, com o
> objetivo de aumentar o número de matemáticos no país, e oferecer uma
> formação matemática mais sólida a jovens profissionais de outras áreas
> científicas e tecnológicas.
>
> Todos estes resultados nacionais e internacionais demonstram que, além
> de influenciar positivamente o ensino da Matemática nas instituições de
> ensino fundamental, médio e superior, conseguimos detectar jovens muito
> talentosos que são estimulados a seguir uma carreira científica, o que é
> fundamental para o crescimento da Ciência e Tecnologia no país.
>
> A Olimpíada Brasileira de Matemática é um projeto conjunto da Sociedade
> Brasileira de Matemática (SBM), do Instituto Nacional de Matemática Pura
> e Aplicada (IMPA) e conta com o apoio do Conselho Nacional de
> Desenvolvimento Científico e Tecnológico (CNPq) e do Instituto Nacional
> de Ciência e Tecnologia de Matemática (INCT -- Mat).
>
> *Informações:*
>
> --
> Secretaria da Olimpíada Brasileira de Matemática
> Estrada Dona Castorina, 110 Jd. Botânico,
> Rio de Janeiro - RJ, 22460-320, Brasil
> Tel: 55-21-25295077 Fax:55-21-25295023
> e-mail:o...@impa.br
> web site:www.obm.org.br
>
>


-- 
/**/
Quadrinista e Taverneiro!

http://tavernadofimdomundo.blogspot.com >> Quadrinhos, histórioas e afins
http://baratoeletrico.blogspot.com />> Um pouco sobre elétrons em movimento
http://bridget-torres.blogspot.com/ >> Personal! Do not edit!

=
Instru��es para entrar na lista, sair da lista e usar a lista em
http://www.mat.puc-rio.br/~obmlistas/obm-l.html
=


[obm-l] Re: [obm-l] Re: [obm-l] Re: [obm-l] Re: [obm-l] Re: [obm-l] Re: [obm-l] Re: [obm-l] O produto de n inteiros cons ecutivos é múltiplo do fatorial de n

2010-12-13 Por tôpico Johann Dirichlet
Em 09/12/10, Henrique Rennó escreveu:
> Em 09/12/10, Johann Dirichlet escreveu:
>> Bem, respondendo:
>> 1 - Errei: para k=0 o valor é 1
>> 2 - Tem uma especie de dispositivo pratico, que funciona na mesma
>> ideia do triangulo de Pascal:
>>
>> 0 0 0 0 0 ... 0 1
>>  0 0 0 0 ... 0 1
>>   0 0 0 ... 0 1
>>0 0 ... 0 1
>>  0 ... 1
>>
>>   1
>>
>> Este e o triangulo das diferenças de f(n,k).
>> Depois de um numero finito de passos (n+1, se nao me engano) a ultima
>> linha fica constante (neste caso igual a 1).
>> Ai e so reverter...
>>
>> Existe uma formula pronta, mas eu quase nao decoro...
>>
>
> Não entendi a relação desse "triângulo de Pascal" com o polinômio e
> como isso determina que o polinômio é sempre divisível por n! para
> quaisquer valores de n e k.

Esta e uma tecnica comum: se P é um polinomio de grau M então
P(x+1)-P(x) tem grau M-1. Iterando M vezes, obtemos uma constante.

Veja que eu fiz isso no polinomio que era dividido por k!, logo se
este polinomio e sempre inteiro entao o original e multiplo de n!


>
> --
> Henrique
>
> =
> Instruções para entrar na lista, sair da lista e usar a lista em
> http://www.mat.puc-rio.br/~obmlistas/obm-l.html
> =
>


-- 
/**/
Quadrinista e Taverneiro!

http://tavernadofimdomundo.blogspot.com >> Quadrinhos, histórioas e afins
http://baratoeletrico.blogspot.com />> Um pouco sobre elétrons em movimento
http://bridget-torres.blogspot.com/ >> Personal! Do not edit!

=
Instru��es para entrar na lista, sair da lista e usar a lista em
http://www.mat.puc-rio.br/~obmlistas/obm-l.html
=


Re: [obm-l] RESULTADO!!!

2010-12-13 Por tôpico Johann Dirichlet
1 - Pergunte uma vez só!
2 - Não posso precisar, mas costuma ser no finzinho de dezembro (tempo
para arrumas as malas para a semana olimpica!)

2010/12/13, charles <9char...@gmail.com>:
> Quando sai o resultado da OBM?
>


-- 
/**/
Quadrinista e Taverneiro!

http://tavernadofimdomundo.blogspot.com >> Quadrinhos, histórioas e afins
http://baratoeletrico.blogspot.com />> Um pouco sobre elétrons em movimento
http://bridget-torres.blogspot.com/ >> Personal! Do not edit!

=
Instru��es para entrar na lista, sair da lista e usar a lista em
http://www.mat.puc-rio.br/~obmlistas/obm-l.html
=


[obm-l] Re: [obm-l] Re: [obm-l] Re: [obm-l] Re: [obm-l] Re: [obm-l] O produto de n inteiros consecutivos é múltiplo do fatorial de n

2010-12-09 Por tôpico Johann Dirichlet
Bem, respondendo:
1 - Errei: para k=0 o valor é 1
2 - Tem uma especie de dispositivo pratico, que funciona na mesma
ideia do triangulo de Pascal:

0 0 0 0 0 ... 0 1
 0 0 0 0 ... 0 1
  0 0 0 ... 0 1
   0 0 ... 0 1
 0 ... 1

  1

Este e o triangulo das diferenças de f(n,k).
Depois de um numero finito de passos (n+1, se nao me engano) a ultima
linha fica constante (neste caso igual a 1).
Ai e so reverter...

Existe uma formula pronta, mas eu quase nao decoro...

Em 09/12/10, Henrique Rennó escreveu:
> Em 28/11/10, Johann Dirichlet escreveu:
>> Por que este povo tem tanto pavor de "uma prova que não use outros
>> conceitos alem do enunciado"?
>> Eu mesmo conheço vários problemas que são resolvidos usando outras
>> técnicas. Na IMO de Glasgow teve um problema de Teoria dos Números com
>> uma solução que usava polinômios. E tem um monte de problemas de
>> teoria dos números que se resolvem usando técnicas de combinatória (o
>> teorema de Euler-Fermat, por exemplo).
>>
>> De todo modo, só pra não perder o propósito da mensagem:
>>
>> Uma maneira seria observar que f(n,k)=(k+1)(k+2)...(k+n)/n! é um
>> polinômio de grau n em k.
>> Ele é completamnte determinado se eu utilizar (n+1) valores de k.
>>
>> Para k de -1 até -n, este polinômio é igual a zero, e para k=n+1 ele vale
>> 1.
>> A partir daí, usando a fórmula de interpolação de Newton (ou uma
>> modificação do triângulo de Pascal), este polinômio é inteiro para
>> todo n inteiro.
>
> Como isso pode ser verificado?
>
>>
>>
>> Em 27/11/10, Carlos Alberto da Silva Victor
>> escreveu:
>>> Olá Paulo,
>>> Verifique se esta ideia satisfaz o que desejas .
>>>
>>>  Por indução :
>>>
>>> 1) para n=1,2 e 3 é fácil de observar tal fato .
>>> 2) hipótese : válida para  n fatores consecutivos.
>>>
>>> 3) Tomemos (n+1) fatores consecutivos :P =  k(k+1)(k+n-1).(k+n) .Por
>>> hipótese k(k+1)(k+n-1) é divisível por n! . Não é difícil mostrar que
>>> o
>>> produto de n fatores consecutivos é divisível por n .Como P possui (n+1)
>>> fatores, temos que o valor (n+1) está em um dos fatores(ou divisor de um
>>> dos
>>> fatores) de P e, já que n e (n+1) são primos entre si , P será divisível
>>> por
>>> n! e (n+1) , ou seja, divisível por (n+1)! , ok ?
>>>
>>> Abraços
>>>
>>> Carlos  Victor
>>>
>>>
>>>
>>>
>>>
>>> Em 27 de novembro de 2010 18:29, Paulo Argolo
>>> escreveu:
>>>
>>>>  Obrigado, Tiago.
>>>>
>>>> O que desejo, na verdade, é obter uma demonstração que não use
>>>> propriedades
>>>> dos coeficientes binomiais, nem recorra à Análise Combinatória. Em suma:
>>>> gostaria de ver uma prova puramente aritmética.
>>>>
>>>> Abraços do Paulo!
>>>>
>>>>
>>>>
>>>
>>
>>
>> --
>> /**/
>> Quadrinista e Taverneiro!
>>
>> http://tavernadofimdomundo.blogspot.com >> Quadrinhos, histórioas e afins
>> http://baratoeletrico.blogspot.com />> Um pouco sobre elétrons em
>> movimento
>> http://bridget-torres.blogspot.com/ >> Personal! Do not edit!
>>
>> =
>> Instru�ões para entrar na lista, sair da lista e usar a lista em
>> http://www.mat.puc-rio.br/~obmlistas/obm-l.html
>> =
>>
>
>
> --
> Henrique
>
> =
> Instru�ões para entrar na lista, sair da lista e usar a lista em
> http://www.mat.puc-rio.br/~obmlistas/obm-l.html
> =
>


-- 
/**/
Quadrinista e Taverneiro!

http://tavernadofimdomundo.blogspot.com >> Quadrinhos, histórioas e afins
http://baratoeletrico.blogspot.com />> Um pouco sobre elétrons em movimento
http://bridget-torres.blogspot.com/ >> Personal! Do not edit!

=
Instru��es para entrar na lista, sair da lista e usar a lista em
http://www.mat.puc-rio.br/~obmlistas/obm-l.html
=


[obm-l] Re: [obm-l] Re: [obm-l] Re: [obm-l] O produto de n i nteiros consecutivos é múltiplo do fatorial de n

2010-11-28 Por tôpico Johann Dirichlet
Por que este povo tem tanto pavor de "uma prova que não use outros
conceitos alem do enunciado"?
Eu mesmo conheço vários problemas que são resolvidos usando outras
técnicas. Na IMO de Glasgow teve um problema de Teoria dos Números com
uma solução que usava polinômios. E tem um monte de problemas de
teoria dos números que se resolvem usando técnicas de combinatória (o
teorema de Euler-Fermat, por exemplo).

De todo modo, só pra não perder o propósito da mensagem:

Uma maneira seria observar que f(n,k)=(k+1)(k+2)...(k+n)/n! é um
polinômio de grau n em k.
Ele é completamnte determinado se eu utilizar (n+1) valores de k.

Para k de -1 até -n, este polinômio é igual a zero, e para k=n+1 ele vale 1.
A partir daí, usando a fórmula de interpolação de Newton (ou uma
modificação do triângulo de Pascal), este polinômio é inteiro para
todo n inteiro.


Em 27/11/10, Carlos Alberto da Silva Victor escreveu:
> Olá Paulo,
> Verifique se esta ideia satisfaz o que desejas .
>
>  Por indução :
>
> 1) para n=1,2 e 3 é fácil de observar tal fato .
> 2) hipótese : válida para  n fatores consecutivos.
>
> 3) Tomemos (n+1) fatores consecutivos :P =  k(k+1)(k+n-1).(k+n) .Por
> hipótese k(k+1)(k+n-1) é divisível por n! . Não é difícil mostrar que o
> produto de n fatores consecutivos é divisível por n .Como P possui (n+1)
> fatores, temos que o valor (n+1) está em um dos fatores(ou divisor de um dos
> fatores) de P e, já que n e (n+1) são primos entre si , P será divisível por
> n! e (n+1) , ou seja, divisível por (n+1)! , ok ?
>
> Abraços
>
> Carlos  Victor
>
>
>
>
>
> Em 27 de novembro de 2010 18:29, Paulo Argolo
> escreveu:
>
>>  Obrigado, Tiago.
>>
>> O que desejo, na verdade, é obter uma demonstração que não use
>> propriedades
>> dos coeficientes binomiais, nem recorra à Análise Combinatória. Em suma:
>> gostaria de ver uma prova puramente aritmética.
>>
>> Abraços do Paulo!
>>
>>
>>
>


-- 
/**/
Quadrinista e Taverneiro!

http://tavernadofimdomundo.blogspot.com >> Quadrinhos, histórioas e afins
http://baratoeletrico.blogspot.com />> Um pouco sobre elétrons em movimento
http://bridget-torres.blogspot.com/ >> Personal! Do not edit!

=
Instru��es para entrar na lista, sair da lista e usar a lista em
http://www.mat.puc-rio.br/~obmlistas/obm-l.html
=


Re: [obm-l] geometria com 20 graus

2010-11-24 Por tôpico Johann Dirichlet
Se não me engano este problema foi proposto numa Eureka! Assim que der
eu vejo qual o número, mas é recente (entre as últimas 8 ou 10).

Em 16/11/10, Luís Lopes escreveu:
>
> Sauda,c~oes,
>
> Pediram-me a solução do problema abaixo. Como muito provavelmente
> tal problema já apareceu por aqui, pergunto se alguém teria a solução
> dele à mão.
>
> Obrigado.
>
> []'s
> Luís
>
>
>
>
>  Prezado Luís mais uma vez venho pedir a sua ajuda na solução do exercicio
> abaixo.
> Dado o triângulo ABC ,isósceles, com BÂC medindo 20 graus ,onde AC = AB.
> Pelo vértice B traça-se até AC, BD tal que AD = BC. Calcule o valor do
> ângulo B^DC.
>
>
>   


-- 
/**/
Quadrinista e Taverneiro!

http://tavernadofimdomundo.blogspot.com >> Quadrinhos, histórioas e afins
http://baratoeletrico.blogspot.com />> Um pouco sobre elétrons em movimento
http://bridget-torres.blogspot.com/ >> Personal! Do not edit!

=
Instru��es para entrar na lista, sair da lista e usar a lista em
http://www.mat.puc-rio.br/~obmlistas/obm-l.html
=


[obm-l] Re: [obm-l] Como provar que C(n,p) é número natura l?

2010-11-24 Por tôpico Johann Dirichlet
A melhor que eu posso imaginar e simplesmente pensar assim:

1 - Determine, para cada primo p, a maior potencia de p que divide n!
(ou seja, descubra na raça a fatoração de n!).
E facil: basta contar quanto cada p, 2p, 3p, ... (p-1)p, p^2, etc vai
contribuir (voce vai obter um somatorio).
Isso tem mais a ver com teoria dos conjuntos que com teoria dos números.

2 - A partir dai, fica facil provar que a potencia de p que aparece no
numerador não é menor que no denominador.



Em 23/11/10, Pedro Chaves escreveu:
>
> Amigos da Lista,
>
>
> Como posso provar que C(n,p) é um número natural, usando apenas a definição
> C(n,p) = n! / [p! (n-p)!]?
> (p e n são números naturais, com p menor ou igual a n)
> Meu objetivo é obter uma prova direta, isto é, que não recorra à Análise
> Combinatória, nem às  propriedades dos números binomiais.
>
> Muito grato!
>
> Pedro Chaves
>   


-- 
/**/
Quadrinista e Taverneiro!

http://tavernadofimdomundo.blogspot.com >> Quadrinhos, histórioas e afins
http://baratoeletrico.blogspot.com />> Um pouco sobre elétrons em movimento
http://bridget-torres.blogspot.com/ >> Personal! Do not edit!

=
Instru��es para entrar na lista, sair da lista e usar a lista em
http://www.mat.puc-rio.br/~obmlistas/obm-l.html
=


[obm-l] Re: [obm-l] mdc (a^x – 1, a^y – 1, a^z – 1, .. .......) = [a^mdc(x, y, z,...)] – 1

2010-11-23 Por tôpico Johann Dirichlet
Para dois caras, é fácil demonstrar na raça, usando Euclides:
MDC(a^x-1,a^y-1)= MDC(a^x-1,a^(x-y)-1). Daí se faz por indução no
número de variáveis.

Em 23/11/10, Paulo  Argolo escreveu:
> Caros Colegas,
> Estou refazendo o enunciado da questão.
>
> Como provar o teorema seguinte sobre máximo divisor comum?
>
> TEOREMA:
>
> O máximo divisor comum (mdc) dos números do tipo
> a^x -1 , onde a e x são números inteiros maiores do que 1, é dado pela
> expressão abaixo:
> mdc(a^x - 1, a^y - 1, a^z - 1, ... ) = [a^mdc(x, y, z,...)] -1
>
> Grato,
>
> Paulo Argolo
> =
> Instru�ões para entrar na lista, sair da lista e usar a lista em
> http://www.mat.puc-rio.br/~obmlistas/obm-l.html
> =
>


-- 
/**/
Quadrinista e Taverneiro!

http://tavernadofimdomundo.blogspot.com >> Quadrinhos, histórioas e afins
http://baratoeletrico.blogspot.com />> Um pouco sobre elétrons em movimento
http://bridget-torres.blogspot.com/ >> Personal! Do not edit!

=
Instru��es para entrar na lista, sair da lista e usar a lista em
http://www.mat.puc-rio.br/~obmlistas/obm-l.html
=


[obm-l] Re: [obm-l] Alguém conseguirá provar?

2010-11-21 Por tôpico Johann Dirichlet
O que significa limitada primorialmente?


Em 05/11/10, Marco Bivar escreveu:
> Prove isto:
>
> Em toda sucessão (c_1, c_2, ..., c_w) de números compostos limitada
> primorialmente, se c_i = z_i . x_i, i=1,2,..., w, onde z_i é um primo ou
> produto de primos tal que z_i<=x_i e z_1x_2>...>x_w,
> onde x_i é um primo ou produto de primos.
>
> Obs.: Tomar apenas números primos positivos.
>
>
> --
> Marco A. B. C. Jr.
>


-- 
/**/
Quadrinista e Taverneiro!

http://tavernadofimdomundo.blogspot.com >> Quadrinhos, histórioas e afins
http://baratoeletrico.blogspot.com />> Um pouco sobre elétrons em movimento
http://bridget-torres.blogspot.com/ >> Personal! Do not edit!

=
Instru��es para entrar na lista, sair da lista e usar a lista em
http://www.mat.puc-rio.br/~obmlistas/obm-l.html
=


Re: [obm-l] Teorema sobre logaritmos irracionais

2010-11-09 Por tôpico Johann Dirichlet
log_b a= x é o mesmo que a^x=b.
Usando o lema da fatoração única, vemos que se x fosse racional então
a e b teriam os mesmos fatores primos e com os expoentes múltiplos.

Em 06/11/10, Pedro Chaves escreveu:
>
> Estou reapresentando o teorema sobre logaritmos, pois não consegui ainda uma
> demonstração completa. Peço, mais uma vez, a colaboração dos colegas.
>
> Teorema:
> Sendo a e b números inteiros maiores do que 1, que não podem ser
> representados como potências de mesma base (inteira), com expoente inteiro,
> então o logaritmo de a, na base b, é um número irracional.
>
>
> Um abraço do Pedro!   


-- 
/**/
Quadrinista e Taverneiro!

http://tavernadofimdomundo.blogspot.com >> Quadrinhos, histórioas e afins
http://baratoeletrico.blogspot.com />> Um pouco sobre elétrons em movimento
http://bridget-torres.blogspot.com/ >> Personal! Do not edit!

=
Instru��es para entrar na lista, sair da lista e usar a lista em
http://www.mat.puc-rio.br/~obmlistas/obm-l.html
=


[obm-l] Re: [obm-l] Polinômios(ajuda)

2010-11-02 Por tôpico Johann Dirichlet
P(x)-7=(x-a)(x-b)(x-c)(x-d)*Q(x), em que Q é um polinomio de
coeficientes inteiros
7=14-7=(x-a)(x-b)(x-c)(x-d)*Q(x), o que se torna impossívl pois 7 é primo


Em 02/11/10, marcone augusto araújo
borges escreveu:
>
> Mostre q,se um polinômio P(x),com coeficientes inteiros,assume o valor 7
> para 4 valores inteiros e distintos de x,então ele não pode assumir o valor
> 14 para nenhum valor inteiro de x.


-- 
/**/
Quadrinista e Taverneiro!

http://tavernadofimdomundo.blogspot.com >> Quadrinhos, histórioas e afins
http://baratoeletrico.blogspot.com />> Um pouco sobre elétrons em movimento
http://bridget-torres.blogspot.com/ >> Personal! Do not edit!

=
Instru��es para entrar na lista, sair da lista e usar a lista em
http://www.mat.puc-rio.br/~obmlistas/obm-l.html
=


[obm-l] Olimpíadas ao Redor do Mundo

2010-10-30 Por tôpico Johann Dirichlet
Eis um problema legal:

Temos três caixas, cada uma com pelo menos uma bolinha dentro.
Podemos dobrar o total de bolinhas de uma das caixas, tirando as
bolinhas de uma das outras caixas para tal.
É possível esvaziar uma das caixas, fazendo uma escolha acertada de
operações permitidas?

-- 
/**/
Quadrinista e Taverneiro!

http://tavernadofimdomundo.blogspot.com >> Quadrinhos, histórioas e afins
http://baratoeletrico.blogspot.com />> Um pouco sobre elétrons em movimento
http://bridget-torres.blogspot.com/ >> Personal! Do not edit!

=
Instru��es para entrar na lista, sair da lista e usar a lista em
http://www.mat.puc-rio.br/~obmlistas/obm-l.html
=


[obm-l] Re: [obm-l] Conjuntos Enumeráveis

2010-10-30 Por tôpico Johann Dirichlet
A ideia não é difícil, e o mais importante é o caso 2: X x Yé
enumerável se X,Y são.
Faz assim: os elementos de X são x1,x2,... e os de Y são y1,y2,y3...
(ambos são enumeráveis, então eu posso colocar índices)

Então podemos fazer assim:
Para cada natural N = 1,2,3,4,5...
liste os pares (xi,yj) tal que i+j=N

Teremos algo assim:
(x1,y1)
(x1,y2),(x2,y1)
(x1,y3),(x2,y2),(x3,y1)
E por aí vai...

Aí, basta aplicar o caso n=2 fazendo X=A1 x A2 x ... x An e Y=A(n+1)

Sem indução é mais fácil ainda: basta utilizar o algoritmo acima.

Em 30/10/10, Luiz Neto Neto escreveu:
> Sejam A1,An conjuntos enumeráveis, então A1xxAn é enumerável(Use
> Indução)
>
>
>
>


-- 
/**/
Quadrinista e Taverneiro!

http://tavernadofimdomundo.blogspot.com >> Quadrinhos, histórioas e afins
http://baratoeletrico.blogspot.com />> Um pouco sobre elétrons em movimento
http://bridget-torres.blogspot.com/ >> Personal! Do not edit!

=
Instru��es para entrar na lista, sair da lista e usar a lista em
http://www.mat.puc-rio.br/~obmlistas/obm-l.html
=


Re: [obm-l] Conjectura Sobre Primos

2010-10-29 Por tôpico Johann Dirichlet
Mais algumas:

Qualquer natural grande pode ser escrito como uma soma de 18 primos.

Qualquer par grande pode ser escrito ou como p+q ou como p+qr, em que
p,q,r são primos.


Em 29/10/10, Ralph Teixeira escreveu:
> Oi, Luiz/Felipe.
>
> A conjectura de Goldbach diz que todo número par (a partir de 4) pode ser
> escrito como a soma de dois primos. Até onde eu sei, ninguém ainda provou
> esta conjectura... mas a Wikipedia e Mathworld dizem que já foram testados
> todos os números até 12*10^17, e ate lá, valeu.
> (http://en.wikipedia.org/wiki/Goldbach's_conjecture
> http://mathworld.wolfram.com/GoldbachConjecture.html))
>
> E, pelo que li lá (não sei nada da área), há resultados "probabilísticos"
> também nesta direção (tipo, "a densidade de inteiros que não satisfaz a
> conjectura é nula" ou algo assim).
>
> Ou seja, se ela for verdadeira, daria para generalizar a sua conjectura
> para:
>
> "Qualquer **ímpar** (não precisa ser primo) maior que 3 pode ser obtido
> através da seguinte relação:
>
> P=P_a+P_b+1"
>
> (nem precisa das outras opções; afinal, P-1 é par, então entra na conjectura
> de Goldbach)
> (ah, e poderia ser -1 ao invés de +1 ali em cima, tanto faz; basta uma das
> opções)
>
> Abraço,
> Ralph
>
>
> 2010/10/29 luiz silva 
>
>>   Prezados,
>>
>> Vcs conhecem alguma conjectura sobre primos que fale o seguinte :
>>
>> Qqer número primo maior que 3 pode ser obtido através das seguintes
>> relações :
>>
>> a) P = Pa +2 ou - 2
>>
>> b) P = Pa+Pb + 1 ou - 1
>>
>> Onde P, Pa e Pb são primos.
>>
>> Eu peguei uma tabela com números primos (2, 3 105xxx - cento e cinco
>> mil e alguma coisa) e fui fazendo este teste  na tabela(matematica
>> experimental..rs)pegando, aleatoriamente, dois primos e aplicando a
>> formula
>> b ou pegando um determinado primo, e procurando por dois outros para
>> aplicar
>> a fórmula b. Para minha surpresa, foi sempre fácil conseguir estes
>> números.
>>
>> Pensei em fazer um experimento para fazer um teste de hipóteses,
>> relacionando o sucesso em obter aleatoriamente uma certa qde de números
>> primos, com as formulas, com a probabilidade que teríamos de obter esta
>> mesma qde de nos. primos, ao escolhermos, diretamente, neste intervalo,
>> sem
>> uso da fórmula.
>>
>> Tipo um teste de sucesso, calculando a probabilide de se obter um primo
>> com
>> a soma de dois primos +-1 ou primo +2 e o total de possibilidades de
>> números
>> possíveis, relacionando com o ocorrido no "experimento"..Como mais ou
>> menos relacionar a probabilidade de um determinado numero escolhido ao
>> acaso, em um intervalo, seja  primo (meio que relacionado a Hipotese de
>> Riemann)
>>
>> Não sei se ficou claro, e se isto foi muita viagem.
>>
>> Abs
>> Felipe
>>
>>
>


-- 
/**/
Quadrinista e Taverneiro!

http://tavernadofimdomundo.blogspot.com >> Quadrinhos, histórioas e afins
http://baratoeletrico.blogspot.com />> Um pouco sobre elétrons em movimento
http://bridget-torres.blogspot.com/ >> Personal! Do not edit!

=
Instru��es para entrar na lista, sair da lista e usar a lista em
http://www.mat.puc-rio.br/~obmlistas/obm-l.html
=


[obm-l] Re: [obm-l] Re: [obm-l] Determinar parte inteira e p arte fracionária

2010-10-28 Por tôpico Johann Dirichlet
Eu creio que a resposta é o famigerado -9.

Melhor ser mais preciso nas definições: "parte inteira de x" ou "maior
inteiro que não supera x"?

Em 27/10/10, Adalberto Dornelles escreveu:
> Olá Pedro,
>
> A resposta depende de como você define "parte fracionária". Parece que não
> há uma definição universalmente aceita. Veja por exemplo:
> http://mathworld.wolfram.com/FractionalPart.html
>
> Talvez a definição mais
> "usual" seja
>
> "Parte fracionaria de x" = x - "arredondar para baixo x"
>
> ou, em notação (também não muito universal),
>
> {x} = x - [x]
>
> Assim,
> {8,25} = 8,25 - [8,25] = 8,25 - 8 = 0,25
> e
> {-8,25} = -8,25 - [-8,25] = -8,25 - (-9) = 0,75
>
> Agora, observe que "arredondar para baixo" significa "encontrar o inteiro n
> à esquerda de x" ou "arredondar na direção de - infinito". No entanto, outra
> definição de "arredondar para baixo" pode ser "arredondar na direção de 0",
> assim
>
> {8,25} = 8,25 - [8,25] = 8,25 - 8 = 0,25
> e
> {-8,25} = -8,25 - [-8,25] = -8,25 - (-8) = -0,25
>
> Em computação, geralmente as linguagens [Matlab, Mathematica, etc.] tem
> comandos relacionados a isso (floor, ceil, fix, round, mod, ...). cuja
> definição não é muito homogênea. Sempre recomendo "leia o manual" para saber
> como a linguagem define cada coisa.
>
> Veja também http://en.wikipedia.org/wiki/Floor_and_ceiling_functions
>
> Abraço,
> Adalberto
>
> Em 27 de outubro de 2010 08:09, Pedro Chaves escreveu:
>
>>  Caros Colegas,
>>
>> Qual é a parte inteira e a parte fracionária do número real negativo
>> -8,25?
>>
>> Obrigado!
>> Pedro Chaves
>>
>


-- 
/**/
Quadrinista e Taverneiro!

http://tavernadofimdomundo.blogspot.com >> Quadrinhos, histórioas e afins
http://baratoeletrico.blogspot.com />> Um pouco sobre elétrons em movimento
http://bridget-torres.blogspot.com/ >> Personal! Do not edit!

=
Instru��es para entrar na lista, sair da lista e usar a lista em
http://www.mat.puc-rio.br/~obmlistas/obm-l.html
=


[obm-l] Re: [obm-l] Re: [obm-l] Fibonacci e Razão Áurea

2010-10-28 Por tôpico Johann Dirichlet
Poxa, alguém tem um exemplo de uma sequencia x_n que sempre é positiva
mas o limite não é?
Eu acho que 1/n tende a zero sempre sendo maior que zero, mas tem que
tomar cuidado com o "estritamente positivo".

P.S.: um treco legal sobre racionais tendendo a irracionais é o artigo
do Gugu na Eureka! 3, sobre frações contínuas. Se eu não me engano os
F/F são reduzidas da fração contínua da razão áurea.


Em 27/10/10, Ralph Teixeira escreveu:
> "Como pode uma razão de números inteiros convergir para um número irracional
> ?"
>
> Bom, como ilustração, pi é irracional, e é o limite da sequencia:
>
> 3
> 3,1=31/10
> 3,14=314/100
> 3,141=3141/1000
> 3,1415=31415/1
> ...
>
> Acho que este exemplo deve te convencer que qualquer número irracional é
> limite de uma sequencia de racionais (razões entre inteiros).
>
> ---///---
>
> Para ponderar: raciocínios do tipo: "se cada x_n tem a propriedade P, então
> lim(x_n) tem a propriedade P" são muito naturais. Infelizmente, este tipo de
> raciocínio está frequentemente errado! Por exemplo, seu espanto acima seria
> representado pela frase:
>
> "se cada x_n é racional (quociente de inteiros), então lim(x_n), se existir,
> também será."
> (FALSO!)
>
> Outras frases FALSAS do mesmo tipo (todos os limites são quando n->+Inf):
> "se cada x_n é positivo, então lim(x_n) é positivo."
> "se cada x_n é menor que 1, então lim(x_n) é menor que 1" (que, no fundo no
> fundo, é o "problema" que o pessoal tem com 0,9...=1)
> "se cada uma das funções f_n(x) é contínua, então f(x)=lim f_n(x) é
> contínua"
> "se cada uma das funções f_n(x) é derivável, então f(x)=lim f_n(x) é
> derivável"
>
> Bom, e assim por diante. O que eu quero dizer é que passar "um raciocínio"
> ao limite é perigoso (mas, quando funciona, é bem legal)
>
> Abraço,
>
> Ralph
>
>
> 2010/10/27 luiz silva 
>
>>   Pessoal,
>>
>> Pelo que lembro, a razão entre dois números consecutivos(an+1/an), da
>> sequência Fibonacci converge para 1,61834 quando n tende a infinito.
>> Porém, pelo que lembro, tb, este número é um número irracional.
>>
>> Como pode uma razão de números inteiros convergir para um número
>> irracional
>> ?
>>
>> Abs
>> Felipe
>>
>>
>


-- 
/**/
Quadrinista e Taverneiro!

http://tavernadofimdomundo.blogspot.com >> Quadrinhos, histórioas e afins
http://baratoeletrico.blogspot.com />> Um pouco sobre elétrons em movimento
http://bridget-torres.blogspot.com/ >> Personal! Do not edit!

=
Instru��es para entrar na lista, sair da lista e usar a lista em
http://www.mat.puc-rio.br/~obmlistas/obm-l.html
=


[obm-l] Re: [obm-l] Parte fracionária = parte decimal?

2010-10-24 Por tôpico Johann Dirichlet
Com certeza! (eu acho...)

Por definicao, a parte inteira de um real e o maior inteiro que fica
abaixo deste real. Por exemplo, 7 e a parte inteira de 7,1234.

A parte fracionaria e esta diferenca entre o numero e sua parte
inteira. No caso, 0,1234.

O lance e que as vezes voce tem um numero feio. Por exemplo,
pi=3,141592..., e e mais pratico escrever pi-3 do que 0,141592...

Um exemplo: qual a parte inteira de  -3,1415?

Em 22/10/10, Guilherme Vieira escreveu:
>
> Caros Colegas,
>
> Dado o número decimal 7,1234, pode-se dizer que sua parte fracionária é
> 0,1234?  Isto é, a parte fracionária é a parte decimal?
>
> Um abração!
> Guilherme
>
>   


-- 
/**/
Quadrinista e Taverneiro!

http://tavernadofimdomundo.blogspot.com >> Quadrinhos, histórioas e afins
http://baratoeletrico.blogspot.com />> Um pouco sobre elétrons em movimento
http://bridget-torres.blogspot.com/ >> Personal! Do not edit!

=
Instru��es para entrar na lista, sair da lista e usar a lista em
http://www.mat.puc-rio.br/~obmlistas/obm-l.html
=


[obm-l] Re: [obm-l] Quadrados mágicos: problema da Eureka 0 1:

2010-10-24 Por tôpico Johann Dirichlet
O unico pre-requisito para se ler uma Eureka! e ler as anteriores.
Desculpe falar algo tao obvio, mas e que nao tem bem um pre-Eureka! no
Brasil, ate onde eu sei. Se voce encara uma leitura em ingles, a
melhor referencia que conheco e o site mathlinks.ro. La tem tutoriais
e artigos de todos os niveis. Tambem tem um arquivo de problemas de
olimpiadas de todo o mundo.
Por ora, esta e minha recomendacao: o site da OBM e as Eureka!s, todas elas.

Bem, eu pretendo lancar um arquivo pessoal contendo solucoes da
Eureka! e de alguns problemas que eu fiz ha milenios em papel, mas
ainda não posso garantir nada...

Quanto ao problema, tente resolver o sistema de equacoes gerado pelas
somas, e voce descobrira que o numero central e igual a 5. Depois eu
posto algo competo.

Em 23/10/10, Rafael escreveu:
> Olá, pessoal.
>
> Antes de comentar sobre um problema da Eureka 01, uma pergunta:
>
> Alguém aqui costuma resolver todos os problemas sem solução da revista
> Eureka e deixar em um arquivo no word, por exemplo ? Se sim, gostaria muito
> de um arquivo com esses problemas que contém apenas o gabarito e não a
> solução.
>
> Uma vez enviaram aqui um arquivo com questões resolvidas do IME, inclusive
> bem antigas. Alguém aqui tem arquivos de questões resolvidas assim também,
> mas olímpicas ? Seja da Eureka ou não. No site Excalibur, há muitos
> problemas assim, mas o nível é bem alto. Gostaria de um arquivo com
> problemas resolvidos de forma preparatória à leitura das Eurekas.
>
> Comecei a ler as Eurekas. Veja este problema da Eureka 01:
>
> Você já conhece o quadrado mágico de ordem 3: a soma dos números das linhas,
> das colunas e das diagonais é 15. A figura a seguir mostra uma das oito
> possibilidades de escrever os números no quadrado:
>
> a11 = 8; a12 = 1; a13 = 6
> a21 = 3; a22 = 5; a23 = 7
> a31 = 4; a32 = 9; a33 = 2
>
> O único número que não pode mudar de posição em todos esses quadrados
> mágicos é:
>
>
>
> a) 1  b) 3  c) 5  d) 7  e) 9
>
>
> Eu percebi que a correta é a C (gabarito), pois se girarmos o quadrado no
> sentido horário ou anti-horário, teremos 4 quadrados (incluindo o original)
> e em todos eles não houve mudança do número 5 em relação ao quadrado do
> enunciado. Eu gostaria de uma solução mais formal e por que são 8
> possibilidades e não 4.
>
> Obs1: Saber todo o conteúdo do ensino médio já é o suficiente para ler e
> entender as Revistas Eurekas OU deve haver uma outra condição prévia, como
> ler algum livro específico ou estudar por problemas de outros sites ?
> Pergunto isso, pois abri aleatoriamente algumas revistas e li alguns "termos
> matemáticos" não abordados em livros normais do ensino médio, daí pensei: "-
> OU os elaboradores das Eureka estão partindo do pressuposto que os leitores
> já saibam determinadas coisas (mesmo que não estejam em livros regulares do
> ensino médio); OU há uma gradação de conhecimentos nas revistas, ou seja, se
> não entendeu algum termo ou conceito OLÍMPICOS em alguma revista, então é
> provável que haja um explicação em alguma das revistas anteriores.
>
>
> Regards,
> Rafael


-- 
/**/
Quadrinista e Taverneiro!

http://tavernadofimdomundo.blogspot.com >> Quadrinhos, histórioas e afins
http://baratoeletrico.blogspot.com />> Um pouco sobre elétrons em movimento
http://bridget-torres.blogspot.com/ >> Personal! Do not edit!

=
Instru��es para entrar na lista, sair da lista e usar a lista em
http://www.mat.puc-rio.br/~obmlistas/obm-l.html
=


Re: [obm-l] Algoritmo de Euclides estendido

2010-10-20 Por tôpico Johann Dirichlet
Suponha que p é divisor de ab, mas não seja de a.
Então a e p serão primos entre si, e assim podemos achar x e y tais que
xa+yp=1
Multiplicando por b, temos
xab+ybp=b
Como xab e ybp são múltiplos de p, a soma também será. É isso!


Em 15/10/10, luiz escreveu:
>
> Alguem pode me ajudar.?
>
>
>
>
>
> O algoritmo de Euclides estendido é o seguinte:
>
> "Dados a e b inteiros, seja d = mdc(a,b) então existem r e s inteiros tais
> que sa+rb=d."
>
> Usando o algoritmo de Euclides estendido mostre que se p é primo e a e b são
> inteiros tais que p é divisor de ab, então p é divisor de a ou p é divisor
> de b.
>


-- 
/**/
Quadrinista e Taverneiro!

http://tavernadofimdomundo.blogspot.com >> Quadrinhos, histórioas e afins
http://baratoeletrico.blogspot.com />> Um pouco sobre elétrons em movimento
http://bridget-torres.blogspot.com/ >> Personal! Do not edit!

=
Instru��es para entrar na lista, sair da lista e usar a lista em
http://www.mat.puc-rio.br/~obmlistas/obm-l.html
=


Re: [obm-l] Semelhantes ou iguais?

2010-10-07 Por tôpico Johann Dirichlet
Pense no mapa de uma cidade. Os  Ãngulos são iguais, mas as distâncias não.

Em 07/10/10, Nathália Santos escreveu:
> Serão sempre semelhantes, mas não necessariamente iguais, já que ângulos
> iguais não determinam sempre lados iguais.
>
> From: rhilbert1...@hotmail.com
> To: obm-l@mat.puc-rio.br
> Subject: [obm-l] Semelhantes ou iguais?
> Date: Wed, 6 Oct 2010 22:25:19 +
>
>
>
>
>
> Colegas, uma discussão sem solução, acontenceu por conta da seguinte dúvida.
>
> "Dois triângulos com os seus ângulos, respectivamente, de mesma medida, são
> iguais (lados  respectivos de mesma medida) ou semelhantes (lados
> respectivos proporcionais)"?
>
> Exemplo: Triângulos ABC e A'B'C'  com   ângulos A=A, B=B e C=C  => AB=A'B',
> AC=A'C' e BC=B'C' ?
>
>
>
>
>
>
>
>
>
>
>   
>   


-- 
/**/
Quadrinista e Taverneiro!

http://tavernadofimdomundo.blogspot.com >> Quadrinhos, histórioas e afins
http://baratoeletrico.blogspot.com />> Um pouco sobre elétrons em movimento
http://bridget-torres.blogspot.com/ >> Personal! Do not edit!

=
Instru��es para entrar na lista, sair da lista e usar a lista em
http://www.mat.puc-rio.br/~obmlistas/obm-l.html
=


[obm-l] Spam político e terrorismo, aqui não!! (era: REPAS SE COM URGENCIA, URGENCIA)

2010-10-05 Por tôpico Johann Dirichlet
Olá pessoas!
Bem, não sei se estou sendo redundandte ou mesmo se o que eu direi irá
ofender a inteligência dos presentes desta lista:
Pesquise as fontes sempre que surgir um e-mail ou mensagem destas por
esta lista (e de qualquer outro meio de comunicação). Como diziam na
Internet, "O Google é seu amigo".

Enfim, é isso. Tem algum problema bom de Matemática aí?

Em 05/10/10, Saon Crispim Vieira escreveu:
> Olá pessoal, boa tarde.
> Isto é pura fantasia! É inaceitável ouvir tais acusações sem indícios e/ou
> provas!
> Sou espírita e não eleitor da Dilma, mas este não é o nível de debate que
> espero desta lista.
> Segue anexa uma matéria interessante, de uma fonte confiável e com tema mais
> relacionado à proposta desta lista.
>
> http://www.nature.com/news/2010/100929/full/467511b.html?s=news_rss
>
> 2010/10/5 Aline Rosane 
>
>>
>>
>>
>>  *PENSE BEM ANTES DE VOTAR NA DONA DILMA E SUA GANG !!!
>> *
>>
>>
>>
>> Cordialmente,
>> Camargo Júnior
>> 69-3421-3061
>>
>>
>>
>> --
>> *Resumo de projetos que podem virar lei após as eleição: ENTÃO VOTE NA
>> DILMA*
>>
>> **
>>
>> *Fica proibido fazer:*
>> ·   Cultos ou evangelismo na rua (Reforma Constitucional)
>> ·   Programas evangélicos na televisão por mais de uma hora por dia.
>> ·   Programa de rádio ou televisão, quem não possuir faculdade
>> de'jornalismo'.
>> ·   Pregar sobre dízimos e ofertas, havendo reclamações, obreiros serão
>> presos.
>>
>> *Quanto aos cultos:*
>> - Cultos somente com portas fechadas (Reforma Constitucional)
>> - As igrejas serão obrigadas a pagarem impostos sobre dízimos,
>> ofertas e contribuições,as católicas também.
>> - Será considerado crime pregar sobre espiritismo, feitiçaria e idolatria,
>> e também veicular mensagem no rádio, televisão, jornais e internet,
>> sobre essas práticas contrárias a Palavra de Deus.
>> - Pastores e Padres que forem presos por pregar sobre práticas condenadas
>> pela
>> Bíblia Sagrada (homossexualismo, idolatria e espiritismo), não terão
>> direito a se defender por meio de ação judicial.
>>
>> *Se estabeleça:*
>> -O dia do “Orgulho Gay” e que seja oficializado em todas as cidades
>> brasileiras e comemorado nas Instituições de Ensino Fundamental
>> (primeira a 8.a série), público e particular.
>> -Que as Igrejas que se negarem a realização das solenidades
>> dos casamentos de homem com homem e de mulher com mulher,
>> estarão fazendo “discriminação”, seja multadas e seus pastores e padres
>> processados criminalmente por discriminação e desobediência civil.
>>
>>
>> *Projeto nº 4.720/03 - Altera a legislação constitucional** **
>>
>> -Projeto nº 3.331/04 –* Altera o artigo 12 da Lei nº 9.250/95, que trata
>> da legislação do imposto de renda das 'pessoas físicas' Se convertidos em
>> Lei, os dois projetos obrigariam as igrejas a recolherem impostos sobre
>> dízimos, ofertas e contribuições.
>>
>>
>> *-**Projeto nº 299/99 –* Altera o código brasileiro de telecomunicações
>> (Lei 4.117/62).
>> Se aprovado, reduziria programas evangélicos
>> no rádio e televisão a apenas uma hora.
>> **
>>
>> *-Projeto nº 6.398/05 –* Regulamenta a profissão de Jornalista
>> Contém artigos que estabelecem que só poderão fazer programas
>> de rádio e televisão, pessoas com formação em JORNALISMO, Significa que
>> pastores,padres e ouros sem a formação em jornalismo não
>> poderão fazer programas através desses meios.
>>
>>
>> *-Projeto nº 1.154/03 –* Proíbe veiculação de programas  em que o teor
>> seja considerado preconceito religioso. Se aprovado, será considerado
>> crime
>> pregar sobre idolatria, feitiçaria e rituais satânicos. Será proibido que
>> mensagens sobre essas práticas sejam veiculadas no rádio, televisão,
>> jornais
>> e internet. A verdade sobre esses atos contrários
>> a Palavra de Deus, não poderá mais ser mostrada.
>>
>>
>> *-Projeto nº 952/03 – *Estabelece que é crime atos religiosos que possam
>> ser considerados abusivos a boa-fé das pessoas. Convertido em Lei, pelo
>> número de reclamações, pastores serão considerados 'criminosos' por
>> pregarem
>> sobre dízimos e ofertas.
>>
>>
>> *-Projeto nº 4.270/04[/b] –** *Determina que comentários feitos contra
>> ações praticadas por grupos religiosos possam ser passíveis de ação civil.
>> Se convertido em Lei, as Igrejas Evangélicas ficariam proibidas de pregar
>> sobre práticas condenadas pela Bíblia Sagrada, como espiritismo,
>> feitiçaria,
>> idolatria e outras. Se o fizerem, não terão direito a se defender por meio
>> de ação judicial.
>>
>>
>> *-**Projeto de nº 216/04[/b] –* Torna inelegível a função religiosa com a
>> governamental. Significa que todo pastor ou líder religioso lançado a
>> candidaturas para qualquer cargo político, não poderá de forma alguma
>> exercer trabalhos na igreja.
>>
>>
>
>
> --
> Saon Crispim Vieira
>


-- 
/**/
Quadrinista e Taverneiro!

http://tavernadofimdomundo.blogspot.com >> Quadrinhos, histórioas e afins
http://baratoeletrico.blogspot.com />> Um pouco

Re: [obm-l] REPASSE COM URGENCIA, URGENCIA

2010-10-05 Por tôpico Johann Dirichlet
Legal, agora temos spam político na OBM-L...

Em 05/10/10, Luís Junior escreveu:
> Concordo com todos os projetos.
>
> 2010/10/5 Aline Rosane 
>
>>
>>
>>
>>  *PENSE BEM ANTES DE VOTAR NA DONA DILMA E SUA GANG !!!
>> *
>>
>>
>>
>> Cordialmente,
>> Camargo Júnior
>> 69-3421-3061
>>
>>
>>
>> --
>> *Resumo de projetos que podem virar lei após as eleição: ENTÃO VOTE NA
>> DILMA*
>>
>> **
>>
>> *Fica proibido fazer:*
>> ·   Cultos ou evangelismo na rua (Reforma Constitucional)
>> ·   Programas evangélicos na televisão por mais de uma hora por dia.
>> ·   Programa de rádio ou televisão, quem não possuir faculdade
>> de'jornalismo'.
>> ·   Pregar sobre dízimos e ofertas, havendo reclamações, obreiros serão
>> presos.
>>
>> *Quanto aos cultos:*
>> - Cultos somente com portas fechadas (Reforma Constitucional)
>> - As igrejas serão obrigadas a pagarem impostos sobre dízimos,
>> ofertas e contribuições,as católicas também.
>> - Será considerado crime pregar sobre espiritismo, feitiçaria e idolatria,
>> e também veicular mensagem no rádio, televisão, jornais e internet,
>> sobre essas práticas contrárias a Palavra de Deus.
>> - Pastores e Padres que forem presos por pregar sobre práticas condenadas
>> pela
>> Bíblia Sagrada (homossexualismo, idolatria e espiritismo), não terão
>> direito a se defender por meio de ação judicial.
>>
>> *Se estabeleça:*
>> -O dia do “Orgulho Gay” e que seja oficializado em todas as cidades
>> brasileiras e comemorado nas Instituições de Ensino Fundamental
>> (primeira a 8.a série), público e particular.
>> -Que as Igrejas que se negarem a realização das solenidades
>> dos casamentos de homem com homem e de mulher com mulher,
>> estarão fazendo “discriminação”, seja multadas e seus pastores e padres
>> processados criminalmente por discriminação e desobediência civil.
>>
>>
>> *Projeto nº 4.720/03 - Altera a legislação constitucional** **
>>
>> -Projeto nº 3.331/04 –* Altera o artigo 12 da Lei nº 9.250/95, que trata
>> da legislação do imposto de renda das 'pessoas físicas' Se convertidos em
>> Lei, os dois projetos obrigariam as igrejas a recolherem impostos sobre
>> dízimos, ofertas e contribuições.
>>
>>
>> *-**Projeto nº 299/99 –* Altera o código brasileiro de telecomunicações
>> (Lei 4.117/62).
>> Se aprovado, reduziria programas evangélicos
>> no rádio e televisão a apenas uma hora.
>> **
>>
>> *-Projeto nº 6.398/05 –* Regulamenta a profissão de Jornalista
>> Contém artigos que estabelecem que só poderão fazer programas
>> de rádio e televisão, pessoas com formação em JORNALISMO, Significa que
>> pastores,padres e ouros sem a formação em jornalismo não
>> poderão fazer programas através desses meios.
>>
>>
>> *-Projeto nº 1.154/03 –* Proíbe veiculação de programas  em que o teor
>> seja considerado preconceito religioso. Se aprovado, será considerado
>> crime
>> pregar sobre idolatria, feitiçaria e rituais satânicos. Será proibido que
>> mensagens sobre essas práticas sejam veiculadas no rádio, televisão,
>> jornais
>> e internet. A verdade sobre esses atos contrários
>> a Palavra de Deus, não poderá mais ser mostrada.
>>
>>
>> *-Projeto nº 952/03 – *Estabelece que é crime atos religiosos que possam
>> ser considerados abusivos a boa-fé das pessoas. Convertido em Lei, pelo
>> número de reclamações, pastores serão considerados 'criminosos' por
>> pregarem
>> sobre dízimos e ofertas.
>>
>>
>> *-Projeto nº 4.270/04[/b] –** *Determina que comentários feitos contra
>> ações praticadas por grupos religiosos possam ser passíveis de ação civil.
>> Se convertido em Lei, as Igrejas Evangélicas ficariam proibidas de pregar
>> sobre práticas condenadas pela Bíblia Sagrada, como espiritismo,
>> feitiçaria,
>> idolatria e outras. Se o fizerem, não terão direito a se defender por meio
>> de ação judicial.
>>
>>
>> *-**Projeto de nº 216/04[/b] –* Torna inelegível a função religiosa com a
>> governamental. Significa que todo pastor ou líder religioso lançado a
>> candidaturas para qualquer cargo político, não poderá de forma alguma
>> exercer trabalhos na igreja.
>>
>>
>


-- 
/**/
Quadrinista e Taverneiro!

http://tavernadofimdomundo.blogspot.com >> Quadrinhos, histórioas e afins
http://baratoeletrico.blogspot.com />> Um pouco sobre elétrons em movimento
http://bridget-torres.blogspot.com/ >> Personal! Do not edit!

=
Instru��es para entrar na lista, sair da lista e usar a lista em
http://www.mat.puc-rio.br/~obmlistas/obm-l.html
=


[obm-l] Re: [obm-l] Teoria dos números

2010-09-24 Por tôpico Johann Dirichlet
1)
Basta demonstrar que (n^8+1)(n^8-1) é múltiplo de 17.
Mais isso sai direto de Euler-Fermat: 17 divide n^16-1 se n não é
múltiplo de 17.

2)

(2y+1)^2-4=x^3
Escrevendo z=2y-1:
(z-2)(z+2)=x^3
Veja que z-2 e z+2 não tem fatores comuns (ambos são ímpares
consecutivos), logo ambos são cubos perfeitos. Daqui fica fácil
seguir...

Em 23/09/10, warley ferreira escreveu:
> Pessoal seria possível ajuda nestas questoes,
>
> Questão 1)
> Seja n um número natural não divisível por 17. Prove que n^8 + 1 ou n^8 -1 é
> divisível por 17.
> Questão 2)
> Determine, caso existam, as soluções inteiras da equação x^3+3 = 4y(y+1).
>
> Desde já agradeço,
>  Warley Souza
>
>
>


-- 
/**/
Quadrinista e Taverneiro!

http://tavernadofimdomundo.blogspot.com >> Quadrinhos, histórioas e afins
http://baratoeletrico.blogspot.com />> Um pouco sobre elétrons em movimento
http://bridget-torres.blogspot.com/ >> Personal! Do not edit!

=
Instru��es para entrar na lista, sair da lista e usar a lista em
http://www.mat.puc-rio.br/~obmlistas/obm-l.html
=


Re: [obm-l] Para mim, 0^0=1

2010-09-22 Por tôpico Johann Dirichlet
Nessas horas eu me pergunto: por que existem tantas arestas
não-aparadas na matemática?

A aresta mais pontuda, na minha opinião, é o paradoxo de
Banach-Tarski: é possível desmontar uma bolinha de gude e juntar os
pedaços de modo a se obter uma bola do tamanho do sol.

Em 16/09/10, Ralph Teixeira escreveu:
> Eu sou um dos defensores de 0^0=1. Apresento dois motivos:
>
> i) Se f(x) e g(x) sao analiticas em 0 com f(x),g(x)->0 quando x->a, entao
> f^g -> 1 quando x-> a (bom, desde que f^g faca sentido em volta de x=a). A
> *unica* excecao a esta regra eh o caso em que f eh identicamente nula,
> quando o limite dah 0 (se f^g faz sentido) ou nao existe (se g<0 ali por
> perto de x=a).
> Isto explica porque 99.9% dos exercicios de limite que ficam "da forma 0^0"
> acabam dando 1 como resposta!
>
> Acho que isto tambem explica porque eu nao faria 0/0=1 ou algo assim -- nao
> ha teorema semelhante para 0/0.
>
> ii) Como escrever um polinomio generico de grau 17 usando somatorios? Acho
> que muita gente concorda que uma boa representacao eh:
> p(x) = SUM (n=0 a 17) a_n x^n
> onde os a_n sao coeficientes arbitrarios. Agora eu pergunto -- quanto vale
> p(0)?
>
> Com a convencao 0^0=1, nada especial precisa ser feito, eh soh substituir
> x=0 no somatorio.
>
> Com a convencao "0^0 nao existe" bom, ai a nossa representacao por
> somatorio ficaria tecnicamente errada. Teriamos que escrever:
>
> p(x) = SUM (n=0 a 17) a_n x^n, se x<>0
> p(x) = a_0, se x=0
>
> ou entao tirar o x^0 do somatorio:
>
> p(x) = a_0 + SUM (n=1 a 17) a_n x^n
>
> (e se voce acha que esta ultima eh bem razoavel -- escreva p'(x). Separou o
> a1? Argh!)
>
> Como eu nao tenho paciencia de ficar escrevendo este a_0 separado toda hora,
> prefiro logo pensar que 0^0=1 e resolvo meus problemas com um somatorio soh.
> :)
>
> Isto tudo dito, claro que eh soh uma convencao, questao de gosto. Mas eu
> *gosto* de 0^0=1. :)
>
> Abraco,
>  Ralph
>
> 2010/9/16 Jorge Luis Rodrigues e Silva Luis 
>
>> Olá, Pessoal! Vale lembrar que o "símbolo do nada" está entre as mais
>> importantes descobertas feita pelo homem. É difícil acreditar que os
>> homens
>> levaram 5 mil anos entre escrever números e conceber o nosso sistema de
>> numeração posicional, ponto crucial num desenvolvimento sem o qual o
>> progresso da ciência moderna seria inconcebível. Hoje parece simples, mas
>> a
>> mentalidade concreta dos antigos gregos, não podia conceber o vazio, o
>> nada,
>> como um número. Apreciaremos ainda mais a grandeza dessa conquista se
>> lembrarmo-nos de que ela escapou ao gênio de Arquimedes e Apolônio, dois
>> dos
>> maiores homens da antiguidade.
>>
>> Existem situações em Análise Combinatória onde há uma certa conveniência
>> em
>> adotar a regra 0^0=1, a fim de estender um pouco mais o campo de validez
>> de
>> algumas fórmulas. Nem por isso 0^0 deixa de ser uma expressão
>> indeterminada.
>> Um caso parecido acontece na Teoria da Medida e da Integral, onde às vezes
>> é
>> conveniente escrever 0*...=0, a fim de que a fórmula da área de um
>> retângulo
>> continue válida quando a base do "retângulo" é toda uma reta e a altura se
>> reduz a um ponto. O curioso é que os defensores de 0^0=1 não reivindiquem
>> o
>> mesmo direito para 0/0. Algum colega saberia o motivo?
>>
>> Afinal! Qual das medidas é a mais precisa? E a mais exata?  a)5,6m
>> b)560m
>> (com aproximação de 10m)   c) 0,056m   d)5600m (com aproximação de 100m)
>>
>> Quantos algarismos significativos temos nesta medida? X=(0,009050 + -
>> 0,02)
>>
>> A propósito! Como se escreve zero em algarismos romanos?
>>
>>
>> Abraços!
>>
>


-- 
/**/
Quadrinista e Taverneiro!

http://tavernadofimdomundo.blogspot.com >> Quadrinhos, histórioas e afins
http://baratoeletrico.blogspot.com />> Um pouco sobre elétrons em movimento
http://bridget-torres.blogspot.com/ >> Personal! Do not edit!

=
Instru��es para entrar na lista, sair da lista e usar a lista em
http://www.mat.puc-rio.br/~obmlistas/obm-l.html
=


[obm-l] Re: [obm-l] Re: [obm-l] Re: [obm-l] RE: [obm-l] Fato rial via Stirling (confirmação)

2010-09-20 Por tôpico Johann Dirichlet
Em 18/09/10, Bernardo Freitas Paulo da Costa escreveu:
> 2010/9/17 Johann Dirichlet :
>> Bem, vou azedar um pouco a coisa: que tal se pudéssemos isolar o r?
>> n! = [(2.n.pi)^(1/2)].[(n/e)^n].(e^r) se e somente se
>> n!/((2.n.pi)^(1/2).(n/e)^n)=(e^r)
>> Passa o log, temos uma expressão em r.
>> Se pudermos provar a existência deste monstrinho, fechou
>
> Eu acho que a fórmula de Euler-MacLaurin é realmente o que é mais
> adaptado para provar esse tipo de horror (expansão assintótica de
> somas finitas, quando a gente passa aos logs). Tem que estudar, mas
> enfim, você não pode querer demonstrar tudo a partir de nada: a
> matemática se constrói passo a passo...
>
> Enfim, esta observação chata é mais porque, de memória, obter o "raiz
> de 2*pi" na fórmula do fatorial é bem difícil. Se você dispensar
> essa exatidão toda, acho que até dá, inclusive por indução (Johann: já
> achou como corrigir a tua?). Daí, a fórmula fica
> n! = (n/e)^n*raiz(n) * erro(n)

Na verdade eu nem tentei :)
Creio que você esteja certo no "erro" da fórmula. No fim das contas
essa constante é difícil de se obter por indução. A bem da verdade não
conheço nenhum problema de limites que use indução.

>
> onde 0 < min < erro(n) < MAX para duas constantes min e MAX (que a
> gente não calculou)
>
>> Em 17/09/10, Guilherme Vieira escreveu:
>>>
>>> Caro Paulo,
>>> Continuo pensando que não há possibilidade de se obter demonstração por
>>> indução finita, pois r depende de n.
>>> Não sei se há outro modo de confirmar a validade da fórmula.
>>> Continuemos tentando!
>>> Um abraço do Guilherme!
>>>
>>>
>>>
>>> From: argolopa...@hotmail.com
>>> To: obm-l@mat.puc-rio.br
>>> Subject: [obm-l] Fatorial via Stirling (confirmação)
>>> Date: Thu, 16 Sep 2010 20:55:27 +
>>>
>>>
>>>
>>>
>>>
>>> Caros amigos,
>>> Repito a questão a que propus.
>>> Não sei se as respostas já dadas tratam efetivamente da mesma questão.
>>> Fiquei em dúvida.
>>>
>>> Gostaria de obter uma demonstração (pode ser por indução finita) do fato
>>> abaixo, proveniente da fórmula de Stirling.
>>>
>>> Fato:
>>> Para todo número inteiro positivo n, existe um número real r, com
>>> 1/(12n+1)
>>> < r
>>> < 1/(12n), de modo que seja válida a igualdade:
>>> n! = [(2.n.pi)^(1/2)].[(n/e)^n].(e^r)
>>>
>>> Muito obrigado!
>>> Paulo Argolo
>>>
>>>
>>>
>>>
>>>
>>>
>>>
>>>
>>
>>
>> --
>> /**/
>> Quadrinista e Taverneiro!
>>
>> http://tavernadofimdomundo.blogspot.com >> Quadrinhos, histórioas e afins
>> http://baratoeletrico.blogspot.com />> Um pouco sobre elétrons em
>> movimento
>> http://bridget-torres.blogspot.com/ >> Personal! Do not edit!
>>
>> =
>> Instru�ões para entrar na lista, sair da lista e usar a lista em
>> http://www.mat.puc-rio.br/~obmlistas/obm-l.html
>> =
>>
>
>
>
> --
> Bernardo Freitas Paulo da Costa
>
> =
> Instru�ões para entrar na lista, sair da lista e usar a lista em
> http://www.mat.puc-rio.br/~obmlistas/obm-l.html
> =
>


-- 
/**/
Quadrinista e Taverneiro!

http://tavernadofimdomundo.blogspot.com >> Quadrinhos, histórioas e afins
http://baratoeletrico.blogspot.com />> Um pouco sobre elétrons em movimento
http://bridget-torres.blogspot.com/ >> Personal! Do not edit!

=
Instru��es para entrar na lista, sair da lista e usar a lista em
http://www.mat.puc-rio.br/~obmlistas/obm-l.html
=


Re: [obm-l] Qual a melhor "mailing list" internacional de Mathematics

2010-09-20 Por tôpico Johann Dirichlet
Mailing list eu não sei, mas se você aceita um fórum, tem o www.mathlinks.ro.

Em 19/09/10, Rafael escreveu:
> Olá, pessoal.
>
>
> Qual a melhor "mailing list" internacional de Matemática ?
>
>
>
> Regards,
> Rafael


-- 
/**/
Quadrinista e Taverneiro!

http://tavernadofimdomundo.blogspot.com >> Quadrinhos, histórioas e afins
http://baratoeletrico.blogspot.com />> Um pouco sobre elétrons em movimento
http://bridget-torres.blogspot.com/ >> Personal! Do not edit!

=
Instru��es para entrar na lista, sair da lista e usar a lista em
http://www.mat.puc-rio.br/~obmlistas/obm-l.html
=


  1   2   3   4   5   6   7   8   9   10   >